Text
                    И.Е.Иродов
ОСНОВНЫЕ ЗАКОНЫ МЕХАНИКИ
В книге рассмотрены основные законы как ньютоновской (классической), так
и релятивистской механики—законы движения и сохранения импульса, энергии
и момента импульса. На большом количестве примеров и задач показано, как
следует применять эти законы при решении различных конкретных вопросов. В
3-м издании B-е—1978 г.) подробнее рассмотрен вопрос о потенциальной
энергии системы частиц, введено понятие полной механической энергии системы
во внешнем поле, даны условия равновесия твердого тела, приведены примеры на
тему кинематики специальной теории относительности и др.
ОГЛАВЛЕНИЕ
Предисловие	5
Система обозначений	6
Введение	7
Часть I. Ньютоновская механика
Глава 1. Основы кинематики	10
§1.1. Кинематика точки	10
§ 1.2. Кинематика твердого тела	17
§ 1.3. Преобразования скорости и ускорения при переходе к другой системе 25
отсчета
Задачи	28
Глава 2. Основное уравнение динамики	34
§2.1. Инерциальные системы отсчета	34
§ 2.2. Основные законы ньютоновской динамики	38
§ 2.3. Силы	43
§ 2.4. Основное уравнение динамики	45
§ 2.5. Неинерциальные системы отсчета. Силы инерции	49
Задачи	53
Глава 3. Закон сохранения импульса	63
§31.0 законах сохранения	63
§ 3.2. Импульс системы	65
§ 3.3. Закон сохранения импульса	68
§ 3.4. Центр масс, //-система	71
§ 3.5. Движение тела переменной массы	76
Задачи	78
Глава 4. Закон сохранения энергии	84
§4.1. Работа и мощность	84
§ 4.2. Консервативные силы. Потенциальная энергия	89
§ 4.3. Механическая энергия частицы в поле	98
§ 4.4. Потенциальная энергия системы	101
§ 4.5. Закон сохранения механической энергии системы	106
§ 4.6. Столкновение двух частиц	113


Задачи 122 Глава 5. Закон сохранения момента импульса 132 §5.1. Момент импульса частицы. Момент силы 132 § 5.2. Закон сохранения момента импульса 138 § 5.3. Собственный момент импульса 144 § 5.4. Динамика твердого тела 148 Задачи 162 Часть П. Релятивистская механика Глава 6. Кинематика специальной теории относительности 172 § 6.1. Трудности дорелятивистской физики 172 § 6.2. Постулаты Эйнштейна 177 § 6.3. Замедление времени и сокращение длины 181 § 6.4. Преобразования Лоренца 190 § 6.5. Следствия из преобразований Лоренца 193 § 6.6. Геометрическая интерпретация преобразований Лоренца 201 Задачи 203 Глава 7. Релятивистская динамика 209 § 7.1. Релятивистский импульс 209 § 7.2. Основное уравнение релятивистской динамики 213 § 7.3. Закон взаимосвязи массы и энергии 216 § 7.4. Связь между энергией и импульсом частицы 220 § 7.5. Система релятивистских частиц 224 Задачи 230 Приложения 237 1. Движение точки в полярных координатах 237 2. О задаче Кеплера 239 3. Доказательство теоремы Штейнера 240 4. Греческий алфавит 241 5. Основные единицы СИ в механике 241 6. Формулы алгебры и тригонометрии 242 7. Таблица производных и интегралов 243 8. Некоторые сведения о векторах 244 9. Единицы механических величин в СИ и СГС 245 10. Десятичные приставки к названиям единиц 245 1' 1. Некоторые внесистемные единицы 246 12. Астрономические величины 246 13. Физические постоянные 246 Предметный указатель 247 ПРЕДМЕТНЫЙ УКАЗАТЕЛЬ Вектор аксиальный 19 — полярный 19 — перемещения 10 Вес 44
Время собственное 185 Гироскоп 159 Градиент 94 Движение вращательное 18, 150 — плоское 21, 154 — поступательное 17 — тела переменной массы 76 Диаграмма импульсов векторная 118 — Минковского 201 — пространства—времени 201 Длина собственная 188 Задача Кеплера 239 Закон всемирного тяготения 43 — Гука 45 — Ньютона второй 40 первый 35 третий 41 — сохранения импульса 68 момента импульса 140 энергии 100, 109—111 Законы сохранения 63 Замедление времени 185 Изотропность пространства 36 Импульс 65 — момента силы 136 — силы 66 Импульс релятивистский 212 — системы 67 Интервал 197 — времениподобный 198 — пространственного добный 198 — светопо добный 198 Килограмм (эталон) 241 Кинематика твердого тела 17 — точки 10 Координата дуговая 14 Линия мировая 201 Лоренц-фактор 223 Масса 38 — покоя 211 — приведенная 114 — релятивистская 212 Метр (эталон) 241 Момент гироскопический 161 — импульса 134 относительно оси 137 системы 138, 189 собственный 146 — инерции 150 — силы 133 относительно оси 138 Мощность 88 Напряженность поля 96 Одновременность 180, 194 Однородность времени 36 — пространства 36 Опыт Майкельсона 175 Оси тела главные 157 — свободные 157 Ось вращения мгновенная 24 Параметр прицельный 120 Парадокс близнецов 187 Пара сил 144 Плечо импульса 133 — пары 146 — силы'134 Поверхность эквипотенциальная 95 Поле стационарное 89 Порог 121 Постулаты Эйнштейна 177 Потенциал 97 Потенциальный барьер 101 Преобразования Галилея 37, 173 — Лоренца 192 — импульса и энергии релятивистской 223 — скорости 25, 26 релятивистской 199 — ускорения 25—28 Прецессия гироскопа 159 Принцип дальнодействия 42 — относительности Галилея 36 Эйнштейна 177 — суперпозиции 41, 97
— эквивалентности 53 Приращение 6, 92 Работа 85 — при вращение твердого тела 1154 — упругой силы 86 — гравитационной силы 86 — однородной силы тяжести 87 Секунда (эталон) 241 Сила 39 — внешняя 66 — внутренняя 66 — гироскопическая 161 Сила гравитационная 43 — диссипативная 107 — инерции центробежная 50 — консервативная 89, 104 — Кориолиса 50 — кулоновская 44 — Лоренца 231 — нормальная 48 — равнодействующая 149 — реактивная 77 — сопротивления 45 — сторонняя 99, 1111 — тангенциальная 48 — трения 45 — тяжести 44 — упругая 45 — центральная 90 Симметрия пространства и времени 35 Синхронизация часов 179 Система замкнутая 68 — консервативная 109 — отсчета 7 гелиоцентрическая 35 инерциальная 35, 49 — центра масс 75 Скорости космические 127 Скорость точки 15 — угловая 19 прецессии 154 — центра масс 72 Событие 178 События причинно-связанные ,194, 198 Сокращение лоренцево 188,196 Столкновение абсолютно неупругое 115 упругое 116 — лобовое 116 — нелобовое 117 — неупругое 120 — экзоэнергетическое 120 — эндоэнергетическое 120 Схема энергетическая ядерной реакции 234 Теорема Штейнера 146, 231 Точка мировая 201 Убыль 6, 92 Уравнение движения центра масс 73 — Мещерского 77 — моментов 133 в //-системе 147 — релятивистской динамики основное 214 — ньютоновской динамики основное 45 Ускорение 11 — кориолисово 27 — нормальное 16 — осестремительное 27 — тангенциальное 16 — угловое 19 Условия начальные 111 — равновесия твердого тела 150 Центр масс 71 //-система 75 ., Частица с нулевой массой покоя 221 Часы "световые" 184 Энергия внутренняя 113 — кинетическая 98 твердого тела 153, 156 релятивистская 216
Энергия покоя (собственная) 219 — Доплера 206 — пороговая 122 поперечный 207 Энергия потенциальная 91 продольный 206 "внешняя" 105, 111 собственная 102, 103,104, 105 — распада 229 — реакции ядерной 234 Эффект гироскопический 161
Предисловие Цель этой книги — сосредоточить внимание на основных законах механики (законах движения и законах сохранения энергии, импуль- са и момента импульса), а также показать, как следует применять эти законы при решении различных конкретных вопросов. При этом автор стремился помочь студентам, приступающим к изучению физи- ки, начать вырабатывать в себе необходимую для будущего специа- листа культуру физического мышления, а также определенную сме- лость в самостоятельном подходе к решению проблемных задач. Книга содержит две части: 1) ньютоновская механика и 2) ре- лятивистская механика. В первой части законы механики рассмотре- ны в ньютоновском приближении, т. е. при скоростях движения, зна- чительно меньших скорости света, во второй — при скоростях, сравни- мых со скоростью света. В каждой главе сначала излагается теория соответствующего вопроса, а затем на ряде наиболее поучительных и интересных в фи- зическом отношении примеров и задач показывается, как следует подходить к их решению. Задачи (их около 80) тесно связаны с ос- новным текстом, часто являются его развитием и дополнением, и по- этому работа над ними не менее важна, чем изучение основного текста. Разрядкой и курсивом выделены важнейшие положения и терми- ны. Петит используется для примеров и задач, а также для материа- ла повышенной трудности (этот материал при первом чтении можно безболезненно опустить). В настоящем издании сделаны некоторые изменения и добавле- ния. Прежде всего изменена (с целью упрощения) последователь- ность изложения: сначала рассматривается закон сохранения импуль- са, а затем закон сохранения энергии (в предыдущих изданиях было наоборот). В связи с такой перестановкой обе главы пришлось до- вольно существенно переработать. Добавлены новые примеры и за- дачи на закон сохранения импульса, более подробно рассмотрен вопрос о потенциальной энергии системы частиц, введено понятие о полной механической энергии системы, находящейся во внешнем поле, даны условия равновесия твердого тела, приведен ряд приме- ров на кинематику специальной теории относительности и др. Книга как учебное пособие рассчитана в основном на студентов первых курсов вузов с расширенной программой по курсу общей фи- зики. Она может быть полезной и студентам старших курсов, а так- же преподавателям вузов. В заключение считаю своим приятным долгом выразить благо- дарность проф. Л. В. Тарасову за полезные замечания и советы. И. Иродов
Система обозначений Векторы обозначены жирным прямым шрифтом (например, г, F); та же буква светлым шрифтом (г, F) означает модуль вектора. Орты — единичные векторы: i, j, k — орты декартовых координат х, у, г, ер, е?, ez — орты цилиндрических координат р, ф, г, п, т — орты нормали и касательной к траектории. Средние величины заключены в угловые скобки <(), например, <v>. <N > Символы А, A, б перед величинами означают: Д — приращение величины, т. е. разность между ее конечным и на- чальным значениями, например Аг = г2—г4, Д?/=G2—?Л; —Л — убыль величины, т. е. разность между ее начальным и конеч- ным значениями, например, —Ar==ri—г2, —AU=Ui—U2; d — дифференциал (бесконечно малое приращение), например dr, AU; б — элементарное значение величины, например 6А — элементарная работа. Производная по времени от произвольной функции обозначена как d//d^ или точкой, стоящей над функцией (/). Системы отсчета обозначены курсивными буквами К, К' и Ц. Ц-система — это система отсчета, связанная с центром масс и движущаяся поступательно по отношению к инерциальным системам. Все величины в Д-системе помечены сверху значком ~ (тильда), на- пример, р, Е.
Введение Механика — это раздел физики, в котором изучается простейшая форма движения материи — механическое, т. е. движение тел в пространстве и времени. Тот факт, что механические явления протекают в пространстве и времени, находит свое отражение в любом механическом законе, содержащем явно или неявно пространственно- временные соотношения — расстояния и промежутки вре- мени. Положение тела в пространстве может быть определе- но только по отношению к каким-либо другим телам. Это же относится и к движению тела, т. е. к изменению его положения с течением времени. Тело (или система непод- вижных относительно друг друга тел), которое служит для определения положения интересующего нас тела, на- зывают телом отсчета. Практически для описания движения с телом отсчета связывают какую-нибудь систему координат, например декартову. Координаты тела позволяют установить его положение в пространстве. Так как движение происхо- дит не только в пространстве, но и во времени, то для описания движения необходимо отсчитывать также и время. Это делается с помощью часов того или иного типа. Совокупность тела отсчета и связанных с ним коорди- нат и синхронизированных между собой часов образует так называемую систему отсчета. Понятие системы отсчета является фундаментальным в физике. Прост- ранственно-временное описание движения при помощи расстояний и промежутков времени возможно только то- гда, когда выбрана определенная система отсчета. Пространство и время сами являются физическими объектами, как и любые другие, однако неизмеримо бо- лее важными и существенными. Чтобы изучить свойства пространства и времени, нужно наблюдать движение тел, которые в них находятся. Исследуя характер движения тел, мы тем самым познаем и свойства пространства и времени. 7
Опыт показывает, что, пока скорости тел малы по сравнению со скоростью света, линейные масштабы и про- межутки времени остаются неизменными при переходе от одной системы отсчета к другой, т. е. не зависят от вы- бора системы отсчета. Это нашло свое выражение в нью- тоновской концепции абсолютности пространства и вре- мени. Механику, изучающую движения тел именно в этих случаях, называют ньютоновской. При переходе же к скоростям, сравнимым со ско- ростью света, обнаруживается, что характер движения тел радикально меняется. При этом линейные масштабы и промежутки времени уже зависят от выбора системы отсчета и в разных системах отсчета будут разными. Ме- ханику, основанную на этих представлениях, называют релятивистской. Естественно, что релятивистская механика является более общей и в частном случае ма- лых скоростей переходит в классическую. Реальные движения тел настолько сложны, что, изу- чая их, необходимо отвлечься от несущественных для рассматриваемого движения деталей (в противном слу- чае задача так усложнилась бы, что решить ее практиче- ски было бы невозможно). С этой целью используют по- нятия (абстракции, идеализации), применимость кото- рых зависит от конкретного характера интересующей нас задачи, а также от той степени точности, с которой мы хотим получить результат. Среди этих понятий большую роль играют понятия материальной точки и абсолютно твердого тела. Материальная точка — это тело, размерами которого в условиях данной задачи можно пренебречь. Ясно, что одно и то же тело в одних случаях можно рас- сматривать как материальную точку, в других же — как протяженное тело. Абсолютно твердое тело, или, короче, твер- дое тел о,— это система материальных точек, расстоя- ния между которыми не меняются в процессе движения. Реальное тело можно считать абсолютно твердым, если в условиях рассматриваемой задачи его деформации пре- небрежимо малы. Механика ставит перед собой две основные задачи: 1. Изучение различных движений и обобщение полу- ченных результатов в виде законов движения — законов, с помощью которых может быть предсказан характер движения в каждом конкретном случае. 8
2. Отыскание общих свойств, присущих любой систе- ме, независимо от конкретного рода взаимодействий меж- ду телами системы. Решение первой задачи привело к установлению Нью- тоном и Эйнштейном так называемых динамических зако- нов, решение же второй задачи — к обнаружению зако- нов сохранения таких фундаментальных величин, как энергия, импульс и момент импульса. Динамические законы и законы сохранения энергии, импульса и момента импульса представляют собой ос- новные законы механики. Изучение их и составляет со- держание этой книги.
Часть I НЬЮТОНОВСКАЯ МЕХАНИКА Глава 1 ОСНОВЫ КИНЕМАТИКИ Кинематика — это раздел механики, где изучаются способы описания движений независимо от причин, обус- ловливающих эти движения. В этой главе будут рассмот- рены три вопроса: кинематика точки, кинематика твер- дого тела, преобразование скорости и ускорения при пе- реходе от одной системы отсчета к другой. ,^- § 1.1. Кинематика точки Существует три способа описания движения точки: векторный, координатный и так называемый естествен- ный. Рассмотрим их последо- вательно. Векторный способ. В этом способе положение интересу- ющей нас точки А задают радиусом-вектором г, прове- денным из некоторой непо- движной точки О выбранной системы отсчета в точку А. Рис. 1.1 При движении точки А ее радиус-вектор меняется в об- щем случае как по модулю, так и по направлению, т. е. радиус-вектор г зависит от времени /. Геометрическое место концов радиуса-вектора г называют траектори- е й точки А. Введем понятие скорости точки. Пусть за промежу- ток времени At точка А переместилась из точки 1 в точ- ку 2 (рис. 1.1). Из рисунка видно, что вектор переме- щения Аг точки А представляет собой приращение ра- диуса-вектора г за время At: Дг=Гг—п. Отношение Ar/At называют средним вектором скорости 10
<v> за время At. Вектор <v> совпадает по направлению с с Аг. Определим вектор скорости v точки в данный момент времени как предел отношения Ar/At при Д^-^-0, т. е. v = hm = . A.1) м-vo М 6t Это значит, что вектор скорости v точки в данный момент времени равен производной от радиуса-вектора г по вре- мени и направлен по касательной к траектории в данной точке в сторону движения точки А (как и вектор dr). Модуль вектора v равен* Движение точки характеризуется также ускорени- е м. Вектор ускорения а определяет скорость изменения вектора скорости точки со временем: a=dv/dz') A.2) т. е. равен производной от вектора скорости по времени. Направление вектора а совпадает с направлением векто- ра dv — приращением" вектора v за время ut. Модуль век- тора а определяется аналогично модулю вектора v. Пример. Радиус-вектор точки зависит от времени t по закону г = А*+В<2/2, где А и В — постоянные векторы. Найдем скорость v и ускорение а точки: v = dr/cM = A + B*, a = dv/ett = B=const. Модуль вектора скорости Таким образом, зная зависимость r(t), можно найти скорость v и ускорение а точки в каждый момент време- ни. Возникает и обратная задача: можно ли найти v(t) и r(t), зная зависимость от времени ускорения a(t)? Оказывается, для получения однозначного решения этой задачи одной зависимости a(t) недостаточно, необ- ходимо еще знать так называемые начальные усло- * Заметим, что в общем случае |dr|=^dr, где г — модуль радиу- са-вектора г и ифАг/Аг. Например, если г меняется только по на- правлению (точка движется по окружности), то r= const, dr=0, но |dr|,&0. 11
в и я, а именно скорость Vo и радиус-вектор г0 точки в не- который начальный момент i = 0. Чтобы в этом убедиться, рассмотрим простейший случай, когда в процессе дви- жения ускорение точки а = const. Сначала определим скорость точки v(t). Согласно A.2), за промежуток времени cU элементарное прираще- ние скорости dv = adf. Проинтегрировав это выражение по времени от / = 0 до t, найдем приращение вектора ско- рости за это время: t Av= Г ad^=a/. о Но величина Av — это еще не искомая скорость v. Чтобы найти v, необходимо знать скорость vo в начальный мо- мент времени. Тогда v= = vo+Av, или v = vo + at. Аналогично решается вопрос и о радиусе-векторе г(/) точки. Согласно A.1), за промежуток времени сЦ эле- ментарное приращение ради- Рис- 1-2 уса-вектора dr=vdt. Инте- грируя это выражение с уче- там найденной зависимости v(/), определим приращение радиуса-вектора за время от t = Q до t: t Дг=[ v(*)d/ = 6 Для нахождения самого радиуса-вектора r(t) необходи- мо знать еще положение точки г0 в начальный момент времени. Тогда г = го + Дг, или Рассмотрим, например, движение камня, брошенного под некоторым углом к горизонту с начальной скоростью Vo. Если считать, что камень движется с постоянным ус- корением a=g, то его положение относительно точки бросания (го = О) определяется радиусом-вектором т. е. в данном случае г представляет собой сумму двух векторов, что показано на рис. 1.2. 12
Итак, для полного решения задачи о движении точ- ки — определения ее скорости v и положения г в зави- симости от времени — недостаточно знать зависимость аG), но еще необходимо знать и начальные условия, т. е. скорость Vo и положение г0 точки в начальный момент времени. В заключение напомним, что в СИ единицами длины, скорости и ускорения являются соответственно метр (м), метр на секунду (м/с) и метр на секун- ду в квадрате (м/с2). Координатный способ. В этом способе с выбранным телом отсчета жестко связывают определенную систему координат (декартову, косоугольную или криволиней- ную) . Выбор той или иной системы координат определя- ется рядом соображений: характером или симметрией задачи, постановкой вопроса, а также стремлением уп- ростить само решение. Ограничимся здесь* декартовой системой координат х, у, z. Запишем проекции на оси х, у, г радиуса-вектора r(t), характеризующего положение интересующей нас точки относительно начала координат О в момент t: x = x(f); y = y(t); z=z{t). Зная зависимость этих координат от времени — з а ко н движения точки, можно найти положение точки в каждый момент времени, ее скорость и ускорение. Дей- ствительно, спроектировав A.1) и A.2), например, на ось х, получим формулы, определяющие проекции век- торов скорости и ускорения на эту ось: vx=dx/dt, A.3) где dx — проекция вектора перемещения dr на ось х; а,= — — , A.4) х dt &V ' у ' где dvx — проекция вектора приращения скорости dv на ось х. Аналогичные соотношения получаются для у- и г-проекций соответствующих векторов. Из этих формул видно, что проекции векторов скорости и ускорения рав- ны соответственно первой и второй производным коорди- нат по времени. * В приложении 1 рассмотрено движение точки в полярных ко- ординатах. 13
Таким образом, зависимости x(t), y(t), z(t) по суще- ству полностью определяют движение точки. Зная их, можно найти не только положение точки, но и проекции ее скорости и ускорения, а следовательно, модуль и на- правление векторов v и а в любой момент времени. На- пример, модуль вектора скорости направление же вектора v задается направляющими ко- синусами по формулам cosa = ^/w; cos^—Vy/v; cosy = vzlv, где а, р, у — углы между вектором v и осями х, у, z со- ответственно. Аналогичными формулами определяются модуль и направление вектора ускорения. Рис. 1.3 Рис. 1.4 Кроме того, можно решить и ряд других вопросов: найти траекторию точки, зависимость пройденного ею пути от времени, зависимость скорости от положения точки и пр. Решение обратной задачи — нахождение скорости и закона движения точки по заданному ускорению — про- водится, как и в векторном способе, путем интегрирова- ния (в данном случае проекций ускорения по времени), причем задача и здесь имеет однозначное решение, если кроме ускорения заданы еще и начальные условия: про- екции скорости и координаты точки в начальный момент. «Естественный» способ. Этот способ применяют тогда, когда траектория точки известна заранее. Положение точки А определяют дуговой координатой / — расстоянием вдоль траектории от выбранного начала от- 14
счета О (рис. 1.3). При этом произвольно устанавливают положительное направление отсчета координаты / (на- пример, так, как показано стрелкой на рисунке). Движение точки определено, если известны ее траек- тория, начало отсчета О, положительное направление отсчета дуговой координаты / и закон движения точки, т. е. зависимость l(t). Скорость точки. Введем единичный вектор т, связанный с движущейся точкой А и направленный по касательной к траектории в сторону возрастания дуго- вой координаты / (рис. 1.3). Очевидно, что t — перемен- ный вектор: он зависит от /. Вектор скорости v точки А направлен по касательной к траектории, поэтому его можно представить так: v=i>tt, A-5) где vx = dl/dt — проекция вектора v на направление век- тора х, причем vx — величина алгебраическая. Кроме того, Ускорение точ к и. Продифференцируем A.5) по времени: Затем преобразуем последний член этого выражения: dx dx d/ о dx с, dx ,л -,х V. = V. =V2 = V2 . A.7) T At T 61 dt T Al d/ Определим приращение вектора т на участке d/ (рис. 1.4). Можно строго показать, что при стремлении точки 2 к точке / отрезок траектории между ними стремится к дуге окружности с центром в некоторой точке О. Эту точку называют центром кривизны траектории в данной точке, а радиус р соответствующей окружности — радиу- сом кривизны траектории в той же точке. Как видно из рис. 1.4, угол 6а= |d/|/p= |с1т|/1, от- куда причем при d/-»-0 dt±t. Введя единичный вектор п нор- к траектории в точке /, направленный к центру 15
кривизны, запишем последнее равенство в векторном виде: dt/d/=n/p. A.8) Подставим A.8) в A.7) и полученное выражение — в A.6). В результате найдем A.9) Здесь первое слагаемое называют тангенциальным ускорением ат, второе — д ах J^ нормальным ап: A.10) Таким образом, полное ускорение а точки может быть представлено как сум- ма тангенциального и нор- мального ускорений. Модуль полного ускоре- ния точки где v — производная модуля скорости по времени. Пример. Точка А движется по дуге окружности радиусом р (рис. 1.5). Ее скорость зависит от дуговой координаты I по закону v=kyi, где k — постоянная. Найдем угол а между векторами пол- ного ускорения и скорости точки как функцию координаты I. Из рис. 1.5 видно, что угол а можно определить по формуле tg а=а„/а.с. Найдем а„ ив,. Нормальное ускорение В нашем случае vz = v, поэтому тангенциальное ускорение dv 6v &l dv a* ~ ~df = ~dT IF = ~dT V' Учитывая зависимость v В результате t| 16 a,= S a=2//p. от I ~ 2 , получим k kV vr kV
§ 1.2. Кинематика твердого тела Теория движения твердого тела помимо самостоя- тельного значения играет важную роль еще и в другом отношении. С твердым телом, как известно, может быть связана система отсчета, служащая для пространственно- временного описания различных движений. Поэтому изу- чение характера движения твердых тел равносильно, по существу, изучению движений соответствующих систем отсчета. Результаты, которые мы получим в этом пара- графе, будут неоднократно использоваться в дальнейшем. Различают пять видов движения твердого тела: 1) поступательное, 2) вращение вокруг неподвижной оси, 3) плоское движение, 4) движение вокруг неподвижной точки и 5) свободное движение. Первые два движения (поступательное и вращение вокруг неподвижной оси) яв- ляются основными движениями твердого тела. Остальные виды движения твердого тела, оказывается, можно све- сти к одному из основных движений или к их совокупно- сти (это будет показано на примере плоского движения). В данном параграфе будут рассмотрены первые три вида движения и вопрос сложения угловых скоростей. Поступательное движение. Это такое движение твердо- го тела, при котором любая прямая, связанная с телом, все время остается параллельной своему начальному по- ложению. Например, вагон, движущийся по прямому участку пути; кабина колеса обозрения и др. При поступательном движении все точки твердого те- ла совершают за один и тот же промежуток времени равные перемещения. Поэтому скорости и ускорения всех точек тела в данный момент времени одинаковы. Это об- стоятельство позволяет свести изучение поступательного движения твердого тела к изучению движения отдельной точки тела, т. е. к задаче кинематики точки. Таким образом, поступательное движение твердого тела может быть полностью описано, если известны за- висимость от времени радиуса-вектора r(t) любой точки этого тела и положение последнего в начальный момент. Вращение вокруг неподвижной оси. Пусть твердое те- ло, вращаясь вокруг неподвижной в данной системе от- счета оси 00', совершило за время dt бесконечно малый поворот. Соответствующий угол поворота будем харак- теризовать вектором dip, модуль которого равен углу поворота, а направление совпадает с осью 00', причем 17
так, что направление поворота отвечает правилу право- го винта по отношению к направлению вектора d<p (рис. 1.6). Теперь найдем элементарное перемещение любой точ- ки А твердого тела при таком повороте. Положение точ- ки А зададим радиусом-вектором г, проведенным из не- которой точки О на оси вра- щения. Тогда линейное пере- мещение конца радиуса-век- тора г (рис. 1.6) связано с углом поворота d<p соотно- шением |dr|=r sin&dcp, или в векторном виде dr = [dq), Г]. A.11) Отметим, что это равен- ство справедливо лишь для бесконечно малого поворота dcp. Другими словами, толь- ис' ' ко бесконечно малые поворо- ты можно рассматривать как векторы *. Кроме того, введенный нами вектор dtp удовлетворя- ет основному свойству векторов — векторному сложению. В самом деле, представим себе, что твердое тело совер- шает два элементарных поворота dcpi и dcp2 вокруг раз- ных осей, проходящих через неподвижную точку О. То- гда результирующее перемещение dr произвольной точ- ки А тела, радиус-вектор которой относительно точки О равен г, можно представить так: dr = dr1-f dr2=^b r] + [dcp2, r] = [dq)., r], где * Как следует из рис. 1.6, для конечного поворота на угол Дер линейное перемещение точки А Отсюда сразу видно, что перемещение Дг нельзя представить как векторное произведение векторов Д ? и г. Это возможно лишь в слу- чае бесконечно малого поворота d<p, в пределах которого радиус-век- тор г можно считать неизменным,
т. е. два данных поворота (dcpi и d<p2) эквивалентны од- ному повороту на угол d(jp=dcpi + d<p2 вокруг оси, совпа- дающей с вектором dcp и проходящей через точку U. Заметим, что при рассмотрении таких величин, как радиус-вектор г, скорость v, ускорение а, не возникал во- прос о выборе их направления: оно вытекало естествен- ным образом из природы самих величин. Подобные век- торы называют полярными. В отличие от них векторы типа dqp, направление которых связывают с направлением вращения, называют аксиальными. Введем векторы угловой скорости и углового ускоре- ния. Вектор угловой скорости со определяют как co = dcp/d/, A.13) где At — промежуток времени, за который тело совер шает поворот dip. Вектор со совпадает по направле- нию с вектором dcp и представляет собой аксиальный вектор. Изменение вектора со со временем характеризуют вектором углового ускорения р, который определяют как p=dco/d*. A.14) Направление вектора 0 совпадает с направлением dco — приращения вектора со. Вектор р, как и со, является ак- сиальным. Единицей угловой скорости в СИ является радиан в секунду (рад/с), а единицей углового ускорения — радиан на секунду в квадрате (рад/с2). Представление угловой скорости и углового ускоре- ния в виде векторов оказывается чрезвычайно плодо- творным, особенно при изучении более сложных движе- ний твердого тела. Это дает возможность во многих слу- чаях получить большую наглядность, а также резко упро- стить как анализ движения, так и соответствующие расчеты. Запишем выражения для угловой скорости и углового ускорения в проекциях на ось вращения z, положитель- ное направление которой свяжем с положительным на- правлением отсчета координаты ср — угла поворота — правилом правого винта (рис. 1.7). Тогда проекции сог и рг векторов со и р на ось z определяются формулами A.15) A.16) 19
Здесь со2 и р2 — величины алгебраические. Их знак харак- теризует направление соответствующего вектора. На- пример, если (ог>0, то направление вектора ю совпадает с положительным направлением оси z; если же ©z<U, то направление вектора ю противоположно. Аналогично и для углового ускорения. Таким образом, зная зависимость <р(/) —закон вра- щения тела, по формулам A.15) и A.16) (i можно найти угловую скорость и угловое ус- корение в каждый момент времени. И наобо- рот, если известны зависимость углового ус- ч>^ ~^)w корения от времени и начальные условия, т. е. угловая скорость ©о и угол ф0 в началь- D , _ ный момент времени, то можно найти Рис17 «о@иФ(Л. Пример. Твердое тело вращается вокруг неподвижной оси по за- кону ф = д<—Ы2/2, где а и Ъ — некоторые положительные постоян- ные. Найдем характер движения этого тела. Согласно A.15) и A.16), <az = а — Ы; р2 = — b = const. Отсюда видно, что тело, вращаясь равнозамедленно (|32<0), оста- навливается в момент to = a/b, а затем направление вращения (знак Юг) изменяется на противоположное. Отметим, что решение всех задач на вращение твер- дого тела вокруг неподвижной оси аналогично по форме задачам на прямолинейное движение точки. Достаточно заменить линейные величины х, vx и wx на соответствую- щие угловые ф, сог и рг, и мы получим все закономерности и соотношения для вращающегося тела. Связь между линейными и угловыми величинами. Найдем скорость v произвольной точки А твердого тела, вращающегося вокруг неподвижной оси 00' с угловой скоростью ю. Пусть положение точки А относительно не- которой точки О оси вращения характеризуется радиу- сом-вектором г (рис. 1.8). Воспользуемся формулой A.11), поделив ее на соответствующий промежуток вре- мени сМ. Так как dr/d/=v и d<p/di = ro, то A.17) т. е. скорость v любой точки А твердого тела, вращаю- щегося вокруг некоторой оси с угловой скоростью ю, равна векторному произведению ю на радиус-вектор г 20
точки Л относительно произвольной точки О оси враще- ния (рис. 1.8). Модуль вектора A.17) a = cor sinO, или где р —радиус окружности, по которой движется точ- ка Л. Продифференцировав A.17) по времени, найдем пол- ное ускорение а точки Л: а = | или A.18) В данном случае (ось враще- ния неподвижна) р||ю, поэтому вектор [рг] представляет собой тангенциальное ускорение ат. Вектор же [©[юг]] — это нор- мальное ускорение а„. Модули этих ускорений равны: Отсюда модуль полного уско- рения Рис. 1.8 Плоское движение твердого тела. Это такое движение, при котором каждая точка твердого тела движется в пло- скости, параллельной некоторой неподвижной (в данной системе отсчета) плоскости. При этом плоская фигура Ф, образованная сечением тела этой неподвижной плос- костью Р (рис. 1.9), в процессе движения все время оста- ется в этой плоскости, например цилиндр, катящийся по плоскости без скольжения (но конус в подобном случае совершает уже более сложное движение). Нетрудно сообразить, что положение твердого тела при плоском движении одназначно определяется поло- жением плоской фигуры Ф в неподвижной плоскости Р. Это позволяет свести изучение плоского движения твердо- го тела к изучению движения плоской фигуры в ее плос- кости. 21
Пусть плоская фигура Ф движется в своей плоскости Р, неподвижной в /(-системе отсчета (рис. 1.10). Поло- жение фигуры Ф на плоскости можно определить задав радиус-вектор г0 произволь- ной точки О' фигуры и угол Ф между радиусом-векто- ром г', жестко связанным с фигурой, и некоторым фик- сированным направлением в /(-системе отсчета. Тогда плоское движение твердого тела будет описываться дву- мя уравнениями: Рис. 1.9 г0=го@; <Р=?@- Если за промежуток времени ctf радиус-вектор г' точки А (рис. 1.10) повернется на угол dcp, то на такой же угол повернется и любой отрезок, связанный с фигу- рой. Другими словами, поворот фигуры на угол йц> не зависит от выбора точки О'. А это значит, что и угловая скорость © фигуры тоже не зависит от выбора точки О', и мы имеем право называть (о угловой скоростью твердо- го тела как такового. Найдем скорость v произ- вольной точки А тела при плоском движении. Введем вспомогательную /('-систему отсчета, которая жестко свя- зана с точкой О' тела и пере- мещается поступательно от- носительно /(-системы (рис. Рис- ^ло 1.10). Тогда элементарное перемещение dr точки А в /(-системе можно записать в виде dr = dro-|-dr', где dr0 — перемещение /('-системы (точки О'), a dr' — перемещение точки А относительно /('-системы. Переме- щение dr' обусловлено вращением тела вокруг неподвиж- ной в /('-системе оси, проходящей через точку О'; соглас- но A.11), dr' = [d<p, г']. Подставив это выражение в пре- дыдущее и поделив обе части полученного равенства на dt, найдем 22
v = vo + I<or'I, A.19) т. е. скорость любой точки А твердого тела при плоском движении * складывается из скорости v0 произвольной точки О' этого тела и скорости v'={wr'], обусловленной вращением тела вокруг оси, проходящей через точку О'. Подчеркнем еще раз, что v' — это скорость точки А от- носительно поступательно движущейся /('-системы отсче- та, жестко связанной с точкой О'. Рис. 1.11 Рис. 1.12 Иначе говоря, плоское движение твердого тела мож- но представить как совокупность двух основных видов движения — поступательного (вместе с произвольной точкой О' тела) и вращательного (вокруг оси, проходя- щей через точку О'). Покажем, что плоское движение можно свести к чис- то вращательному. Действительно, при плоском движе- нии скорость Vo произвольной точки О' тела перпенди- кулярна вектору и, а это значит, что всегда найдется та- кая точка М, жестко связанная с телом**, скорость которой v=0 в данный момент. Из условия O = vo+|©r'M] можно найти положение точки М, т. е. ее радиус-вектор т'м относительно точки О' (рис. 1.11). Этот вектор пер- пендикулярен векторам о> и Vo, его направление соответ- ствует векторному произведению vo=—[юг'м], а модуль г'м=щ1&. Точка М определяет и положение соответствующей оси (она совпадает по направлению с вектором о>). Дви- жение твердого тела в данный момент времени представ- * Заметим, что формула A.19) оказывается справедливой и для любого сложного движения твердого тела. ** Точка М может оказаться и вне тела, 23
ляет собой чистое вращение вокруг этой оси. Такую ось называют мгновенной осью вращения. Положение мгновенной оси, вообще говоря, меняется со временем. Например, в случае катящегося по плоско- сти цилиндра мгновенная ось в каждый момент совпада- ет с линией касания цилиндра и плоскости. Сложение угловых скоростей. Рассмотрим движение твердого тела, вращающегося одновременно вокруг двух пересекающихся осей. Сообщим некоторому телу враще- ние с угловой скоростью ©' вокруг оси О А (рис. 1.12) и затем эту ось приведем во вращение с угловой скоро- стью ©о вокруг оси ОВ, неподвижной в /(-системе отсче- та. Найдем результирующее движение тела в /(-системе. Введем вспомогательную /('-систему отсчета, жестко связанную с осями ОА и ОВ. Ясно, что эта система вра- щается с угловой скоростью «о, и тело вращается отно- сительно нее с угловой скоростью «'. За промежуток времени cU тело совершит поворот dq/ вокруг оси ОА в /('-системе и одновременно поворот сЦро вокруг оси ОВ вместе с K'-системой. Суммарный по- ворот, согласно A.12), есть d<p = сЗфо + d<p'. Поделив обе части этого равенства на dt, получим « = <»0+о»'. A.20) Таким образом, результирующее движение твердого тела в /(-системе представляет собой чистое вращение с угло- вой скоростью © вокруг оси, совпадающей в каждый мо- мент с вектором а) и проходящей через точку О (рис. 1.12). Эта ось перемещается относительно /(-системы — она поворачивается с угловой скоростью too вместе с осью О А вокруг оси ОВ. Нетрудно сообразить, что даже в том случае, когда угловые скорости а/ и wo не меняются по модулю, тело будет обладать в /(-системе угловым ускорением (i, на- правленным, согласно A.14), за плоскость (рис. 1.12). Вопрос об угловом ускорении твердого тела более под- робно рассмотрен в задаче 1.10. И последнее замечание. Поскольку вектор угловой скорости а) удовлетворяет основному свойству векто- ров — векторному сложению, ю можно представить как векторную сумму составляющих на определенные направ- ления, т. е. ю = (Oi + oJ + ..., где все векторы относятся к одной и той же системе отсчета. Этим удобным и полез- ным приемом часто пользуются при анализе сложного движения твердого тела. 24
§ 1.3. Преобразования скорости и ускорения при переходе к другой системе отсчета Приступая к изучению этого вопроса, напомним, что в рамках ньютоновской механики длина масштабов и вре- мя считаются абсолютными. Любой масштаб одинаков в разных системах отсчета, т. е. не зависит от движения. Это же касается и течения времени, которое также оди- наково во всех системах. Постановка вопро- са. Имеются две произволь- ные системы отсчета К и К', движущиеся определенным образом относительно друг друга. Известны скорость v и ускорение а некоторой точ- ки А в /(-системе. Каковы соответствующие значения v' и а' этой точки в /С'-систе- ме? Рассмотрим последовательно три наиболее важных случая движения одной системы отсчета относительно другой. 1. /('-система движется поступательно по отношению к /(-системе. Пусть в /(-системе начало отсчета /('-систе- мы характеризуется радиусом-вектором г0, а ее скорость и ускорение — векторами Vo и а0. Если положение точки А в /(-системе определяется радиусом-вектором г, а в /('-системе — радиусом-вектором г', то г=го + г' (рис. 1.13). Пусть далее за промежуток времени At точка А совершит в /(-системе элементарное перемещение dr. Это перемещение складывается из перемещения dr0 вмес- те с /('-системой и перемещения dr' относительно Д'-сис- темы, т. е. dr = dro + dr'. Поделив данное выражение на dt, получим следующую формулу преобразования ско- рости: Рис. 1.13 A.21) Продифференцировав A.21) по времени, найдем форму- лу преобразования ускорения: а=а„+а'. A.22) 25 v=vo
Отсюда видно, в частности, что при ао = О а = а', т. е. при движении /('-системы без ускорения относительно К- системы, уско'р?ния точки А в обеих системах отсчета бу- дут одинаковы. 2. К'-система вращается с постоянной угловой скоро- стью © вокруг оси, неподвижной в /С-системе. Рис. 1.14 Возьмем начала отсчета К- и /('-систем в произволь- ной точке О на оси вращения (рис. 1.14, а). Тогда ради- ус-вектор точки А в обеих системах отсчета будет один и тот же: г=г'. Если точка А неподвижна в /('-системе*, то это значит, что ее перемещение dr в /(-системе за время d^ обуслов- лено только поворотом радиуса-вектора г на угол-cUp (вместе с /('-системой) и равно, согласно A.11), вектор- ному произведению {d<p, r]. Если же точка А движется относительно /('-системы со скоростью v', то за время dt она совершит дополни- тельное перемещение v'd^ (рис. 1.14, а) и тогда dr = v'(«+[d<p, г]. A.23) Поделив это выражение на dt, получим следующую фор- мулу преобразования скорости: v=v'-f[«r], A.24) где v и v' — скорости точки А в К- /('-системах отсчета соответственно. Теперь перейдем к ускорениям. В соответствии с A.24) приращение dv вектора v за время d^ в /(-системе 26
должно складываться из суммы приращений векторов v' и [юг], т. е. dv=dv' + [o>, dr]. A.25) Найдем dv'. Если точка А движется в /('-системе с v' = = const, то приращение этого вектора в /(-системе обус- ловлено только его поворотом на угол d<p (вместе с К'- системой) и равно, как и в случае с г, векторному произ- ведению [d<p, v']. В этом нетрудно убедиться, совместив начало вектора v' с осью вращения (рис. 1.14, б). Если же точка А имеет ускорение а' в /('-системе, то за время dt вектор v' получит еще дополнительное приращение &'&t и тогда dv' = a\d/ + [d<f, v']. A.26) Подставим A.26) и A.23) в равенство A.25) и получен- ное выражение разделим на dt. В результате найдем следующую формулу преобразования ускорения: a=a' + 2[wv'] + [©["Tl], A.27) где а и а' — ускорения точки А в К- и /('-системах отсче- та. Второе слагаемое в правой части этой формулы но- сит название кориолисова (или поворотного) уско- рения аКОр, а третье слагаемое — осестремительно- го ускорения * аос: A.28) Таким образом, ускорение а точки относительно К- системы равно сумме трех ускорений: ускорения а' отно- сительно /('-системы, кориолисова ускорения акор и осе- стремительного ускорения аос. Осестремительное ускорение можно представить в ви- де аос = —со2р, где р — радиус-вектор, перпендикулярный оси вращения и характеризующий положение точки А относительно этой оси. Тогда формулу A.27) можно за- писать так: 3. К'-система вращается с постоянной угловой скоро- стью со вокруг оси, перемещающейся поступательно со скоростью vo и ускорением а0 по отношению к К-системе. * Осестремительное ускорение не следует путать с нормальным ускорением. 27
Этот случай объединяет два предыдущих. Введем вспомогательную 5-систему отсчета, которая жестко свя- зана с осью вращения /С'-системы и перемещается посту- пательно в К-системе. Пусть v и Vs — скорости точки А в К- и 5-системах отсчета, тогда в соответствии с A.21) v=vo + Vs. Заменив vs, согласно A.24), выражением vs = = v'+[©r], где г — радиус-вектор точки Л относительно произвольной точки на оси вращения /('-системы, полу- чим следующую формулу преобразования скорости: A.30) Аналогичным образом, используя A.22) и A.29), найдем формулу преобразования ускорения: Напомним, что в последних двух формулах v, v' и а, а'—• скорости и ускорения точки А соответственно в К- и К1 - системах отсчета, vo и ао — скорость и ускорение оси вращения /('-системы в /(-системе, г — радиус-вектор точки А относительно произвольной точки на оси вра- щения /('-системы, р —радиус-вектор, перпендикулярный оси вращения и характеризующий положение точки Л относительно этой оси. Рассмотрим в заключение следующий пример. Пример. Диск вращается с постоянной угловой скоростью ю во- круг собственной оси, укрепленной на столе. По диску движется точ- ка Л с постоянной относительно стола скоростью v. Найдем скорость v' и ускорение а' точки А относительно диска в момент, когда ра- диус-вектор, характеризующий ее положение по отношению к оси вращения, равен р Скорость v' точки А, согласно A.24), v' = v — [юр]. Ускорение же а' найдем с помощью A.29), учтя, что в данном слу- чае а=0, ибо v = const. Тогда а' = —2[«v']+o2p. После подстановки в эту формулу выражения для v' получим а' = 2 [vw] — ев2 р. Задачи 01.1. Радиус-вектор, характеризующий положение частицы М относительно неподвижной точки О, меняется со временем по закону r=Asinwt-f Bcos(o<, где А и В — постоянные векторы, причем AJ_B; <о — положительная постоянная. Найти ускорение а частицы и уравнение ее траектории у(х), взяв оси х и у совпадающими по 28
направлению с векторами А и В соответственно и имеющими начало в точке О. Решение. Продифференцировав г по времени дважды, по- лучим а = — uJ (A sin <at + В cos at) = — ш2г, т. е. вектор а все время направлен к точке О, а его модуль пропор- ционален расстоянию частицы до этой точки. Теперь найдем уравнение траектории. Спроецировав г нз оси х и у, получим Исключив at из этих двух уравнений, найдем Это уравнение эллипса, А и В — его полуоси (рис 1.15, где стрелкой показано направление движения частицы М) \лх Рис. 1.15 Рис. 1.16 #1.2. Перемещение и путь. Частице в момент <=0 сообщили скорость v0, после чего ее скорость стала меняться со временем t по закону v=vo(l—1/%), где т — положительная постоянная. Найти за первые t секунд движения: 1) вектор перемещения Лг частицы; 2) пройденный ею путь s. Решение. 1. Согласно A.1), dr=vd;=v0 A—t/i)dt. Проин- тегрировав это уравнение по времени от 0 до t, получим 2. Путь s, пройденный частицей за время t, определяется как t s = где v — модуль вектора v. В данном случае .1 *, i (wo0—*/*). если г <ч>» 29
Отсюда следует, что при />т интеграл для вычисления пути необхо- димо разбить на две части: от 0 до т и от т до t. Проведя интегри- рование для обоих случаев, получим < vot A — t/2x), если /<t, — \i/2i/ot[1+A —*/tJ], если t>x. На рис. 1.16 показаны графики зависимостей v(t) и s(t). Здесь же штриховыми линиями показаны графики зависимостей от t проек- ций Их и Ах векторов v и Дг на ось х, направленную вдоль векто- ра v0. ф1.3. Трамвай движется прямолинейно от остановки А до сле- дующей остановки В с ускорением, меняющимся по закону а=а0— —6s, где а0 и 6 — положительные постоянные, s — расстояние от остановки А до трамвая. Найти расстояние между этими остановка- ми и максимальную скорость трамвая. Решение. Сначала найдем зависимость скорости от расстоя- ния s. За промежуток времени dt приращение скорости dv = adt. Приведем это выражение к виду, удобному для интегрирования, вос- пользовавшись тем, что dt = ds/v; тогда v dv = (а0— bs) ds. Проинтегрировав это уравнение (левую часть — от 0 до v, правую — от нуля до s), получим = а0 s — bs2/2, или v = у Bа0 — bs) s. Отсюда видно, что расстояние между остановками, т е. значение So, при котором и = 0, есть so = 2a0/^- Максимальную же скорость найдем из условия dt>/ds = O или, проще, из условия максимума под- коренного выражения. Отсюда значение sm, соответствующее иМакс, определяется как sm = aojb, и УМакс = ао/У&. ф1.4 Частица движется в плоскостях х, у из точки с координа- тами х=(/ = 0 со скоростью \ = а\ + Ьх], где а и b — некоторые по- стоянные, i и j — орты осей х и у. Найти уравнение ее траектории У(х). Решение. Запишем приращения у- и х-координат частицы за промежуток времени dt: dy =vy dt, dx =vx dt, где vv = bx, vx = a. Взяв их отношение, получим йу = (b/ а) х их. Интегрируем это уравнение: X У = т. е. траекторией точки является парабола. 01.5. Закон движения точки А обода колеса, катящегося равно- мерно по горизонтальному пути (ось х), имеет вид 30
где 6 ив — положительные постоянные. Кайти скорость v точки А, путь s, пройденный ею между двумя последовательными касаниями полотна дороги, а также модуль и направление вектора ускорения а точки А. Решение. Скорость v точки А и пройденный ею путь s опре- деляются формулами = but y^2(l — CGSwt) =26ю| sin I = 46 [ 1 — ее •{'¦" где ti — промежуток времени между двумя последовательными каса- ниями. Из уравнения y=y(t) находим, что y(ti)=O при w/i=2n. Поэтому s = 86. Ускорение точки А * = ]/4 Покажем, что вектор а, постоянный по модулю, все время направлен к центру колеса — точке С. Действительно в /С'-системе отсчета, свя- занной с точкой С и перемещающейся поступательно и равномерно относительно полотна дороги, точка А движется равномерно по окружности с центром в точке С. Поэтому ускорение точки А в К'- системе направлено к центру колеса. А так как /С'-система движется равномерно, то вектор а будет таким же и относительно полотна дороги. #16. Тангенциальное и нормальное ускорения. Точка движется замедленно по окружности радиуса г так, что ее тангенциальное и нормальное ускорения в каждый момент равны друг другу по моду- лю. В начальный момент точке была сообщена скорость у<>. Найти скорость v и модуль полного ускорения а точки в зависимости от пройденного пути s. Решение. По условию, dvjdt=—v2/r. Представив dt как dsfv, преобразуем исходное уравнение к виду dv/v = — ds/r. Интегрирование этого уравнения с учетом начальной скорости при- водит к следующему результату: В данном случае |ат \=ап, поэтому полное ускорение а=У2 а„ = =У2 vzjr, или #1.7. Точка движется по плоской траектории так, что ее таигенци- чльное ускорение ах =flo, а нормальное ускорение an=W, где а0 и Ъ — положительные постоянные, t — время. В момент ^=0 точка на- чала двигаться с нулевой начальной скоростью. Найти радиус кри- визны р траектории точки и ее полное ускорение а в зависимости от пройденного пути s. 31
Решение. Элементарное приращение скорости точки dv = a^ut. Проинтегрировав это уравнение, получим v = aat. Пройденный путь s=aot2/2. Радиус кривизны траектории, согласно A.10), можно предста- вить как p = y2/an = ao2/&/2, или Полное ускорение а=Уа2х + а1 = а0 ф1.8. Частица движется равномерно со скоростью v по парабо- лической траектории (/ = ?л:2, где k-—положительная постоянная. Найти ускорение а частицы в точке х = 0. Решение. Продифференцируем дважды уравнение траектории по времени: y=2kxx, у =2k(x2 + х'х). Так как частица движется равномерно, то это значит, что ее ускоре- ние во всех точках траектории чисто нормальное и в точке х=0 совпадает с производной у в этой точке. Имея в виду, что в точке х=0 величина jxj = f, получим Заметим, что в приведенном способе решения мы обошли вычисле- ние радиуса кривизны траектории в точке х=0, который обычно бывает необходимо знать для определения нормального ускорения (an = v4p). #1.9. Вращение твердого тела. Твердое тело вращается вокруг неподвижной оси с угловым ускорением Р = Ро cos ф, где Ро—по- стоянный вектор, ф — угол поворота тела из начального положения. Найти угловую скорость wz тела в зависимости от угла ф, если при <р—0 она была равна нулю. Решение Выберем положительное направление осн г вдоль вектора Ро Согласно A.16), dto2 = PzcW. Представив At по формуле A.15) как с1ф/со2, преобразуем предыдущее уравнение к виду Интегрирование этого уравнения с учетом начального условия (аг = = 0 при ф = 0) дает (o22/2=j3o sin ф. Отсюда <*г = ± }^2posin9 • График зависимости a>z(<f) показан на рис. 1.17. Из него видно, что с ростом угла ф вектор ю сначала увеличивается, совпадая по на- правлению с вектором Ро(иг>О), достигает максимума при ф=я/2 и затем начинает уменьшаться, обращаясь в нуль при <р=я. После этого тело подобным же образом начинает вращаться в противопо- ложном направлении (о)г<0). В результате тело будет совершать колебания около положения ф=я/2 с амплитудой, равной я/2. 01.10. Круглый конус с радиусом основания г и высотой h ка- тится без скольжения по поверхности стола, как показано на рис. 32
1.18. Вершина конуса закреплена шарннрно в точке О на уровне точ- ки С — центра основания конуса. Точка С движется с постоянной скоростью и. Найти относительно стола: 1) угловую скорость о> ко- нуса; 2) его угловое ускорение р. Решение. 1. Согласно A.20), о> = о>о + о>', где о>о и <о' — угловые скорости вращения вокруг осей 00' и ОС соответственно. Модули векторов Щ и о>' легко найти с помощью рис. 1.18: uiQ = v/h, <o' = v] r. их отношение (uo/w'=r//i. Отсюда следует, что вектор ю совпадает в каждый момент с образующей конуса, которая проходит через точ- ку касания А. Рис. 1.17 Рис. 1.18 Модуль вектора й) -/-8 + О)'2 = + 07ЛJ. 2. Угловое ускорение Р конуса, согласно A.14), есть производ- ная вектора ** по времени. Так как Щ == const, то Вектор се', оставаясь постоянным по модулю, поворачивается вокруг оси 00' с угловой скоростью ft»o . Его приращение за промежуток времени At равно по модулю ]dto' |=ш'-соосМ, или в векторном виде d«/ = [ft)oft»' ]dt. Таким образом, P = [wow']. Модуль этого вектора § = v2lrh. 01.11. Преобразования скорости и ускорения. Горизонтально рас- положенный стержень вращается с постоянной угловой скоростью вокруг вертикальной оси, укрепленной на столе и проходящей через один из концов стержня. По стержню движется небольшая муфта. Ее скорость относительно стержня меняется по закону v'=6r, где Ъ — постоянная, г — радиус-вектор, характеризующий расстояние муфты от оси вращения. Найти: 1) скорость v и ускорение а муфты относительно стола в зависимости от г; 2) угол между векторами v и а в процессе движения. 33
Решение. 1. Согласно A.24), v = bt + [wr]. Модуль этого вектора v = г } Ускорение а находим по формуле A.29), где в нашем случае a'=dv'/d^=62r. Тогда а = F2 _ №2) г + 26 [юг]. Модуль этого вектора а=F2+со2)г. 2. Для определения угла а между векторами v и а воспользуем- ся их скалярным произведением, из которого следует, что cosa = = \a/va. После соответствующих преобразований получим cos <х= Отсюда видно, что в данном случае угол а остается постоянным при движении. Глава 2 ОСНОВНОЕ УРАВНЕНИЕ ДИНАМИКИ § 2.1. Инерциальные системы отсчета Закон инерции. В кинематике, где речь идет лишь об описании движений и не затрагивается вопрос о причи- нах, вызывающих эти движения, никакой принципиаль- ной разницы между различными системами отсчета нет, и все они в этом отношении равноправны. Совершенно иначе обстоит дело в динамике — при изучении законов движения. Здесь обнаруживается существенное различие между разными системами отсчета и преимущества од- ного класса систем отсчета по сравнению с другими. В принципе можно взять любую из бесчисленного мно- жества систем отсчета. Однако законы механики в раз- ных системах отсчета имеют, вообще говоря, различный вид и может оказаться, что в произвольной системе от- счета законы даже совсем простых явлений будут весь- ма сложными. Естественно, возникает задача отыскания такой системы отсчета, в которой законы механики были бы возможно более простыми. Такая система отсчета, оче- видно, наиболее удобна для описания механических яв- лений. Для решения этой задачи рассмотрим ускорение мате- риальной точки относительно некоторой произвольной системы отсчета. Какова причина этого ускорения? Опыт показывает, что этой причиной могут быть как дей- ствие на данную точку каких-то определенных тел, так 34
и свойства самой системы отсчета (действительно, отно- сительно разных систем отсчета ускорение в общем слу- чае будет различным). Можно, однако, предположить, что существует такая система отсчета, в которой ускорение материальной точ- ки целиком обусловлено только взаимодействием ее с другими телами. Свободная материальная точка, не под- верженная действию никаких других тел, движется отно- сительно такой системы отсчета прямолинейно и равно- мерно, или, как говорят, по инерции. Такую систему от- счета называют и н е р ц и а л ь н о й. Утверждение, что инерциальные системы отсчета су- ществуют, составляет содержание первого закона механики — закона инерции Галилея — Ньютона. Существование инерциальных систем отсчета подт- верждается опытом. Первоначальными опытами было ус- тановлено, что такой системой отсчета является Земля. Последующие более точные опыты (опыт Фуко и все ана- логичные ему) показали, что эта система отсчета не сов- сем инерциальная *, а именно: были обнаружены ускоре- ния, существование которых нельзя объяснить действием каких-либо определенных тел. В то же время наблюде- ния над ускорениями планет показали инерциальность гелиоцентрической системы отсчета, связанной с цент- ром Солнца и «неподвижными» звездами. В настоящее время инерциальность гелиоцентрической системы отсче- та подтверждается всей совокупностью опытов. Любая другая система отсчета, движущаяся равно- мерно и прямолинейно относительно гелиоцентрической системы, является также инерциальной. Действительно, если в гелиоцентрической системе отсчета ускорение те- ла равно нулю, то оно равно нулю и в любой другой из этих систем отсчета. Таким образом, существует не одна, а бесчисленное множество инерциальных систем отсчета, движущихся относительно друг друга прямолинейно и равномерно. Си- стемы отсчета, движущиеся с ускорением относительно инерциальных систем, называют неинерциальными. О свойствах симметрии пространства и времени. Важной особен- ностью инерциальных систем отсчета является то, что по отношению к ним пространство и время обладают определенными свойства- * Заметим, что во многих случаях систему отсчета, связанную с Землей, можно считать практически инерциальной. 35
ми симметрии. А именно: опыт убеждает, что в этих системах отсчета пространство однородно и изотропно, а время однородно. Однородность и изотропность пространства заключаются в том, что свойства пространства одинаковы в различных точках (однород- ность), а в каждой точке одинаковы во всех направлениях (изотроп- ность). Однородность времени заключается в том, что протекание физи- ческих явлений (в одних и тех же условиях) в разное время их на- блюдения одинаково. Иначе говоря, различные моменты времени эквивалентны друг другу по своим физическим свойствам. Заметим, что по отношению к неинерциальным системам отсчета пространство является неодно- родным и неизотропным. Это значит, что если какое-либо те- ло не взаимодействует ни с ка- кими другими телами, то тем не менее его различные поло- жения в пространстве и его различные ориентации в меха- ническом отношении не эквива- лентны. То же самое относится в общем случае и ко времени, —1 которое будет неоднородным (в неинерциальных системах), т. е. его различные моменты не эквивалентны. Ясно, что такие Рис. 2.1 свойства пространства и време- ни вносили бы большие услож- нения в описание механических явлений. Так, например, тело, не под- верженное действию со стороны других тел, не могло бы покоиться: если «го скорость в некоторый момент времени и равна нулю, то уже в следующий момент тело начало бы двигаться в определенном на- правлении. Принцип относительности Галилея. Для инерциаль- ных систем отсчета справедлив принцип относительно- сти, согласно которому все инерциальные системы по сво- им механическим свойствам эквивалентны друг другу. Это значит, что никакими механическими опытами, про- водимыми «внутри» данной инерциальной системы, нель- зя установить, покоится эта система отсчета или движет- ся. Во всех инерциальных системах отсчета свойства пространства и времени одинаковы, одинаковы также и все законы механики. Данное утверждение составляет содержание прин- ципа относительности Галилея — одного из важнейших принципов ньютоновской механики. Этот принцип является обобщением опыта и подтверждается всем многообразием приложений ньютоновской механи- ки к движению тел, скорости которых значительно мень- ше скорости света. 36
Все сказанное достаточно ясно свидетельствует об исключительности свойств инерциальных систем отсчета, в силу которых именно эти системы должны, как пра- вило, использоваться при изучении механических яв- лений. Преобразования Галилея. Найдем формулы преобра- зования координат при переходе от одной инерциальной системы отсчета к другой. Пусть инерциальная система К' движется со скоростью V относительно другой инер- циальной системы К. Выберем оси координат хг, у', z' /('-системы параллельно соответствующим осям х, у, z /(-системы так, чтобы оси х' и х совпадали между собо? и были направлены вдоль вектора V (рис. 2.1). Взяв за начало отсчета времени момент, когда начала координат О' и О совпадали, запишем соотношение между радиуса- ми-векторами г' и г одной и той же точки А в К'- и К- системах: r'=r-W B.1) и, кроме того, t'=t. B.2) Здесь подразумевается, что длина отрезков и ход вре- мени не зависят от состояния движения и, следовательно, одинаковы в обеих системах отсчета. Предположение об абсолютности пространства и времени лежит в самой ос- нове представлений ньютоновской механики, представле- ний, основанных на обширном экспериментальном мате- риале, относящемся к изучению движений со скоростя- ми, значительно меньшими скорости света. Соотношения B.1) и B.2) представляют собой так называемые преобразования Галилея. В коор- динатах эти преобразования имеют вид: B.3) Продифференцировав B.1) по времени, найдем клас- сический закон преобразования скорости точки при пе- реходе от одной инерциальной системы отсчета к другой: B.4) Дифференцируя это выражение по времени с учетом того, что V=const, получаем а'=а, т. е. ускорение точ- ки одинаково во всех инерциальных системах отсчета. 37
§ 2.2. Основные законы ньютоновской динамики Изучая на опыте различные движения, мы обнаружи- ваем, что в инерциальных системах отсчета всякое уско- рение тела вызывается действием на него каких-либо других тел. При этом степень влияния (действия) каж- дого из окружающих тел на состояние движения интере- сующего нас тела А— это вопрос, на который в каждом конкретном случае может дать ответ только опыт. Влияние другого тела (или тел), вызывающее уско- рение тела Л, называют силой. Итак, причиной ускоре- ния тела является действующая на него сила. Одной из важнейших характеристик силы является ее материальное происхождение. Говоря о силе, мы все- гда неявно предполагаем, что в отсутствие посторонних тел сила, действующая на интересующее нас тело, равна нулю. Если же обнаруживается, что сила действует, мы ищем ее источник в виде того или иного конкретного те- ла или других тел. Все силы, с которыми имеет дело механика, обычно подразделяют на силы, возникающие при непосредствен- ном контакте тел (силы давления, трения), и силы, воз- никающие через посредство создаваемых взаимодейству- ющими телами полей (силы гравитационные, электро- магнитные). Заметим, однако, что такое подразделение сил имеет условный характер: в сущности и при непос- редственном контакте силы взаимодействия обусловле- ны также наличием тех или иных полей, создаваемых мо- лекулами или атомами тел. Таким образом, все силы взаимодействия между телами обусловлены в конечном счете полями. Вопрос о природе сил взаимодействия вы- ходит за рамки механики и рассматривается в других разделах физики. Масса. Опыт показывает, что всякое тело «оказывает сопротивление» при любых попытках изменить его ско- рость— как по модулю, так и по направлению. Это свой- ство, выражающее степень неподатливости тела к изме- нению его скорости, называют инертностью. У раз- личных тел оно проявляется в разной степени. Мерой инертности служит величина, называемая массой. Те- ло с большей массой является более инертным, и наобо- рот. Введем понятие массы т, определив отношение масс двух различных тел по обратному отношению ускорений, сообщаемых им равными силами: 38
B.5) Отметим, что такое определение не требует предвари- тельного измерения самих сил. Достаточно лишь распо- лагать критерием равенства сил. Например, если на два различных тела, лежащих на гладкой горизонтальной плоскости, последовательно подействовать одной и той же пружиной, ориентировав ее горизонтально и растянув на одну и ту же длину, то можно утверждать, что в обо- их случаях влияние пружины на каждое тело одинаково, другими словами, одинакова и сила. Таким образом, сравнение масс двух тел, на которые действует одна и та же сила, сводится к сравнению ус- корений этих тел. Взяв некоторое тело за эталон массы, мы имеем возможность сравнить массу любого тела с этим эталоном. Единицей массы в СИ является, как известно, кило- грамм (кг). Как показывает опыт, в рамках ньютоновской меха- ники масса обладает следующими двумя важнейшими свойствами: 1) масса—величина аддитивная, т. е. масса составно- го тела равна сумме масс его частей; 2) масса тела как такового — величина постоянная, не изменяющаяся при его движении. Сила. Вернемся к опыту по сравнению ускорений двух различных тел под действием одинаково растянутой пру- жины. Тот факт, что в обоих случаях пружина была рас- тянута одинаково, позволил нам высказать утверждение об одинаковости действия пружины, или силы со сторо- ны пружины. С другой стороны, сила является причиной ускоре- ния тела. Ускорения же различных тел под действием од- ной и той же одинаково растянутой пружины разные. На- ша задача так определить силу, чтобы, несмотря на раз- личие ускорений разных тел в рассматриваемом опыте, сила была бы одной и той же. Для этого прежде всего надо выяснить: что является одинаковым в данных опытах? Ответ очевиден: произ- ведение та. Эту величину и естественно взять за опреде- ление силы. Учитывая, что ускорение — вектор, будем считать и силу вектором, совпадающим по направлению с вектором ускорения а. Итак, в ньютоновской механике сила, действующая на тело массы т, определяется как произведение та. 39
Оправданием именно такого определения силы кроме со- ображений наибольшей простоты и удобства послужила дальнейшая проверка всех вытекающих из него следст- вий. Второй закон Ньютона. Изучая на опыте взаимодей- ствие различных материальных точек с окружающими телами, мы обнаруживаем, что та. зависит от величин, характеризующих как состояние самой материальной точки, так и состояние окружающих тел. Это является весьма существенным физическим фак- том, лежащим в основе одного из наиболее фундамен- тальных обобщений ньютоновской механики: произведение массы материальной точки на ее ускоре- ние является функцией положения этой точки относитель- но окружающих тел, а иногда также и функцией ее ско- рости. Эту функцию обозначают F и называют силой. Именно в этом и состоит фактическое содержание второго закона Ньютона, который кратко формулируют обычно так: произведение массы материальной точки на ее ускорение равно действующей на нее силе, т. е. B.6) Это уравнение называют уравнением движения мате- риальной точки. Сразу же подчеркнем, что второй закон Ньютона и уравнение B.6) получают конкретное содержание только после того, как установлен вид функции F — зависимость от определяющих ее величин, или, как говорят, закон силы. Установление вида этой зависимости в каждом конкретном случае является одной из основных задач физической механики. Определение силы как та, лежащее в основе уравнения B.6), обладает тем исключительным достоинством, что законы сил при этом оказываются очень простыми. Правда, переход к изучению дви- жений с релятивистскими скоростями показал, что законы сил потре- бовалось бы модифицировать, сделав их сложным образом завися- щими от скорости материальной точки. Теория стала бы громоздкой и запутанной. Существует, однако, простой выход из этого затруднения, если дать несколько иное определение силы, а именно: сила есть произ- водная импульса р материальной точки по времени, т. е. dp/d?, и уравнение B.6) записывать в виде 40
В ньютоновской механике это определение силы тождественно та, ибо p = mv, m = const и dp/d< = ma. В релятивистской же меха- нике импульс, как мы увидим, зависит от скорости материальной точки более сложным образом. Но важно другое. При таком опреде- лении силы (как dpldt) законы сил, оказывается, остаются теми же и в релятивистской области. Так что простое выражение данной си- лы через физическое окружение изменять не потребуется при перехо- де к релятивистской механике. Это обстоятельство мы учтем в даль- нейшем, с . _ssd Единицей силы в СИ является ньютон (Н). Нью- тон— это такая сила, которая сообщает телу массой 1 кг ускорение 1 м/с2. О сложении сил. На всякую материальную точку в данных конкретных условиях действует, строго говоря, всего только одна сила F, модуль и направление кото- рой определяются расположением этой точки относи- тельно всех окружающих тел, а иногда также и ее ско- ростью. И тем не менее часто бывает удобно эту силу F представлять как суммарный результат действия отдель- ных тел, или сил Fb F2, ... . Опыт показывает, что если тела, являющиеся источниками сил, не влияют друг на друга и поэтому не меняют своего состояния от присут- ствия других тел, то сила где F,- — сила, с которой действовало бы на данную ма- териальную точку t-e тело в отсутствие других тел. Если это так, то говорят, что силы Fb F2, ... подчи- няются принципу суперпозиции. Такое утвер- ждение надо рассматривать как обобщение опытных фактов. Третий закон Ньютона. Во всех случаях, когда в опы- те участвуют только два тела Л и Б и тело А сообщает ускорение телу В, обнаруживается, что и тело В сообща- ет ускорение телу А. Отсюда мы заключаем, что дейст- вия тел друг на друга имеют характер взаимодейст- вия. Ньютон постулировал следующее общее свойство всех сил взаимодействия — третий закон Ньютона: силы, с которыми две материальные точки действуют друг на друга, всегда равны по модулю и направлены в противоположные стороны вдоль прямой, соединяющей эти точки, т. е. рц^-Га. | B.7) 41
Это значит, что силы взаимодействия всегда появляются парами. Обе силы приложены к разным материальным точкам и, кроме того, являются силами одной природы. Закон B.7) распространяется на системы из произ- вольного числа материальных точек. Мы исходим из пред- ставления, что и в этом случае взаимодействие сводится к силам попарного взаимодействия между материаль- ными точками. В третьем законе Ньютона предполагается, что обе силы равны по модулю в любой момент времени незави- симо от движения точек. Это утверждение соответствует ньютоновскому представлению о мгновенном распрост- ранении взаимодействий — предположению, которое но- сит название принципа дальнодействия нью- тоновской механики. Согласно этому принципу, взаимо- действие между телами распространяется в пространстве с бесконечно большой скоростью. Иначе говоря, если из- менить положение (состояние) одного тела, то сразу же можно обнаружить хотя бы очень слабое изменение во взаимодействующих с ним телах, как бы далеко они ни находились. В действительности это не так — существует конечная максимальная скорость распространения взаимодейст- вий, которая равна скорости света в вакууме. Поэтому третий закон Ньютона (а также и второй) имеет опреде- ленные пределы применимости. Однако при скоростях тел, значительно меньших скорости света, с которыми имеет дело ньютоновская механика, оба закона выпол- няются с очень большой точностью. Свидетельством это- му являются хотя бы расчеты траекторий планет и ис- кусственных спутников, которые проводятся с «астроно- мической» точностью именно с помощью законов Нью- тона. Законы Ньютона являются основными законами ме- ханики. Они позволяют, по крайней мере в принципе, решить любую механическую задачу; кроме того, из них могут быть выведены и все остальные законы механики. В соответствии с принципом относительности Гали- лея законы механики одинаковы во всех инерциальных системах отсчета. Это значит, в частности, что уравнение B.6) будет иметь один и тот же вид в любой инерциаль- ной системе отсчета. Действительно, масса m материаль- той точки как таковой не зависит от скорости, т. е. оди- накова во всех системах отсчета. Кроме того, для инер- циальных систем отсчета одинаковым является и ускоре- 42
ййе a точки. Сила ? тоже не зависит от выбора системы отсчета, поскольку она определяется только взаимным расположением и скоростью материальной точки относи- тельно окружающих тел, а эти величны, согласно нере- лятивистской кинематике, в разных инерциальных систе- мах отсчета одинаковы. Таким образом, все три величины, т, а и F, входящие в уравнение B.6), не меняются при переходе от одной инерциальной системы отсчета к другой, а следовательно, не меняется и само уравнение B.6). Другими словами, уравнение ma=F инвариантно относительно преобразо- ваний Галилея. § 2.3. Силы Чтобы свести нахождение закона движения частицы к чисто математической задаче, необходимо прежде все- го — в соответствии с уравнением B.6) — знать дейст- вующую на частицу силу, т. е. зависимость силы от оп- ределяющих ее величин. Каждая такая зависимость по- лучена в конечном счете на основании обработки резуль- татов опыта и, по существу, всегда опирается на урав- нение B.6), как на определение силы. Наиболее фундаментальные силы, лежащие в основе всех механических явлений, — это силы гравитационные и электрические. Приведем выражения для этих сил в самом простом виде, когда взаимодействующие массы (заряды) покоятся или движутся с малой (нерелятивист- ской) скоростью. Сила гравитационного притяжения, действующая между двумя материальными точками. В соответствии с законом всемирного тяготения эта сила про- порциональна произведению масс точек Ш\ и пг2, обрат- но пропорциональна квадрату расстояния г между ними и направлена по прямой, соединяющей эти точки: где у— гравитационная постоянная. Фигурирующие в этом законе массы называют грави- тационными в отличие от инертной массы, входящей во второй закон Ньютона. Из опыта, однако, установлено, что гравитационная и инертная массы любого тела стро- го пропорциональны друг другу. Поэтому можно считать их равными (т. е. выбрать один и тот же эталон для из- мерения обеих масс) и говорить просто о массе, кото- 43
рая Bbicfynaet как Мера инертности тела или как мера гравитационного действия. Кулоновская сила, действующая между двумя точеч- ными зарядами Ц\ и <7г, B.9) где г — расстояние между зарядами, k — коэффициент пропорциональности, зависящий от выбора системы еди- ниц. В отличие от гравитационной силы кулоновская си- ла может быть как силой притяжения, так и силой оттал- кивания. Заметим, что закон Кулона B.9) перестает выпол- няться точно, если заряды движутся. Электрическое вза- имодействие движущихся зарядов оказывается сложным образом зависящим от их движения. Одну из частей это- го взаимодействия, обусловленную движением, называ- ют магнитной силой (отсюда и другое название данного взаимодействия — электромагнитное). При малых (нере- лятивистских) скоростях магнитная сила составляет пре- небрежимо малую часть электрического взаимодействия и оно с высокой степенью точности описывается законом B.9). Несмотря на то, что гравитационные и электрические взаимодействия лежат в основе всего бесчисленного раз- нообразия механических явлений, анализ явлений, осо- бенно макроскопичрских, оказался бы весьма сложным, если бы во всех случаях мы исходили из этих фундамен- тальных взаимодействий. Поэтому удобно ввести дру- гие, приближенные, силы (которые в принципе могут быть получены из фундаментальных сил). Это необходи- мо для того, чтобы упростить математически задачу на- столько, чтобы ее можно было практически решить. С этой целью вводят, например, следующие силы. Однородная сила тяжести F=mg, B.10) где m — масса тела, g — ускорение свободного паде- ния *. * Заметим, что в отличие от силы тяжести вес Р — это си- ла, с которой тело действует на опору (или подвес), неподвижную относительно данного тела. Например, если тело с опорой (подве- сом ) неподвижны относительно Земли, то вес Р совпадает с силой тяжести. В противном случае вес P=m(g—а), где а — ускорение тела (с опорой) относительно Земли. 44
Упругая сила — сила, пропорциональная смещению материальной точки из положения равновесия и направ- ленная к положению равновесия: F=-*r, B.11) где г — радиус-вектор, характеризующий смещение час- тицы из положения равновесия; % — положительный ко- эффициент, зависящий от «упругих» свойств той или иной конкретной силы. Примером такой силы является сила упругой деформации при растяжении (сжатии) пру- жины или стержня; в соответствии с законом Гука эта сила определяется как F=kAI, где Л/ — величина уп- ругой деформации. Сила трения скольжения, возникающая при скольже- нии данного тела по поверхности другого тела, F=kRn, B.12) где k — коэффициент трения скольжения, зависящий от природы и состояния соприкасающихся поверхностей (в частности, от их шероховатости); Rn — сила нормального давления, прижимающая трущиеся поверхности друг к другу. Сила F направлена в сторону, противоположную направлению движения данного тела относительно дру- гого. Сила сопротивления, действующая на тело при его поступательном движении в газе или жидкости. Эта сила зависит от скорости v тела относительно среды, причем направлена противоположно вектору v: F=~kv, B.13) где k — положительный коэффициент, характерный для данного тела и данной среды. Этот коэффициент зависит, вообще говоря, от скорости v, однако при малых скорос- тях во многих случаях его можно практически считать постоянным. § 2.4. Основное уравнение динамики Основное уравнение динамики материальной точки представляет собой не что иное, как математическое вы- ражение второго закона Ньютона: tn^=F. B.14) At v 45
Уравнение B.14) есть, по существу, дифференциаль- ное уравнение движения точки в векторном виде. Его решение — основная задача динамики материальной точ- ки. При этом возможны две противоположные постанов- ки задачи. 1. Найти действующую на точку силу F, если извест- ны масса т точки и зависимость от времени ее радиуса- вектора r(t). 2. Найти закон движения точки, т. е. зависимость от времени ее радиуса-вектора г(^), если известны масса т точки, действующая на нее сила F (или силы F{) и на- чальные условия — скорость v0 и положение г0 точки в на- чальный момент времени. В первом случае задача сводится к дифференцирова- нию r(t) по времени, во вто- ром — к интегрированию уравнения B.14). Математи- ческая сторона этого вопроса достаточно подробно была рассмотрена в кинематике точки. В зависимости от характера и постановки конкрет- ной задачи решение уравнения B.14) проводят или в век- торной форме, или в координатах, или в проекциях на касательную и нормаль к траектории в данной точке. Выясним, как записывают уравнение B.14) в последних двух случаях. В проекциях на оси декартовых координат. Записы- вая обе части уравнения B.14) в проекциях на оси х, у, z, получим три дифференциальных уравнения вида Рис. 2.2 m~w-> m4T=i fTl , , —i B.15) где Fx, Fy, Fz — проекции вектора F на оси x, у, z. Необ- ходимо помнить, что эти проекции — величины алгебраи- ческие: в зависимости от ориентации вектора F они мо- гут быть как положительными, так и отрицательными. Знак проекции результирующей силы F определяет и знак проекции вектора ускорения. 46
Проследим на конкретном примере, в чем заключает- ся стандартный подход к решению задач с по- мощью уравнений B.15). Пример. Небольшой брусок массы т скользит вниз по наклонной плоскости, составляющей угол а с горизонтом. Коэффициент трения равен к. Найдем ускорение бруска относительно плоскости (эта сис- тема отсчета предполагается инерциальной). Прежде всего следует изобразить силы, действующие на брусок. Это сила тяжести mg, нормальная сила реакции R со стороны плос- кости и сила трения FTp (рис. 2.2), направленная в сторону, противо положную движению бруска. После этого свяжем с системой отсчета «наклонная плоскость» систему координат х, у, г. Вообще говоря, систему координат можно ориентировать как угодно, однако во многих случаях выбор направ- ления осей диктуется характером движения. В нашем случае, напри- мер, заранее известно направление движения бруска, поэтому наибо- лее целесообразно оси координат расположить так, чтобы одна из них совпадала с направлением движения. Тогда задача сведется к решению только одного уравнения B.15). Итак, выберем ось х, как показано на рис. 2.2, обязательно указав при этом ее положительное направление (стрелкой). И только теперь приступим к составлению уравнений B.15): слева — произведение массы от бруска на проекцию его ускорения ах и справа — проекции всех сил на ось х. Тогда та г = \ ах = ™ёх + Rx+ • трх В данном случае gx=g sin a, Rx=0 и Flpi = —.FTp, поэтому тах = mg sin a — Frp. Так как брусок движется только вдоль оси х, то это значит, соглас- но второму закону Ньютона, что сумма проекций всех сил на любое перпендикулярное оси х направление равна нулю. Взяв в качестве такого направления ось у (рис. 2.2), получим R = mg cos a , /7tp = kR - В результате тах = mg sin a — kmg cos а. Если правая часть этого уравнения окажется положительной, то ах>0, а это значит, что вектор а направлен вниз по наклонной плос- кости, и наоборот. В проекциях на касательную и нормаль к траектории в данной точке. Записывая обе части B.14) в проекциях на подвижные орты t и п (рис. 2.3) и используя полу- ченные ранее выражения A.10) для тангенциального и нормального ускорений, получим B.16) 47
где Ft и Fn — проекции вектора F на орты t и п. На рис. 2.3 обе проекции положительные. Векторы FT и Fn назы- вают тангенциальной и нормальной составляющими си- лы F. Напомним, что направление орта t выбирают в сто- рону возрастания дуговой координаты /, а направление орта п — к центру кривизны траектории в данной точке. Уравнениями B.16) удобно пользоваться, если зара- нее известна траектория материальной точки. Рис. 2.3 Пример. Небольшое тело А соскальзывает с вершины гладкой сферы радиуса г. Найдем скорость тела в момент отрыва от поверх* ности сферы, если его начальная скорость пренебрежимо мала. Изобразим силы, действующие на тело А (это сила тяжести mg и нормальная сила реакции R), и запишем уравнения B.16) в про- екциях на орты тип (рис. 2.4): 7=' 4 ml = / Здесь индекс т несуществен, поэтому мы его опустили. Преобразуем первое уравнение к виду, удобному для интегриро- вания. Воспользовавшись тем, что dt=dl/v = rdQ[v, где d/— элемен- тарный путь тела А за промежуток времени dt, перепишем первое уравнение в виде v dv =gr sinft dft. Проинтегрировав левую часть этого выражения от 0 до v, правую — от 0 до О, найдем i/2 = 2gr(I — cos»). В момент отрыва ^? = 0, поэтому второе исходное уравнение прини* мает вид где v и О соответствуют точке отрыва. Исключив cos О из последних двух равенств, получим v = |/2/з|Т.
§ 2.5. Неинерциальные системы отсчета. Силы инерции Основное уравнение динамики в неинерциальной си- стеме. Ранее было отмечено, что основное уравнение ди- намики справедливо только в инерциальных системах отсчета. Между тем имеется много случаев, когда реше- ние интересующей нас задачи необходимо получить в неинерциальных системах (например, движение матема- тического маятника в ускоренно движущемся вагоне, дви- жение спутника относительно поверхности Земли и др.). Поэтому возникает вопрос: как следует изменить основ- ное уравнение динамики, чтобы оно оказалось справед- ливым и для неинерциальных систем отсчета? С этой целью возьмем две системы отсчета: инерци- альную Д"-систему и неинерциальную /С'-систему. Пусть известны масса т частицы, сила F, действующая на нее со стороны окружающих тел, и характер движе- ния /("'-системы относительно /С-системы. Рассмотрим достаточно общий случай, когда /С'-си- стема вращается с постоянной угловой скоростью о во- круг оси, перемещающейся поступательно с ускорением а0 относительно /С-системы. Воспользуемся формулой преобразования ускорений A.31). Из нее следует, что ус- корение частицы в К'-сястеме Н B.17) где v' — скорость частицы относительно К'-системы, р — радиус-вектор, перпендикулярный оси вращения и харак- теризующий положение частицы относительно этой оси. Умножив обе части B.17) на массу пг частицы и уч- тя, что в инерциальной системе отсчета ?rca = F, получим ma'=F—i B.18) Это и есть основное уравнение динамики в неинерциальной системе отсчета, которая вращается с постоянной угловой скоростью ю вокруг оси, перемещающейся поступательно с ускорением а0. Из не- го видно, что даже при F=0 частица будет двигаться в этой системе с ускорением, в общем случае отличным от нуля, причем так, как если бы на нее действовали неко- торые силы, соответствующие последним трем членам уравнения B.18). Эти силы назвали силами инер- ции, 49
Уравнение B.18) показывает, что введение сил инер- ции позволяет сохранить по форме основное уравнение динамики и для неинерциальных систем: слева — произ- ведение массы частицы на ее ускорение (но уже по от- ношению к неинерциальной системе отсчета), справа — силы. Однако кроме силы F, обусловленной действием окружающих тел (силы взаимодействия), необходимо учесть и силы инерции — остальные слагаемые в правой части уравнения B.18). Силы инерции. Перепишем уравнение B.18) в таком виде: ота'=Р+Рин+Рцб+РкоР> B.19) где B.20) ^ин — поступательная сила инерции, обусловлен- ная поступательным движением неинерциальной систе- мы отсчета; — центробежная сила инерции; B.21) B.22) — сила Кориолиса, или кориолисова сила инерции. Последние две силы обусловлены вращатель- ным движением системы отсчета. Мы видим, таким образом, что силы инерции зависят от свойств неинерциальной системы отсчета (а0, ш0), а также от расстояния р и скорости \' частицы в этой си- стеме отсчета. Если, например, неинерциальная система отсчета дви- жется поступательно (по отношению к инерциальной си- стеме отсчета), то в этой системе на свободную частицу действует только сила B.20), направление которой про- тивоположно ускорению а0 данной системы отсчета. Вспомним, как при резком торможении вагона сила ине- рции бросает нас вперед, т. е. в сторону, противополож- ную вектору а0. Другой случай: система отсчета вращается с угловой скоростью са вокруг неподвижной оси, и тело А покоится в этой системе (например, вы сидите на горизонтальном вращающемся круге аттракциона «колесо смеха»). На 50
тело А кроме силы взаимодействия с окружающими те- лами действует центробежная сила инерции B.21), на- правленная от оси вращения вдоль радиуса-вектора р. Пока тело А покоится относительно круга (v'=0), эта сила компенсирует силу взаимодействия. Но как толь- ко тело придет в движение, т. е. появится скорость V, начнет действовать и сила Кориолиса B.22), направле- ние которой определяет векторное произведение [v'o]. Заметим, что сила Ко- риолиса появляется в дополнение к центро- бежной силе инерции, действующей независи- мо от того, покоится те- а) т \ т Рис. 2.5 Рис. 2.6 ло или движется во вращающесйя системе отсчета. Ранее было отмечено, что система отсчета, связанная с земной поверхностью, во многих случаях может счи- таться практически инерциальной. Однако существует ряд явлений, истолкование которых в этой системе отсче- та невозможно без учета ее неинерциальности. Известно, например, что ускорение свободного паде- ния тел относительно поверхности Земли имеет наиболь- шее значение у полюсов. Уменьшение этого ускорения по мере приближения к экватору объясняется не только не- сферичностью Земли, но и возрастающим действием цен- тробежной силы инерции. Или такие явления, как откло- нение свободно падающих тел к востоку, размыв правых берегов рек в северном полушарии и левых берегов — в южном, вращение плоскости качания маятника Фуко и др. Подобные явления связаны с движением тел относи- тельно поверхности Земли и могут быть объяснены дей- ствием сил Кориолиса. 51
Пример. Поезд массы т движется по меридиану на широте ф со скоростью v'. Найдем силу R' бокового давления, с которой поезд действует на рельсы. В системе отсчета, связанной с Землей (она вращается с угловой скоростью ш ), составляющая ускорения поезда, перпендикулярная плоскости меридиана, равна нулю. Поэтому и сумма проекций сил, действующих на поезд в этом направлении, также равна нулю. А это значит, что сила Кориолиса FKOp (рис. 2.5) должна уравновешивать- ся силой R бокового давления, действующей на поезд со стороны правого по ходу движения рельса, т. е. FKoP = —R. По третьему за- кону Ньютона, поезд будет действовать на этот рельс в горизонталь- ном направлении с силой R'=—R. Следовательно, R' = FKop = = 2m[v'o> ). Модуль вектора R' равен fl'=2mt>'a> sincp. Следующий простой пример показывает, как «возни- кают» силы инерции при переходе от инерциальной си- стемы отсчета к неинерциальной. Пример. На поверхности стола находится горизонтальный диск D, свободно вращающийся вокруг вертикальной оси с постоянной угловой скоростью о) Над диском висит шарик массы т, как пока- зано на рис. 2.6, а. Рассмотрим поведение этого шарика в /С-системе отсчета, связанной со столом (она предполагается инерциальной), и в ^('-системе, связанной с вращающимся диском. В инерциальной /(-системе на шарик действуют две силы: сила тяжести и сила натяжения со стороны нити. Эти силы компенсируют друг друга, и шарик покоится. В неинерциальной iC'-снстеме шарик движется равномерно по ок- ружности с нормальным ускорением <о2р, где р — расстояние от ша- рика до оси вращения. Легко убедиться, что это ускорение обуслов- лено действием сил инерции. В самом деле, в ^'-системе помимо указанных выше двух сил, компенсирующих друг друга, действуют еще центробежная сила инерции и сила Кориолиса (рис. 2.6, б). Взяв проекции этих снл на нормаль п к траектории в точке нахож- дения шарика, запишем та' = Укор — Fn6 = 2mi>' ш — mu>2 р = тш2 р, где учтено, что в данном случае и' = сор. Отсюда видно, что а'п — = <о2р. Особенности сил инерции. Подводя итог, перечислим важнейшие особенности сил инерции, отличающие их от сил взаимодействия: 1. Силы инерции обусловлены не взаимодействием тел, а свойствами самих неинерциальных систем отсчета. По- этому на силы инерции третий закон Ньютона не рас- пространяется. 2. Эти силы существуют только в неинерциальных си- стемах отсчета — это необходимо твердо помнить во из- бежание недоразумений. В инерциальных системах от- счета сил инерции вообще нет, и понятие сила в этих си- стемах отсчета применяется только в ньютоновском смысле, как мера взаимодействия тел. 52
m, №•> 3. Все силы инерции, подобно силам тяготения, про- порциональны массе тела. Поэтому в однородном поле сил инерции, как и в поле сил тяготения, все тела дви- жутся с одним и тем же ускорением независимо от их масс. Это весьма существенный факт с далеко идущими последствиями. Принцип эквивалентности. Тот факт, что силы инерции, как и силы тяготения, пропорциональны массам тел, приводит к следущему важному заключению. Представим себе, что мы находимся в некото- рой закрытой лаборатории и не имеем возможности наблюдать внеш- ний мир. Допустим, кроме того, что мы не знаем, где находится ла- боратория: в космическом прост- ранстве или, скажем, на Земле. За- мечая, что все тела независимо от их массы падают в лаборатории с одинаковым ускорением, мы не можем на основании только этого факта установить, чем вызвано это ускорение — полем тяготения, ус- • „ ^~ - *• коренным поступательным движе- рис 2 7 нием самой лаборатории или, на- конец, обеими этими причинами вместе. Никакие опыты по свободному падению тел в такой лабора- тории не могут отличить однородное поле тяготения от однородного поля сил инерции. Эйнштейн высказал предположение, что вообще никакими физи- ческими опытами невозможно отличить однородное поле тяготения от однородного поля сил инерции. Это предположение, возведенное в постулат, и составляет содержание так называемого принципа эквивалентности сил тяготения и сил инерции: все физичес- кие явления в однородном поле тяготения происходят совершенно так же, как и в соответствующем однородном поле сил инерции. Глубокая аналогия между силами инерции и силами тяготения послужила отправным пунктом при построении Эйнштейном общей теории относительности, или релятивистской теории гра- витации. В заключение необходимо отметить, что любую ме- ханическую задачу можно решить в инерциальной и не- инерциальной системах отсчета. Выбор той или иной системы отсчета обычно диктуется или постановкой во- проса, или стремлением получить решение возможно бо- лее простым путем. При этом часто наиболее удобно по- льзоваться именно неинерциальными системами отсчета (см. задачи 2.9—2.11). Задачи #2.1. Основное уравнение динамики. Брусок массы <mt находит- ся на доске массы т2, которая лежит на гладкой горизонтальной плоскости (рис. 2.7). Коэффициент трения между бруском и доской равен k. К доске приложили горизонтальную силу F, зависящую от 53
времени t по закону F=at, где а — постоянная. Найти: 1) MOMeHf времени to, когда доска начнет выскальзывать из-под бруска; 2) ус- корения бруска aL и доски а2 в процессе движения. Решение. 1. Запишем основное уравнение динамики для брус- ка и доски, взяв положительное направление оси х, как показано на рисунке: -F-Ftt. (I) По мере возрастания силы F будет расти и сила трения FTp (внача- ле она является силой трения покоя). Но FTV имеет предел Fif.uiiKu = kniig. Пока этот предел не достигнут, оба тела будут дви- гаться как единое целое с одинаковыми ускорениями. Когда же сила Рис. 2.8 Рис. 2.9 /Чр достигнет предела, доска начнет выскальзывать из-под бруска, т. е. а2 > аг. Подставив сюда выражения для at и аг из A) с учетом того, что F-iV = kmig, получим где знак равенства соответствует моменту t=to- Отсюда 2. Если t<ito, то если же t>>to, то Графики зависимостей а.\ и вг от t показаны на рис. 2.8. % 2.2. В установке (рис. 2.9) наклонная плоскость составляет угол а = 30° с горизонтом. Отношение масс тел r]=mi/.m2=2/3. Ко- эффициент трения между телом гпг и плоскостью k = 0,10. Массы блока и нити пренебрежимо малы. Найти модуль и направление ускорения тела mi, если система пришла в движение из состояния покоя. Решение. Здесь сразу же возникает вопрос, связанный с на- правлением силы трения, действующей на тело mi- Без ответа на этот вопрос нельзя записать основное уравнение двинамики для тела 54
пц в проекциях и задача оказывается неопределенной Будем рассуждать так в отсутствие силы трения тело т2 начало бы скользить по наклонной плоскости, допустим, вверх. Ясно, что «включение» силы трения не может изменить направления движения, а только уменьшит ускорение Таким образом, направление силы трения, действующей на тело т2, будет определено, если найти на- правление ускорения этого тела в отсутствие трения (fe = 0). С этого мы и начнем Запишем основное уравнение динамики для обоих тел в проекциях, взяв положительные направления осей х\ и х2, как показано на рис 29 /raj ах = m\g — Т, m.<iux = T — m2S где Т — сила натяжения нити Сложив почлен- но левые и правые части этих уравнений, полу- чим •г) —sin a Рис. 2Л0 Подстановка в это выражение г] = 2/з и а=30° дает ах>0, т е тело т2 начнет двигаться вверх по наклонной плоскости Следовательно, сила трения, действующая на это тело, будет на- правлена в противоположную сторону С учетом этого обстоятельст- ва снова запишем уравнения движения mi a.' =m1? — 7", ш2 а == Г — > ia — i cos a. Отсюда , -л — sin a — k cos a a _J gz= T) + 1 #2.3. Через блок (рис 2 10) перекинута нерастяжимая нить, на концах которой висят грузы 1 и 2 с массами nil и т2 соответствен- но. Блок начали поднимать вверх с ускорением ао относительно Земли Полагая, что нить скользит по блоку без трения, найти уско- рение п\ груза / относительно Земли. Решение Выберем положительное направление оси х вверх и запишем для обоих грузов основное уравнение динамики в проек- ции на эту ось: mialx = T — mxg, (I) B) mi пчх = 7* — Эти уравнения содержат три неизвестных ления третьего уравнения воспользуемся между ускорениями Зп, а2х и Т. Для состав- кинематической связью 55
где а' — ускорение груза / относительно блока. Сложив почленно левые и правые части этих равенств, получим ai+a2=2ao, или в проекциях на ось х Решив совместно уравнения A), B) и C), найдем 2т2 До + (т2 — mi) g аи = Отсюда видно, что при заданном а0 знак aix зависит от соотношения масс nil и т2. Рис. 2.11 Рис. 2.12 ф 2.4. Небольшая шайба движется по наклонной плоскости, ко- эффициент трения которой fe = tga, где a — угол наклона плоскости к горизонту. Найти зависимость скорости v шайбы от угла <р между вектором v и осью х (рис. 2.11), если в начальный момент v = v0 и Ф = я/2. Решение. Ускорение шайбы вдоль плоскости определяется со- ставляющей силы тяжести на эту плоскость Fx = mg sin a и силой трения F?p = kmg cos a. В нашем случае &=tga, поэтому Ftp = Fx — mg sin a. Найдем проекции ускорения на направление касательной к траекто- рии и на ось х: та =Fxcos <p — Fxp = mg sin« (cos a—I), max = Fx — ftp cos <p = mg sin a A — cos <p). Отсюда видно, что ar=—ах, а это значит, что скорость v и ее про- екция vx различаются лишь на некоторую постоянную С, не завися- щую от времени, т. е. и = — vx + C, где и* = и cos <p. Постоянную С находим из начального условия v — vo, откуда C=vo. В результате получим I*' С ростом времени ф-»-0 и х>- ф2.5. Тонкий однородный упругий шнур массы т и длины /0( в нерастянутом состоянии) имеет коэффициент упругости %. Склеив 56
торцы, шнур положили на гладкую горизонтальную плоскость, при- дали ему форму окружности и раскрутили до угловой скорости и вокруг вертикальной оси, проходящей через его центр. Найти силу натяжения шнура в этом состоянии. Решение. Мысленно выделим малый элемент шнура массы 8т, как показано на рис. 2.12, а. Этот элемент движется по окруж- ности под действием силы, представляющей собой геометрическую сумму двух векторов, каждый из которых равен по модулю искомой силе натяжения Т (рис. 2.12, б). Поэтому согласно основному урав- нению динамики, Ът-иРг = Т-Ъа. (I) Учтем, что бт=(т/2я)ба и г=//2я, / — длина шнура во враща- ющемся состоянии. Тогда A) примет вид дасо2//4л2 = Г. B) С другой стороны, по закону Гука, Г=х(/-/о). C) Исключив I из B) и C), получим Т 4я2х/ото>2— 1 Заметим, что в случае нерастяжимого шнура (%=оо) Г=ти2/о/4я2. #2.6. Интегрирование уравнений движения. Частица массы m движется под действием силы F. В момент ^=0 известны ее радиус- вектор г@) и скорость v@) —начальные условия. Найти положение частицы, т. е. ее радиус-вектор г, в зависимости от времени t, если: r@) = 0, v@) = 0; 2)F=— kv, r@) = 0, v@) = v0; 3) F= — %r, r@) = r0, v@) = v0, причем v0||r0. Здесь Fo — постоянный вектор; со, k, %— положительные постоянные. Решение. 1. Согласно основному уравнению динамики, в дан- ном случае ускорение Отсюда находим элементарное приращение вектора скорости dv за время сМ и затем приращение этого вектора за время от 0 до t: t v @ — v @) = (F0/m) f sin id &t. Учитывая, что v@)=0, после интегрирования получим v (t) = A — cos lot) F0/wtw. Теперь найдем dr — элементарное перемещение, или приращение ра- диуса-вектора г частицы за время At: dr=v(/)d/. Приращение же радиуса-вектора за время от 0 до t 57
г (t) — г @) = (F0/mco) \ (I — cos id) it., В результате интегрирования находим г (t) = (со/ — sin со*) F0/ffz<o2. где учтено, что г@)=0. На рис. 2.13 показаны графики зависимостей vx(t) и x(t) —про- екций векторов v и г на ось х, выбранную в направлении движения частицы, т. е. в направлении вектора Fo. 2л Рис. 2.13 2. В этом случае ускорение Рис. 2.14 = -(k/m)v. Для интегрирования этого уравнения перейдем к скалярной форме - к модулю вектора v: Интегрирование этого уравнения (с учетом начальных условий) дает \n(v/vo)~ — (k\m)t. После потенцирования возвращаемся к вектор- ной форме: v =i Последнее уравнение интегрируем еще раз (и также с учетом на- чальных условий); t г = Г v dt = A — e~kl/m) m\Q/k. На рис. 2.14 показаны графики скорости v и пройденного пути s в зависимости от времени t (в нашем случае s — r). 3. В данном случае движение будет происходить вдоль прямой, совпадающей с радиусом-вектором г. Возьмем в этом направлении ось х и сразу запишем основное уравнение динамики в проекции на эту ось, причем в такой форме: Зс + со2дг=0, A) 58
где х — вторая производная координаты по времени (проекция век- тора ускорения), со2 = vjm. Уравнение A) называют уравнением гармонических колебаний. В математике доказывается, что общее решение этого уравнения имеет вид x{f) = Azosu>t + B$\aid, B) где А и В— произвольные постоянные. Ограничения, накладываемые на эти постоянные, определяются обычно из начальных условий. Например, в нашем случае в момент ^=0 х@)=х0 и vx@)=v0x, C) где х0 и vox—проекции векторов г о и v0 на ось х. После подстанов- ки B) в C) получим Л = х0, fi = uox/w. Дальнейшее очевидно. ф 2.7. Частица массы т движется ? некоторой плоскости под действием пь стоянной по модулю силы F, направле- ние которой поворачивается в этой плос- кости с постоянной угловой скоростью со. В момент ^ = 0 скорость частицы равна нулю. Найти модуль скорости частицы как функцию времени t, а также путь, о' проходимый частицей между двумя по- следовательными остановками. рис 2.15 Решение. Свяжем с данной плос- костью систему координат х, у, взяв ось х в направлении, которое имел вектор силы в момент /=0 (рис. 2.15). Тогда основное уравне- ние динамики в проекциях на оси хну будет иметь вид т &vxJAt = F cos u>t, т dvyjdt =F swat. Проинтегрировав эти уравнения по времени с учетом начального ус- ловия и@)=0, получим vx = (.Р/тиш) sin at, vy = (Finn*) A — cos at). Модуль вектора скорости частицы v = У vx + v2y = 21 sin (<ut/2)\F/m<». Отсюда видно, что скорость v обращается в нуль через промежуток времени Д^, который можно найти из условия соД//2=я. Поэтому искомый путь д/ s = Г v dt = X Заметим, что траектория частицы имеет вид циклоиды. ф 2.8. Автомашина движется с постоянным тангенциальным ус- корением а по горизонтальной поверхности, описывая окружность радиуса R. Коэффициент трения между колесами машины и поверх- ностью равен к. Какой путь s пройдет машина без скольжения, если начальная скорость ее была равна нулю? 59
Решение. По мере увеличения скорости будет расти как нор- мальное, так и полное ускорение машины. Движение будет происхо- дить без скольжения, пока необходимое полное ускорение будет обеспечиваться силой трения. Максимально возможное значение этой силы FunKc — kmg, где т — масса машчны. Поэтому максимальное значение полного ускорения (согласно основному уравнению дннамн- ки ma = F) Дмакс = *?- (О С другой стороны, B) где v — скорость машины в момент, когда ее ускорение станет мак- симально возможным. Эта скорость и искомый путь s связаны формулой »2 = 2a/s. C) Выразив о из A) и B) и подставив в C), полу- чим I Рис. 2.16 Нетрудно видеть, что ре- шение имеет смысл прн ф 2.9. Неинерциальные системы отсчета. Спутник движется в эк- ваториальной плоскости Земли с запада на восток по круговой орбите радиуса г. Пренебрегая ускорением, обусловленным движе- нием Земли вокруг Солнца, найти ускорение а' спутника в системе отсчета, связанной с Землей. Решение. Пусть К — инерциальная (по условию) система от- счета, в которой ось вращения Земли покоится, а К' — неинерциаль- ная система отсчета, которая связана с Землей и вращается с угло- вой скоростью со относительно /С-системы. Нас интересует ускорение а' спутника в /('-системе. Для этого прежде всего изобразим все силы, действующие на спутник, в этой системе отсчета: силу тяготения F, силу Кориолиса FKOp и центро- бежную силу инерции Fue (рис. 2.16, вид со стороны Северного по- люса). Теперь воспользуемся уравнением B.18), считая ао=О (по усло- вию). Спутник движется по окружности в /С'-системе, поэтому запи- шем уравнение B.18) в проекциях на нормаль п к траектории: та' =Р — 2mv' ш — ти& г, A) где F=ymM[rz, т и М — массы спутника и Земли. Остается найти скорость v' спутника в /С'-системе. Для этого воспользуемся форму- лой A.24) в скалярном виде v'=v — <*>r, B) 60
где v — скорость спутника в /С-системе. Эту скорость можно опреде- лить с помощью уравнения движения спутника в /С-системе: mv2/r = ут М/г*. C) Решив совместно уравнения A), B) и C), получим уМ а' =¦ /¦2 В частности, а'=0 прил= у^уМ/ш2 — 4,2-103 км. Такой спут- ник называют стационарным: он неподвижен относительно по- верхности Земли. ко/ Рис. 2.17 ф2.10. Небольшая муфта массы т свободно скользит по глад- кому горизонтальному стержню, который вращают с постоянной угловой скоростью w вокруг неподвижной вертикальной оси, прохо- дящей через один из его концов. Найти горизонтальную составляю- щую силы, действующей на муфту со стороны стержня в момент, ког- да она находится на расстоянии г от оси вращения. (В начальный момент муфта находилась непосредственно около оси и имела пре- небрежимо малую скорость.) Решение. Рассмотрим движение муфты во вращающейся сис- теме отсчета, жестко связанной со стержнем. В этой системе отсчета муфта движется прямолинейно, а это значит, что искомая сила R уравновешивается силой Кориолиса (рис. 2.17, вид сверху): R=-Frop = 2m[Mv']. A) Задача, таким образом, сводится к нахождению скорости v' муфты относительно стержня. Согласно B.19), Преобразуем это уравнение, имея в виду, что &t=dr/v'. Тогда полу- чим v'dv — <x>2rdr. Проинтегрировав последнее уравнение с учетом начальных условий (и' = 0, г = 0), найдем w'=co/", или в векторном виде 61
v'=wr. B) Подстановка B) в A) дает R = 1тч> [мг]. 02 11. Устойчивость движения. Небольшая муфта М может скользить без трения по гладкому проводу, изогнутому в форме по- луокружности радиуса г (рис. 2.18). Систему привели во вращение с постоянной угловой скоростью со вокруг вертикальной оси 00'. Найти угол до, соответствующий устойчивому положению муфты. Решение. Рассмотрим поведение муфты в системе отсчета, связанной с вращающимся проводом. Движение муфты вдоль прово- да будет определяться проекцией Fт результирующей силы на на- правление орта z в точке М. Из рис. 2.18 видно, что F =/:<uecosft—mg sin 9, где справа записаны проекции центробежной силы инерции и силы тяжести. Учтя, что /?Цб=яш2г sin О, перепишем предыдущее выра- жение так: F oj sin ft (cos» — g/ ы2/-). A) Из условия равновесия (Fx =0) найдем два значения угла Фо, при которых оно возможно: sin O0 = 0 и cos '&o = gr/co2r Первое усло- вие может быть осуществлено при любых значениях со, второе же — только при g/coV<l. Таким образом, при малых со существует толь- ко одно положение равновесия — в нижней точке (/&о = О); при боль- ших же со((в>У§//') возможно и другое положение равновесия, опре- деляемое вторым условием. Для устойчивости определенного состояния равновесия необхо- димо, чтобы сила ,FT при выведении муфты из положения равновесия (в любую сторону) была направлена обратно — к положению равно- весия. Другими словами, знак Fх должен быть противоположен зна- ку отклонения ДФ от равновесного угла Фо- При малом отклонении dO от угла fto возникающая сила 6FT может быть найдена как дифференциал выражения A): s&o — gl^r)— sin2fto]dft. В нижнем положении равновесия (/&о = О) В/=т oo(I— g/ofir)db. B) Это положение равновесия будет устойчивым, когда выражение, сто- ящее в скобках, отрицательно, т. е. при <a<ygjr. В другом положении равновесия (cos /6'o=g/coV) Видно, что это положение равновесия (если оно существует) всегда устойчиво. Итак, пока существует лишь нижнее положение равновесия (при w<Tg/'')> оно всегда устойчиво. При появлении же другого положе- ния равновесия (когда со>У§/г) нижнее положение, согласно B), 62
становится неустойчивым и муфта сразу переходит из нижнего по- ложения в верхнее, которое всегда устойчиво. Глава 3 ЗАКОН СОХРАНЕНИЯ ИМПУЛЬСА § 3.1. О законах сохранения Любое тело (или совокупность тел) представляет со- бой, по существу, систему материальных точек, или час- тиц. Если система с течением времени изменяется, то го- ворят, что изменяется ее состояние. Состояние сис- темы характеризуется одновременным заданием положе- ний (координат) и скоростей всех ее частиц. Зная законы действующих на частицы системы сил и состояние системы в некоторый начальный момент вре- мени, можно, как показывает опыт, с помощью уравне- ний движения предсказать ее дальнейшее поведение, т. е. найти состояние системы в любой момент времени. Так, например, решается задача о движении планет Солнеч- ной системы. Однако детальное рассмотрение поведения системы с помощью уравнений движения часто бывает настолько затруднительно (например, из-за сложности самой систе- мы), что довести решение до конца представляется прак- тически невозможным. А в тех случаях, когда законы дей- ствующих сил вообще неизвестны, такой подход оказы- вается в принципе неосуществимым. Кроме того, сущест- вует ряд задач, в которых детальное рассмотрение дви- жения отдельных частиц просто и не имеет смысла (на- пример, описание движения отдельных молекул газа). При таком положении естественно возникает вопрос: нет ли каких-либо общих принципов, являющихся следст- вием законов Ньютона, которые позволили бы иначе по- дойти к решению задачи и помогли бы в какой-то степе- ни обойти подобные трудности? Оказывается, такие принципы есть. Это так называе- мые законы сохранения. Как уже было сказано, при движении системы ее со- стояние изменяется со временем. Существуют, однако, такие величины, которые обладают весьма важным и за- мечательным свойством сохраняться во времени. Среди этих сохраняющихся величин наиболее важную роль иг- рают энергия, импульс и момент импульса. 63
Эти три величины имеют важное общее свойство адди- тивности: их значение для системы, состоящей из частей, взаимодействие которых пренебрежимо мало, равно сум- ме значений для каждой из частей в отдельности (впро- чем, для импульса и момента импульса свойство аддитив- ности выполняется и при наличии взаимодействия). Именно свойство аддитивности и придает этим трем вели- чинам особую роль. Законы сохранения энергии, импульса и момента импульса име- ют, как выяснилось впоследствии, весьма глубокое происхождение, связанное с фундаментальными свойствами времени и пространства— однородностью и изотропностью. А именно: закон сохранения энер- гии связан с однородностью времени, а законы сохранения импульса и момента импульса — соответственно с однородностью и изотропно- стью пространства. Сказанное следует понимать в том смысле, что перечисленные законы сохранения можно получить из второго закона Ньютона, если к нему присоединить соответствующие свойства сим- метрии времени и пространства. Более подробно обсуждать этот вопрос мы, однако, не будем. Законы сохранения энергии, импульса и момента им- пульса относятся к числу тех фундаментальных принци- пов физики, значение которых трудно переоценить. Роль этих законов особенно возросла после того, как выясни- лось, что они далеко выходят за рамки механики и пред- ставляют собой универсальные законы природы. Во вся- ком случае, до сих пор не обнаружено ни одного явления, где бы эти законы нарушались. Они безошибочно «дейст- вуют» и в области элементарных частиц, и в области кос- мических объектов, в физике атома и физике твердого тела и являются одними из тех немногих наиболее общих законов, которые лежат в основе современной физики. Открыв возможность иного подхода к рассмотрению различных механических явлений, законы сохранения стали весьма мощным и эффективным инструментом ис- следования, которым повседневно пользуются физики. Эта важнейшая роль законов сохранения как инструмен- та исследования обусловлена рядом причин: 1. Законы сохранения не зависят ни от траекторий частиц, ни от характера действующих сил. Поэтому они позволяют получить ряд весьма общих и существенных заключений о свойствах различных механических процес- сов, не вникая в их детальное рассмотрение с помощью уравнений движения. Если, например, выясняется, что та- кой-то процесс противоречит законам сохранения, то сра- зу можно утверждать: этот процесс невозможен, и бес- смысленно пытаться его осуществить. 64
2. Тот факт, что законы сохранения не зависят от ха- рактера действующих сил, позволяет использовать их да- же тогда, когда силы вообще неизвестны. В этих случаях законы сохранения являются единственным и незамени- мым инструментом исследования. Так, например, обстоит дело в физике элементарных частиц. 3. Даже в тех случаях, когда силы в точности извест- ны, законы сохранения могут оказать существенную по- мощь при решении многих задач о движении частиц. Хо- тя все эти задачи могут быть решены с помощью уравне- ний движения (в этом отношении из законов сохранения мы не получим никакой дополнительной информации), привлечение законов сохранения очень часто позволяет получить решение наиболее простым и изящным путем, избавляя нас от громоздких и утомительных расчетов. Поэтому при решении новых задач обычно принято при- держиваться следующего порядка: прежде всего один за другим применяют соответствующие законы сохранения и, только убедившись, что этого недостаточно, переходят затем к решению с помощью уравнения движения. Изучение законов сохранения начнем с закона сохра- нения импульса. § 3.2. Импульс системы Импульс частицы. По определению, импульс час- тицы * где m и v — ее масса и скорость. Воспользовавшись поня- тием импульса, запишем основное уравнение динамики B.6) в иной форме: C.1) т. е. производная импульса материальной точки по вре- мени равна действующей на нее силе. В частности, если F=0, то р = const. Заметим, что в неинерциальной системе отсчета си- ла F в C.1) включает в себя не только силы взаимодей- ствия данной частицы с другими телами, но и силы инер- ции. * Другое название этой величины — количество движе- ния. 65
Уравнение C.1) позволяет найти приращение импуль- са частицы за любой промежуток времени, если известна зависимость силы F от времени. Действительно, из C.1) следует, что элементарное приращение импульса части- цы за промежуток времени dt есть dp = FcU. Проинтегри- ровав это выражение по времени, найдем приращение им- пульса частицы за конечный промежуток времени t: C.2) ат2/б Рис. 3.1 Величину, стоящую в правой части этого уравнения, на- зывают импульсом си- л ы. Таким образом, прира- щение импульса частицы за любой промежуток времени зависит не только от значе- ния силы, но и от продолжи- тельности ее действия, или, другими словами, равно им- пульсу силы за это время. В частности, если F= = const, то вектор F можно вынести из-под интеграла и тогда рг—pi = F?. Рассмотрим пример на использование уравнения C.2). Пример. На частицу, которая в момент ^=0 имела импульс ро, действует в течение промежутка времени т сила, зависящая от време- ни t как F = a^(l—tjx), где а — постоянный вектор. Найдем им- пульс р частицы после окончания действия этой силы. т Согласно C.2) р = р0 + \ F At = р0 + а и2/6 (рис. 3.1). о Импульс системы. Рассмотрим произвольную систему частиц. В общем случае частицы этой системы могут взаимодействовать как между собой, так и с телами, не входящими в данную систему. В соответствии с этим си- лы взаимодействия между частицами системы называют внутренними, а силы, обусловленные действием других тел, не входящих в данную систему,— внешни- м и. Ясно, что такое разделение сил на внутренние и внешние условно — оно целиком зависит от выбора инте- ресующей нас системы частиц. Заметим также, что в не- инерциальных системах отсчета к внешним силам отно- сятся и силы инерции. 66
Теперь введем понятие импульса системы как векторную сумму импульсов ее отдельных частиц: C.3) где р, — импульс 1-й частицы. Заметим, что импульс сис- темы— величина аддитивная, т. е. импульс системы ра- вен сумме импульсов ее отдельных частей независимо от того, взаимодействуют они между собой или нет. Найдем физическую величину, которая определяет из- менение импульса системы. Для этого продифференциру- ем C.3) по времени: Согласно C.1), 2 k где F,ft — силы, действующие на t-ю частицу со стороны других частиц системы (внутренние силы); F» — сила действующая на эту же частицу со стороны других тел, не входящих в рассматриваемую систему (внешние силы). Подставив последнее выражение в предыдущее, получим 2 i k с Двойная сумма справа — это сумма всех внутренних сил. В соответствии с третьим законом Ньютона силы взаимо- действия между частицами системы попарно одинаковы по модулю и противоположны по направлению. Поэтому результирующая сила в каждой паре взаимодействия равна нулю, а значит, равна нулю и векторная сумма всех внутренних сил. В результате последнее уравнение принимает следующий вид: dp/d/=FBHeiu, C.4) где FBHeni — результирующая всех внешних сил, FEnein = 2 2 Уравнение C.4) означает: производная импульса сис- темы по времени равна векторной сумме всех внешних сил, действующих на частицы системы. Как и в случае одной частицы, из уравнения C.4) следует, что приращение импульса системы за конечный промежуток времени t есть 67
C.5) т. е. приращение импульса системы равно импульсу ре- зультирующей всех внешних сил за соответствующий про- межуток времени. И здесь, конечно, FEHem — результирую- щая всех внешних сил. Уравнения C.4) и C.5) справедливы как в инерциаль- ной, так и в неинерциальной системах отсчета. Следует только иметь в виду, что в неинерциальной системе отсче- та необходимо учитывать и действие сил инерции, играю- щих роль внешних сил, т. е. под Fmjeni в этих уравнениях надо понимать сумму FB3 + FHH, где FB3 — результирующая всех внешних сил взаимодействия, a FBH — результирую- щая всех сил инерции. § 3.3. Закон сохранения импульса Прежде всего введем понятие замкнутой (или изо- лированной) системы. Так называют систему частиц, на которую не действуют никакие посторонние тела (или их воздействие пренебрежимо мало). Другими словами, система замкнута, если внешние силы отсутствуют. Оче- видно, что понятие замкнутой системы имеет смысл толь- ко по отношению к инерциальным системам отсчета, по- скольку в неинерциальных системах отсчета всегда дей- ствуют силы инерции, играющие роль внешних сил. По- нятие замкнутой системы является естественным обобще- нием понятия изолированной материальной точки и игра- ет весьма важную роль в физике. Согласно уравнению C.4), импульс системы может изменяться под действием только внешних сил. Внутрен- ние силы не могут изменить импульс системы. Отсюда не- посредственно вытекает закон сохранения им- пульса: импульс замкнутой системы частиц остается постоянным, т. е. не меняется со временем: C.6) При этом импульсы отдельных частиц или частей замкну- той системы могут меняться со временем, что и подчерк- нуто в последнем выражении. Однако эти изменения всег- да происходят так, что приращение импульса одной час- ти системы равно убыли импульса оставшейся части сие-
темы. Другими словами, отдельные части замкнутой сис- темы могут только обмениваться импульсами. Обнаружив в некоторой системе приращение импульса, мы можем утверждать, что это приращение произошло за счет убы- ли импульса в окружающих телах. В этом смысле уравнения C.4) и C.5) следует рас- сматривать как более общую формулировку закона со- хранения импульса, формулировку, в которой указана причина изменения импульса у незамкнутой системы — действие других тел (внешних сил). Сказанное справед- ливо, разумеется, только по ».¦ отношению к инерциальным системам отсчета. Импульс может сохра- няться и у незамкнутой сис- темы при условии, что ре- зультирующая всех внешних сил равна нулю. Это непо- средственно вытекает из ¦'"' уравнений C.4) и C.5). В практическом отношении со- Рис- 3-2 хранение импульса в этих случаях представляет особый интерес, ибо дает возмож- ность получать достаточно простым путем ряд сведений о поведении системы, не вникая в детальное рассмотре- ние процесса. И еще. У незамкнутой системы может сохраняться не сам импульс р, а его проекция рх на некоторое направле- ние х. Это бывает тогда, когда проекция результирующей внешней силы FBHein на направление х равна нулю, т. е. вектор FBhpih перпендикулярен ему. Действительно, спрое- цировав уравнение C.4), получим dpJtt^F^x, C.7) откуда следует, что если РРНешж = 0, то px=const. Напри- мер, при движении системы в однородном поле сил тя- жести сохраняется проекция ее импульса на любое гори- зонтальное направление, что бы в системе ни происхо- дило. Рассмотрим примеры на закон сохранения импульса. Пример 1. Движущаяся частица распалась на две частицы с им- пульсами pi и р2, угол между которыми равен 0. Найдем модуль им> пульпа п распавшейся частицы. Подобного рода вопросы проще всего решать с помощью тре> угольника импульсов (рис. 3.2), который выражает закон сохранения
импульса: p = Pi+pa- Остается воспользоваться теоремой косинусов, и мы сразу можем записать, что -V, ¦ 2pl pi cos 8 . В этих рассуждениях предполагалось, что система замкнута. Если же она находится под действием каких-то внешних сил, то под импульсами р, р4 и р2 надо понимать те значения этих величин, ко- торые они имели непосредственно до и после распада, а сам про- цесс распада считать протекающим за очень малое время. Послед- нее необходимо для того, чтобы импульс внешних сил за время рас- пада был пренебрежимо мал. Пример 2. Человек массы mi находится на узком плоту массы тг, который покоится на поверхности озера. Человек совершил пере- мещение Аг' относительно плота и остановился. Сопротивление воды пренебрежимо мало. Найдем соответствующее перемещение Аг2 пло- та относительно берега. В данном случае результирующая всех внешних сил, действую- щих на систему человек — плот, равна нулю, поэтому импульс этой системы меняться не будет, оставаясь равным нулю в процессе дви- жения: mi Vi -\-m4V1 = О, где Vi и V2 — скорости человека и плота относительно берега. Но ско- рость человека относительно берега можно представить в виде Vi = =V2+V, где v' — скорость человека относительно плота. Исключив Vi из этих двух уравнений, получим Щ ,' уп = — V 2 т1 + т.2 Умножив обе части на At, найдем связь между элементарными пере- мещениями плота dr2 и человека dr' относительно плота. Такая же связь будет, очевидно, и для конечных перемещений: Дг2=- ^—Дг'. Отсюда видно, что перемещение плота dr2 не зависит от характера движения человека, т. е. не зависит от закона v'(t). Подчеркнем еще раз: закон сохранения импульса вы- полняется только в инерциальных системах. Это, однако, не исключает случаев, когда импульс системы сохранял- ся бы и в неинерциальных системах отсчета. Для этого достаточно, чтобы в уравнении C.4), справедливом и в неинерциальных системах отсчета, внешняя сила FBHem (она включает в себя и силы инерции) была равна нулю. Ясно, что такое положение может осуществляться лишь при специальных условиях. Соответствующие случаи до- вольно редки и имеют частный характер, 70
Теперь покажем, что если импульс системы сохраняется в одной инерциальной /(-системе отсчета, то он сохраняется и в любой дру- гой инерциальной Д'-системе. Пусть в /(-системе ^ »t; v; = const. Если /('-система движется относительно /(-системы со скоростью V, то скорость г-й частицы в /(-системе можно представить как v,- = = v,' + V, где Vi' — скорость этой частицы в ./('-системе. Тогда выра- жение для импульса системы можно преобразовать к следующему виду: 2 mi y't -f^miV = const. Вторая сумма в этом равенстве не зависит от времени. А это значит, что и первая сумма — импульс системы в /('-системе отсчета — тоже не зависит от времени, т. е. ^ mi v'{ = const'. Полученный результат полностью соответствует принципу относи- тельности Галилея, согласно которому законы механики одинаковы во всех инерциальных системах отсчета. Рассуждения, которые привели нас к закону сохра- нения импульса, целиком опирались на справедливость законов Ньютона. В частности, предполагалось, что мате- риальные точки замкнутой системы взаимодействуют между собой попарно и это взаимодействие подчиняется третьему закону Ньютона. А как обстоит дело в случае систем, не подчиняющихся законам Ньютона, например в системах с электромагнитным излучением? Ответ на этот вопрос дает опыт, который со всей убе- дительностью показывает, что закон сохранения импуль- са оказывается справедливым и для таких систем. Одна- ко в этих случаях в общем балансе импульса необходимо учитывать не только импульсы частиц, но и импульс, ко- торым обладает, как выясняется в электродинамике, само электромагнитное поле. Таким образом, опыт показывает, что закон сохране- ния импульса, надлежащим образом обобщенный, пред- ставляет собой фундаментальный закон природы, не знающий никаких исключений. Но в таком широком по- нимании он уже не является следствием законов Ньюто- на, а должен рассматриваться как самостоятельный об- щий принцип, являющийся обобщением опытных фактов. § 3.4. Центр масс. Д-система Центр масс. В любой системе частиц имеется одна за- мечательная точка С, называемая центром масс, ко- 71
торая обладает рядом интересных и важных свойств. Ее положение относительно начала О данной системы от- счета характеризуется радиусом-вектором гс, определяе- мым как т C.8) где nit и ri — масса и радиус-вектор i-й частицы, т — масса всей системы (рис. 3.3). г Рис. 3.4 Пример. Покажем, что центр масс системы из двух частиц с мас- сами mi и тг находится на прямой, их соединяющей, в точке С, ко- торая делит расстояние между частицами в отношении U : /2 = = т2: тх. Пусть п, г2, Гс — радиусы-векторы частиц 1, 2 и точки С (рис. 3.4). Тогда положения этих частиц относительно точки С характери- зуются радиусами-векторами = Г1 — ГС> Г2 = Г2—ГС- После подстановки в эти равенства согласно C.8 (т\Г1 + т2Г2)/'(т\-\-т2) получим выражения гс = г^ == т2 (гх — r2), r2 = m1(r2 — гх). Отсюда следует, что векторы г/ и г2' коллинеарны (причем г/|| значит точка С лежит на прямой, проходящей через частицы (рис. 3.4). Кроме того, модули этих векторов, т. е. расстояния h и 1%, об- ратно пропорциональны массам частиц. Следует заметить, что центр масс системы совпадает с ее центром тяжести. Впрочем, это утверждение спра- ведливо лишь в том случае, когда поле сил тяжести в пределах данной системы можно считать однородным. Теперь найдем скорость Vc центра масс сис- темы. Продифференцировав C.8) по времени, получим m 72
Если скорость центра масс равна нулю, то говорят, что система как целое покоится. Это вполне естественное обобщение понятия покоя отдельной материальной точки. Скорость же Vc приобретает смысл скорости движения всей системы как целого. Из последней формулы с учетом C.3) следует, что p=mVC) C.10) т. е. импульс системы равен произведению массы систе- мы на скорость ее центра масс. Уравнение движения центра масс. Понятие центра масс позволяет придать уравнению C.4) иную форму, ко- торая часто бывает более удобной. Для этого достаточно C.10) подставить в C.4) и учесть, что масса системы как таковой есть величина постоянная. Тогда получим m dyc _F - '" .. —* внеш> at C.11) где FBIiem — результирующая всех внешних сил, действую- щих на систему. Это и есть уравнение движения центра масс системы — одно из важнейших уравне- ний механики. Согласно этому уравнению, центр масс любой системы частиц движется так, как если бы вся масса системы была сосредоточена в этой точке и к ней были бы приложены все внешние силы. При этом ускорение центра масс совершенно не зави- сит от точек приложения внешних сил. Далее, из уравнения C.11) следует, что если FBHein=0, то dVc/cU=0, а значит, Vc = const. Таков, в частности, случай замкнутой системы (в инерциальной системе от- счета). Кроме того, если Vc = const, то, согласно C.11), и импульс системы р = const. Таким образом, если центр масс системы движется равномерно и прямолинейно, то это означает, что ее им- пульс сохраняется в процессе движения. Разумеется, справедливо и обратное утверждение. Уравнение C.11) по форме совпадает с основным уравнением динамики материальной точки и является его естественным обобщением на систему частиц: ускоре- ние системы как целого пропорционально результирую- щей всех внешних сил и обратно пропорционально сум- марной массе системы. Напомним, что в неинерциальных системах отсчета результирующая всех внешних сил 73
включает в себя как силы взаимодействия с окружающи- ми телами, так и силы инерции. Рассмотрим три примера на движение центра масс системы. Пример 1. Покажем, как можно иначе решить задачу с челове- ком на плоту (см. пример 2 на с. 70), если воспользоваться понятием центра масс. Так как сопротивление воды пренебрежимо мало, то результиру- ющая всех внешних сил, действующих на систему человек — плот, равна нулю. А это значит, что положение центра масс данной сис- темы в процессе движения человека (и плота) меняться не будет, т. е. /Я; ri + т-2 Г2 = const, где 14 и Гг — радиусы-векторы, характеризующие положения центров масс человека и плота относительно некоторой точки берега. Из это- го равенства найдем связь между приращениями векторов ri и г^: /rei'Ari + mi Дг2 = 0. Имея в виду, что приращения An и Дг2 представляют собой переме- щения человека и плота относительно берега, причем Дг1 = Дг2+Дг', найдем перемещение плота: Дг2 = — Дг'. Ш\ + /Я 2 Пример 2. Человек прыгает с вышки в воду. Движение прыгуна в общем случае имеет весьма сложный характер. Однако если сопро- тивление воздуха пренебрежимо мало, то можно сразу утверждать, что центр масс прыгуна движется по параболе, как материальная точка, на которую действует постоянная сила mg, где т — масса че- ловека. Пример 3. Замкнутая цепочка, соединенная нитью с концом оси центробежной машины, равномерно вращается вокруг вертикальной оси с угловой скоростью и (рис. 3.5). При этом нить образует угол О с вертикалью. Как ведет себя центр масс цепочки? Прежде всего ясно, что при равномерном вращении центр масс цепочки не движется в вертикальном направлении. Это значит, что вертикальная составляющая силы Г натяжения нити компенсирует силу тяжести mg (рис. 3.5, справа). Горизонтальная же составляю- щая силы натяжения постоянна по модулю и все время направлена к оси вращения. Отсюда следует, что центр масс цепочки (точка С) движется по горизонтальной окружности, радиус которой р легко найти с помощью уравнения C.11), записав его в виде т о>2 р = mg tg 9, где т — масса цепочки. При этом точка С все время находится меж- ду осью вращения и нитью, как показано на рисунке. ^-система. В тех часто встречающихся случаях, когда нас интересует лишь относительное движение частиц внутри системы, а не ее движение как целого, наиболее целесообразно пользоваться системой отсчета, в которой 74
центр масс покоится. Это позволяет значительно упрос- тить и анализ явления, и соответствующие расчеты. Систему отсчета, жестко связанную с центром масс и перемещающуюся поступательно по отношению к инер- циальным системам, называют системой центра масс или, кратко, Ц-с и с т е м о й. Отличительной особенностью /(-системы является то, что полный импульс системы частиц в ней всегда равен нулю — это непосредственно следует из (Ьормулы C.10), ибо в /(-системе Vc = 0. Дру- гими словами, любая систе- ма частиц как целое покоит- ся в своей /(-системе. Для замкнутой системы частиц ее /(-система являет- ся инерциальной, для незам- кнутой — в общем случае не- инерциальной. Необходимо отметить, что /(-система играет существен- ную роль в физике. Это обус- ловлено рядом несомненных преимуществ, которые дает ее применение во многих си- туациях. В дальнейшем мы будем обращаться к этой сис- теме отсчета неоднократно (в теории столкновений час- тиц, в динамике твердого тела и др.). Система из двух частиц. Пусть массы частиц равны Ш\ и т% а их скорости в исходной /(-системе отсчета — V] и V2 соответственно. Найдем импульсы этих частиц в //-системе. Будем помечать все величины в /(-системе сверху значком ~ (тильда). Тогда искомые импульсы можно записать так: Рис. 3.5 где Vc — скорость /(-системы относительно /(-системы отсчета. После подстановки в эти формулы выражения V^= (mtVi + moVo)l(ml + m2) получим ,-vJ. C.12) pi= :2 (vi-v2), p2 nil ~r ni2 m\ ~r m.2 Видно, что импульсы обеих частиц в /(-системе оди- наковы по модулю и противоположны по направлению: 75
pi=—p2. Это так и должно быть, поскольку суммарный импульс частиц в Д-системе всегда равен нулю. Полученные результаты справедливы независимо от того, замкнута эта система или нет, а также независимо от наличия взаимодействия между частицами. § 3.5. Движение тела переменной массы Имеется много случаев, когда масса тела изменяется в процессе движения за счет непрерывного отделения или присоединения вещества (ракета, реактивный самолет, платформа, нагружаемая на ходу, и др.). Наша задача: найти уравнение движения такого тела. Рассмотрим решение этого вопроса для материальной точки, называя ее для краткости телом. Пусть в некото- рый момент t масса движущегося тела А равна т, а при- соединяемое (или отделяемое) вещество имеет скорость и относительно данного тела. Введем вспомогательную инерциальную /С-систему отсчета, скорость которой такова же, как и скорость те- ла Л в данный момент t. Это значит, что в момент t те- ло А покоится в /(-системе. Пусть далее за промежуток времени от t до t-\-dt те- ло А приобретает в К-системе импульс mdv. Этот импульс тело А получит, во-первых, вследствие присоединения (отделения) массы 8т, которая приносит (уносит) им- пульс Ьт-а, и, во-вторых, вследствие действия силы F со стороны окружающих тел или силового поля. Таким об- разом, можно записать, что mdv=Fd/± Ьти, где знак плюс соответствует присоединению массы, а знак минус — отделению. Оба эти случая можно объединить, представив ±б/и в виде приращения dm массы тела А (действительно, в случае присоединения массы dm=+6m, а в случае отделения dm ——8т). Тогда предыдущее уравнение примет вид C.13) •и. Поделив это выражение на dt, получим dv „ . dm т — =F-4 u, At ' dt где u — скорость присоединяемого (или отделяемого) ве- щества относительно рассматриваемого тела. 76
Это уравнение является основным уравнением Динамики точки переменной массы. Его называют уравнением Мещерского. Будучи по- лученным в одной инерциальной системе отсчета, это уравнение в силу принципа относительности справедливо и в любой другой инерциальной системе. Заметим, что если система отсчета неинерциальна, то под силой F сле- дует понимать результирующую как сил взаимодействия данного тела с окружающими телами, так и сил инерции. Последний член уравнения C.13) носит название ре- активной силы: R= (dm/dt)u. Эта сила возникает в результате действия на данное тело присоединяемой (или отделяемой) массы. Если масса присоединяется, то dm/d?>0 и вектор R совпадает по направлению с векто- ром и; если же масса отделяется, то dm/dt<0 и вектор R противоположен вектору и. Уравнение Мещерского по своей форме совпадает с основным уравнением динамики материальной точки постоянной массы: слева — произведение массы тела на ускорение, справа — действующие на него силы, включая реактивную силу. Однако в случае переменной массы нельзя внести массу т под знак дифференцирования и представить левую часть уравнения как производную по времени от импульса, ибо md\/dtФ d (m\) /dt. Обратим внимание на два частных случая: 1. Если и = 0, т. е. масса присоединяется или отделя- ется без скорости относительно тела, то R=0 и уравне- ние C.13) принимает вид m(t)^r = F; C.14) где т{1)—масса тела в данный момент времени. Это уравнение определяет, например, движение платформы, из которой свободно высыпается песок (см. задачу 3.7, п. 1). 2. Если u = —v, т. е. присоединяемая масса непод- вижна в выбранной системе отсчета или отделяемая масса становится неподвижной в этой системе, то урав- нение C.13) принимает другой вид: m(dv/d/) + (dm/dt)v=F, или (mv)=F. C.15) dr Иначе говоря, в этом частном случае — и только этом — действие силы F определяет изменение импульса тела с 77
переменной массой. Данный случай реализуется, напри- мер, при движении платформы, нагружаемой сыпучим веществом из неподвижного бункера (см. задачу 3.7, п. 2). Рассмотрим пример на применение уравнения Мещер- ского. Пример. Ракета движется в инерциальной /(-системе отсчета в отсутствие внешнего силового поля, причем так, что газовая струя вылетает с постоянной относительно ракеты скоростью и. Найдем зависимость скорости v ракеты от ее массы т, если в момент старта ее масса была равна то- В данном случае F = 0 и из уравнения C.13) следует dv =u dm/m. Проинтегрировав это выражение с учетом начальных условий, по- лучим v=—u In (mo/m), A) где знак минус показывает, что вектор v (скорость ракеты) про- тивоположен по направлению вектору и. Отсюда видно, что скорость ракеты в данном случае (u = const) не зависит от времени сгорания топлива: v определяется только отношением начальной массы то ра- кеты к оставшейся массе т. Заметим, что если бы вся масса горючего была одновременно выброшена со скоростью и относительно ракеты, то скорость послед- ней оказалась бы иной. Действительно, если ракета вначале покои- лась в выбранной инерциальной системе отсчета, а после одновремен- ного выброса всего горючего приобрела скорость v, то из закона со- хранения импульса для системы ракета — горючее следует О = /rev + (m0 — т) (u + v), где u+v — скорость горючего относительно данной системы отсчета. Отсюда v= -u(l-m/m0). B) Скорость v ракеты в этом случае оказывается меньше, км в преды- дущем (при одинаковых значениях отношения то/т). В этом не- трудно убедиться, сравнив характер зависимости v от mo/m в обоих случаях. С ростом то/гп в первом случае (когда вещество отделяется непрерывно) скорость v ракеты, согласно A), растет неограниченно, во втором же (когда вещество отделяется одновременно) скорость v, согласно B), стремится к пределу, равному — и. Задачи 0 3.1. Частица движется с импульсом р(^) под действием силы F(^). Пусть а и b — постоянные векторы, причем aJ_b. Полагая, что 1) р(^)=а+^A—с^)Ь, где а — положительная постоянная, най- ти вектор F в те моменты времени, когда F_Lp; 2) F(^)=a + 2rt> и р(О)=ро, где р0 — вектор, противоположный по направлению вектору а, найти вектор р в момент to, когда он окажется повернутым на 90° по отношению к вектору ро- Решение. 1. Сила F=dp/d/=(l—2at)b, т. е. вектор F все время перпендикулярен вектору а. Следовательно, вектор F будет 78
перпендикулярен вектору р в те моменты, когда коэффициент при b в выражении для р(^) обращается в нуль. Отсюда ^ = 0 и t2=lja. Соответствующие значения вектора F равны: F1 = b, F2=— b. 2. Приращение вектора р за промежуток времени &t есть dp = = Fdt Интегрируя это уравнение с учетом начальных условий, на- ходим тдт р—р0 = о где, по условию, ро противоположен вектору а. Вектор р окажется перпендикулярным вектору ро в момент to, когда ato = po. В этот / момент р=(ро/аJЬ. /*>V / *- ф 3.2. Орудие массы т со- скальзывает по гладкой наклонной плоскости, составляющей угол а с горизонтом. В момент, когда ско- рость орудия оказалась равной v, m& произвели выстрел, в результате которого орудие остановилось, а Рис. 3.6 вылетевший в горизонтальном на- правлении снаряд «унес» импульс р. Пусть продолжительность вы- стрела равна т. Найти среднее за это время значение силы реакции R со стороны наклонной плоскости. Решение. Здесь система орудие — снаряд незамкнутая. За время т эта система получает приращение импульса, равное р — т\. Изменение импульса системы обусловлено действием двух внешних сил: силы реакции R (она перпендикулярна наклонной плоскости) и силы тяжести mg. Поэтому можно написать р — /nv = < R > и + mgt, где <R> —среднее за время т значение вектора R. Это соотношение очень полезно представить графически (рис. 3.6). Из данного рисун- ка сразу видно, что искомое значение модуля <R> определяется формулой </?> = (р sin a + mg t]cos а)/и. 0 3.3. Закон сохранения импульса. Две тележки, каждая массы М, движутся друг за другом по инерции (без трения) с одинаковой скоростью Vo. На задней тележке находится человек массы т. В не- который момент человек прыгнул в переднюю тележку со скоростью и относительно своей тележки. Какой стала скорость передней те- лежки? Решение. Импульс всей системы в результате того, что чело- век перепрыгнул из задней B) тележки в переднюю A), не изменит- ся поэтому BМ + т) v0 = М \'2 + (М + т) \[ , где vi' и v2' — конечные скорости тележек. 79
Аналогично запишем баланс импульсов для задней течежки с че- ловеком (до н после перепрыгивания): (М + т) v0 = M v'2 -f m (v'2 + u), где v2' + u — скорость спрыгнувшего человека относительно полотна дороги. Из этих двух уравнений следует, что тМ (Bi- ll. ГТ) 3.4. На краю покоящейся тележки массы М стоят два челове- ка, каждый массы т. Пренебрегая трением, найти У///////А скорость тележки после того, как оба человека спрыгнут с одной и той же горизонтальной ско- ростью и относительно тележки: 1) одновременно; 2) друг за другом. В каком случае скорость те- лежки будет больше и во сколько раз? Решение. 1. Согласно закону сохранения импульса, . Mv' +2m(v' +u) =0, Ar где v — скорость тележки, v' + u — скорость чело- века (обе скорости относительно полотна дороги). Отсюда v = — и. М + 2т 2. В этом случае необходимо записать два уравнения. Когда спрыгнул один человек, то (М + т) v' + m (v' + и) = 0, где v' — скорость тележки с оставшимся вторым человеком. Когда же спрыгнул другой человек, то (М + т) v' = M v" + m (v" + u), где х" — скорость пустой тележки. Исключив из последних двух уравнений v', найдем М-т Рис. 3.7 v" = BМ + 3/га) т (М -+- т) (М + 2т) -и. Отношение скорости тележки v" в случае 2) к скорости v' в слу- чае 1) равно V" v' = !+¦ 2 (М + т) ф 3.5. Центр масс. Через блок перекинут шнур, на одном конце которого находится лестница с человеком, а на другом конце — урав- новешивающий груз массы М (рис. 3.7). Человек, масса которого т, совершил вверх перемещение Дг' относительно лестницы и остано- 80
вился. Пренебрегая массами блока и шнура, найти перемещение центра масс этой системы. Решение. В системе отсчета, связанной с осью блока, положе- ние центра масс данной системы характеризуется радиусом-вектором гс = [М rj + Щ — т) г2 + т Т3]/2М, где ri, г2 и гз — радиусы-векторы центров масс уравновешивающего груза, лестницы и человека — все относительно некоторой точки О выбранной системы отсчета. Отсюда перемещение Агс центра масс системы l—m) Дг3]/2Л1, Рис. 3.8 где Дг1, Дг2 и Лгз — перемещения уравновешивающего груза, лестни- цы и человека. Имея в виду, что = — Дг2, Дг3 = Дг2+ Дг', получим в результате Дгс = (т/2М) Дг'. Таким образом, перемещение центра масс всей системы совпадает по направлению с перемещением человека относительно лестницы, и по- лученный результат не зависит от характера движения человека. Замечание. На первый взгляд может показаться, что данная система «замкнута», т. е. результирующая всех внешних сил равна нулю, и центр масс системы не должен переместиться. Однако это не так. Когда человек начинает подниматься, он действует на лест- ницу с дополнительной силой, направленной вниз. В результате натя- жение шнура возрастает и внешняя сила, действующая на систему со стороны подвеса, окажется больше суммарной силы тяжести. Поэто- му результирующая всех внешних сил будет направлена вверх — она и обусловливает перемещение вверх центра масс всей системы. ф 3.6. Д-система. Две небольшие шайбы, массы которых т\ и т2, связаны между собой нитью длины / и движутся по гладкой го- ризонтальной плоскости. В некоторый момент скорость одной шайбы равна нулю, а другой — v, причем ее направление перпендикулярно нити (рис. 3.8, а). Найти силу натяжения нити в процессе движения. Решение. Перейдем в систему центра масс — Д-систему. В этой системе отсчета шайбы движутся по окружностям вокруг 81
центра масс С (рис. 3.8, б), поэтому искомую силу можно найти как F = mCv\lh, A) где Hi — скорость шайбы массы т\, U — радиус окружности по ко- торой она движется. Подобное выражение можно было бы, конечно, записать и для другой шайбы — это несущественно. Найдем значения 1\ и Ь\. В примере на с. 72 было показано, что отношение hlh=m^lmi. Кроме того, /i + /2 = /. Из этих двух соотно- шений следует /1 = //и2/(/га1 + /га2). B) Далее, скорость vt = Vi—Vc. В нашем случае У[ = 0 и Vc = m2vlmx->r +m2). Поэтому модуль вектора Vi = m2v/ (щ + тг). C) Эта величина в процессе движения остается постоянной. После подстановки B) и C) в A) получим ф 3.7. Движение тела переменной массы. Железнодорожная платформа в момент ^=0 начинает двигаться под действием посто- янной силы тяги F. Пренебрегая трением в осях, найти зависимость от времени скорости платформы v(t), если: 1) платформа нагружена песком, который высыпается через от- верстие в ее дне с постоянной скоростью ц (кг/с), а в момент ^=0 масса платформы с песком равна /п0; 2) на платформу, масса которой т0, в момент ^ = 0 начинает вы- сыпаться песок из неподвижного бункера так, что скорость погрузки постоянна и равна ц(кг/с). Решение. 1. В этом случае реактивная сила равна нулю и уравнение C.13) имеет вид (m0—y.t)dv/dt = F, откуда Проинтегрировав это уравнение с учетом начальных условий, по- лучим F т0 v=—¦ In —— . 2. Здесь горизонтальная составляющая реактивной силы (а только эта составляющая нас и интересует) R=i|i(—v), где v — ско- рость платформы. Поэтому уравнение C.13) приводится к виду C.15), или Интегрирование с учетом начальных условий дает /rev = F t, где m = mo + \it. Отсюда V = 82
Полученные в обоих случаях выражения справедливы, разуме- ется, лишь в процессе разгрузки (или погрузки) платформы. ф 3.8. Ракета поддерживается в воздухе на постоянной высоте, выбрасывая вертикально вниз струю газа со скоростью и. Найти: 1) сколько времени ракета сможет оставаться на этой высоте, если начальная масса топлива составляет т]-ю часть ее массы (без топлива); 2) какую массу ii(t) газов должна ежесекундно выбрасывать ракета, чтобы оставаться на постоянной высоте, если начальная мас- са ракеты (с топливом) равна «о. Решение. 1. В данном случае dv/cU=0 и уравнение C.13) примет вид и = 0, или после разделения переменных d/и/да = — (g/u) At. A) Интегрирование этого уравнения дает 1п(л|/1Ио)=— (g/u)t. B) Отсюда t = (u/g) In (mo/m) = (u/g) In A + т)), где учтено, что г|= (та—m)Jm. 2. Из уравнения A) предыдущего пункта следует, что --(g/u) m, где т находим из B): tn=moe'~g^u . В результате По такому закону ц меняется со временем в течение промежутка времени, найденного в п. 1. ф 3.9. Ракета поднимается с нулевой начальной скоростью вер- тикально вверх в однородном поле тяжести. Первоначальная масса ракеты (с топливом) равна т0. Скорость газовой струя постоянна и равна и относительно ракеты. Пренебрегая сопротивлением воздуха, найти скорость v ракеты в зависимости от ее массы т и времени подъема t. Решение. Запишем уравнение движения ракеты — уравнение (З'ЛЗ) —в проекциях на вертикальную ось с положительным направ- лением вверх: du Am Перепишем это уравнение так: d Am m—(v + gt)=~u—, откуда A (v + gt) = — и dm/да. 83
Проинтегрировав с учетом начальных условий последнее уравнение, получим v + gt = — ula(m/m0). Искомая скорость ракеты v = и In (щ/т) — gt. фЗ.10. Космический корабль массы т0 движется в отсутствие внешнего силового поля с постоянной скоростью vo- Для изменения направления движения был включен реактивный двигатель, который стал выбрасывать струю газа с постоянной относительно корабля скоростью и, причем вектор и все время перпендикулярен направле- нию движения корабля. В конце работы двигателя масса корабля стала равной т. На какой угол изменилось направление движения корабля за время работы двигателя? Решение. Найдем приращение вектора скорости корабля за промежуток времени di. Умножив обе части уравнения C.13) на d/ и учитывая, что F=0, получим dv =u dm/т. Здесь dm<0. Так как вектор u все время перпендикулярен вектору v (скорости корабля), то модуль вектора v не меняется и остается равным своему первоначальному значению: | v ] = fо- Отсюда следу- ет, что угол поворота da вектора v за время d/ определяется как da = | dv \/v0 = (u/v0) | dm/m | . Проинтегрировав это уравнение, найдем a = (и/v0) In (щ/т). Глава 4 ЗАКОН СОХРАНЕНИЯ ЭНЕРГИИ § 4.1. Работа и мощность Работа. Пусть частица под действием силы F соверша- ет перемещение по некоторой траектории /—2 (рис. 4.1). В общем случае сила F в процессе движения частицы может меняться как по модулю, так и по направлению. Рассмотрим элементарное перемещение dr, в пределах которого силу F можно считать постоянной. Действие силы F на перемещении dr характеризуют величиной, равной скалярному произведению Fdr, кото- рую называют элементарной работой силы F на перемещении dr. Ее можно представить и в другом виде: Fdr=/7cosi где a — угол между векторами F и dr, ds= |dr| —эле- ментарный путь, Fs — проекция вектора F на вектор dr (рис. 4.1). 84
Итак, элементарная работа силы F на перемещении dr bA=Fdr = F^is. D.1) Величина &А — алгебраическая: в зависимости от угла между векторами F и dr (или от знака проекции Fs век- тора г на вектор dr) она может быть как положительной, так и отрицательной и, в частности, равной нулю (если F±dr, т. е. Ft = 0). Рис. 4.1 Рис. 4.2 Суммируя (интегрируя) выражение D.1) повеем эле- ментарным участкам пути от точки / до точки 2, найдем работу силы FHa данном пути: D.2) Отметим следующее важное обстоятельство: формула D.2) справедлива не только для частицы, но и вообще для любого тела (или системы тел). Надо только иметь в виду, что под dr (или ds) следует понимать перемеще- ние точки приложения силы F. Игнорирование этого об- стоятельства зачастую приводит к ошибочным резуль- татам. Выражению D.2) можно придать наглядный геомет- рический смысл. Изобразим график Fs как функцию по- ложения частицы на траектории. Пусть, например, этот график имеет вид, показанный на рис. 4.2. Из рисунка видно, что элементарная работа бЛ численно равна пло- щади заштрихованной полоски, а работа А на пути от точки / до точки 2 — площади фигуры, ограниченной кри- вой, ординатами / и 2 и осью s. При этом площадь фигу- ры над осью s берется со знаком плюс (она соответствует положительной работе), а площадь фигуры под осью s — 85
СО знаком минус (она соответствует отрицательной ра- боте). Рассмотрим несколько примеров на вычисление ра- боты. Работа упругой силы F=—хг, где г — радиус-вектор частицы М относительно точки О (рис. 4.3, а). Перемес- тим частицу М, на которую действует эта сила, по про- извольному пути из точки / в точку 2. Найдем сначала элементарную работу силы F на элементарном переме- щении dr: Скалярное произведение rdr=r(dr)r , где (dr)r —проек- ция dr на вектор г. Эта проекция равна dr — приращению модуля вектора г. Поэтому rdr=rdr и ЬА = — хг dr = — d (xr2/2). Теперь вычислим работу данной силы на всем пути, т. е. проинтегрируем последнее выражение от точки / до точ- ки 2: А= — \ d У.Г2 ХГ, D.3) Работа гравитационной (или кулоновской) силы. Пусть в точке О (рис. 4.3,6) находится неподвижный си- ловой центр — материальная точка, действующая на час- тицу М с силой F, которая как для гравитационного, так и для кулоновского взаимодействий может быть пред- ставлена в виде 86
где а — соответствующая постоянная (—ymitn2 или kqiq2), г— расстояние от точки О до частицы М, ег— орт радиуса-вектора г. Элементарная работа этой силы на перемещении dr 8yl=Fdr = (a/r2)erdr. Скалярное произведение e/-dr = dr, т. е. равно приращению модуля вектора г, поэтому ЬА = айг/г2= — d(a/r). Работа же этой силы на всем пути от точки / до точки 2 к, О ч Рис. 4.4 D.4) Работа однородной силы тяжести F=mg. Запишем эту силу в виде F =—mgk, где к — орт вертикальной оси z, положительное направление которой выбрано вверх (рис. 4.4). Элементарная работа силы тяжести на перемеще- нии dr bA=F dr=—mgk dr. Скалярное произведение kdr= (dr)k, где (dr)k —проек- ция dr на орт к, равная d2 — приращению координаты г. Поэтому kdr=d2 и ЪА = — mg dz= — ( Работа же данной силы на всем пути от точки 1 до точ- ки 2 А=- =mg{zx — zz). D.5) Рассмотренные силы интересны в том отношении, что их работа, как видно из формул D.3) — D.5), не зависит от формы пути между точками 1 и 2, а зависит только от положения этих точек. Эта весьма важная особенность данных сил присуща, однако, не всем силам. Например, сила трения этим свойством не обладает: работа этой силы зависит не только от положения начальной и ко- нечной точек, но и от формы пути между ними, 87
До сих пор речь шла о работе одной силы. Если же на частицу в процессе движения действуют несколько сил, результирующая которых F=Fi + F2+..., то нетрудно по- казать, что работа результирующей силы F на некотором перемещении равна алгебраической сумме работ, совер- шаемых каждой из сил в отдельности на том же переме- щении. Действительно, Ь--. D-6) Единицей работы в СИ является джоуль (Дж). Джоуль — это работа силы в 1 Н на пути в 1 м (при ус- ловии, что направление силы совпадает по направлению с перемещением), или 1 Дж=1 Н-м. Мощность. Для характеристики скорости, с которой совершается работа, вводят величину, называемую мощ- ностью. Мощность, по определению,— это работа, со- вершаемая силой за единицу времени. Если за промежу- ток времени At сила F совершает работу Fdr, то мощ- ность, развиваемая этой силой в данный момент времени, есть iV = Fdr/d/. Учитывая, что dr/d^ = v, получим D.7) Таким образом, мощность, развиваемая силой F, рав- на скалярному произведению вектора силы на вектор скорости, с которой движется точка приложения данной силы. Как и работа, мощность — величина алгебраиче- ская. Зная мощность силы F, можно найти и работу, кото- рую совершает эта сила за промежуток времени t. В са- мом деле, представив подынтегральное выражение в D.2) в виде Fdr=Fvd^ = iVdz1, получим Единицей мощности в СИ является ватт (Вт), рав- ный джоулю в секунду (Дж/с). В заключение обратим внимание на одно весьма су- щественное обстоятельство. Когда говорят о работе (или мощности), то необходимо в каждом конкретном случае четко указывать или представлять себе, работа какой
именно силы (или сил) имеется в виду. В противном слу- чае, как правило, неизбежны недоразумения. § 4.2. Консервативные силы. Потенциальная энергия Консервативные силы. Если в каждой точке простран- ства на помещенную туда частицу действует сила, то го- ворят, что частица находится в поле сил. Так, напри- мер, частица может находиться в поле сил тяжести, в по- ле упругих сил, в поле сил сопротивления (в потоке жид- кости, газа) и т. д. Поле, остающееся постоянным во времени, называют стационарным. Стационарное поле в одной системе отсчета может оказаться нестационарным ч другой сис- теме отсчета. В стационарном силовом поле сила, дейст- вующая на частицу, зависит только от ее положения. Работа, которую совершают силы поля при перемеще- нии частицы из точки / в точку 2, зависит, вообще гово- ря, от пути между этими точками. Вместе с тем имеются стационарные силовые поля, в которых работа, совершае- мая над частицей силами поля, не зависит от пути меж- ду точками 1 я 2. Силы, обладающие таким свойством, называют консервативными. Это свойство консервативных сил можно сформулировать и ина- че: силы поля являются консервативными, если в стационарном слу- чае их работа на любом замкнутом пути равна нулю. Чтобы убе- диться в этом, разобьем произвольный замкнутый контур на две части: 1а2 и 2Ы (рис. 4.5). Тогда работа А на замкнутом пути А=. Нетрудно сообразить, что Агы = —А\ъг> поэтому А — АХа2— АХЬ2. 89
А так как в нашем случае работа не зависит от пути, т. е. /4ia2= =ЛцJ, то в результате и оказывается, что работа на произвольном замкнутом пути действительно равна нулю: А = 0. Все силы, не являющиеся консервативными, называют неконсервативными. К числу неконсервативных сил относятся, например, силы трения и сопротивления. Работа этих сил зависит, вообще говоря, от пути между начальным и конечным положениями частицы (и не рав- на нулю на любом замкнутом пути). Поле центральных сил. Всякое силовое поле вызыва- ется действием определенных тел. Сила, действующая на частицу М в таком поле, обусловлена взаимодействием этой частицы с данными телами. Силы, зависящие только от расстояния между взаимодействующими частицами и направленные по прямой, проходящей через эти частицы, называют центральными. Примером последних яв- ляются силы гравитационные, кулоновские и упругие. Центральную силу, действующую на частицу М со стороны частицы О, можно представить в таком виде: F = /(r)er, D.8) где f(r)—функция, зависящая при данном характере взаимодействия только от г — расстояния между части- цами; ег — единичный вектор, задающий направление радиуса-вектора частицы М относительно частицы О (рис. 4.6). Оказывается, центральные силы являются консерва- тивными. Для доказательства этого утверждения найдем сначала работу центральной силы в случае, когда сило- вое поле вызвано наличием одной неподвижной части- цы О. Элементарная работа силы D.8) на перемещении dr есть 6А = Fdr = /(r)erdr. Так как erdr = dr — проекция вектора dr на вектор ег или на соответствующий радиус- вектор г (см. рис. 4.4), то 8A = f(r)dr. Работа же этой силы на произвольном пути от точки 1 до точки 2 2 Полученное выражение зависит только от вида функции /(г), т. е. от характера взаимодействия, и от значений г\ и г2 — начального и конечного расстояний между части- цами М и О. От пути оно никак не зависит. Обобщим полученный результат на стационарное си- ловое поле, вызванное наличием совокупности неподвиж- 90
То обстоя- ных частиц, действующих на частицу М с силами Fb F2, ..., каждая из которых является центральной. В этом слу- чае работа результирующей силы при перемещении час- тицы М из одной точки в другую равна алгебраической сумме работ отдельных сил. А так как работа каждой из этих сил не зависит от пути, то и работа результирующей силы также не зависит от пути. Вывод: поскольку центральные силы обладают таким свойством, они являются консервативными. Потенциальная энергия частицы в поле. тельство, что работа консерватив- ных сил в случае стационарного v поля зависит только от начально- го и конечного положений части- цы, дает возможность ввести чре- звычайно важное понятие потен- циальной энергии. Представим себе стационарное поле консервативных сил, в кото- ром мы перемещаем частицу из разных точек /\- в некоторую фик- сированную точку О. Так как работа сил поля не зави- сит от пути, то остается зависимость ее только от поло- жения точки Р (при фиксированной точке О). А это зна- чит, что данная работа будет некоторой функцией радиу- са-вектора г точки Р. Обозначив эту функцию ?/(г), за- пишем о Apo=\?te=U{r). D.9) р Функцию U(г) называют потенциальной энерги- е й частицы в данном поле. Найдем работу сил поля при перемещении частицы из точки 1 в точку 2 (рис. 4.7). Так как работа не зависит от пути, выберем путь, проходящий через точку О. Тогда работа на пути 102 может быть представлена в виде Рис. 4.7 или с учетом D.9) D,10)
Выражение, стоящее справа, есть убыль* потенциаль- ной энергии, т. е. разность значений потенциальной энер- гии частицы в начальной и конечной точках пути. Таким образом, работа сил поля на пути 1—2 равна убыли потенциальной энергии частицы в данном поле. Очевидно, частице, находящейся в точке О поля, всег- да можно приписать любое наперед выбранное значение потенциальной энергии. Это соответствует тому обстоя- тельству, что работа сил поля определяет лишь разность потенциальных энергий в двух точках, но не их абсолют- ное значение. Однако как только фиксирована потенци- альная энергия в какой-либо точке, значения ее во всех остальных точках поля однозначно определяются фор- мулой D.10). Формула D.10) дает возможность найти выражение U(r) для любого стационарного поля консервативных сил. Для этого достаточно вычислить работу, совершае- мую силами поля на любом пути между двумя точками, и представить ее в виде убыли некоторой функции, кото- рая и есть потенциальная энергия U{r). Именно так и было сделано при вычислении работы в полях упругой и гравитационной (кулоновской) сил, а также в однородном поле сил тяжести [см. формулы D.3) — D.5)]. Из этих формул сразу видно, что потенци- альная энергия частицы в данных силовых полях имеет следующий вид: Г) в поле упругой силы ?/(г)=хг2/2; D.11) 2) в гравитационном (кулоновском) поле материаль- ной точки U[r)=afr, D.12) * Изменение какой-либо величины X можно характеризовать либо ее приращением, либо убылью. Приращением величи- ны X называют разность конечного (Х2) и начального (Xi) значений этой величины: приращение Д X = X 2 — Хх. Убылью величины X называют разность ее начального (Х4) и конеч- ного (Х2) значений: убыль X-i — Xi = — ДА', т. е. убыль величины X равна ее приращению, взятому с обратным знаком. Приращение и убыль — величины алгебраические: если, напри- мер, Хг<Хи то приращение отрицательно, а убыль положительна. 92
3) в однородном поле сил тяжести D.13) Еще раз подчеркнем, что потенциальная энергия U — функция, которая определяется с точностью до прибав- ления некоторой произвольной постоянной. Это обстоя- тельство, однако, совершенно несущественно, ибо во все формулы входит только разность значений U в двух по- ложениях частицы. Поэтому произвольная постоянная, одинаковая для всех точек поля, выпадает. В связи с этим ее обычно опускают, что и сделано в трех предыду- щих выражениях. И еще одно важное обстоятельство. Потенциальную энергию следует относить не к частице, а к системе взаи- модействующих между собой частицы и тел, вызывающих силовое поле. При данном характере взаимодействия по- тенциальная энергия взаимодействия частицы с данными телами зависит только от положения частицы относитель- но этих тел. Потенциальная энергия и сила поля. Взаимодействие частицы с окружающими телами можно описывать двумя способами: с помощью сил или с помощью потенциаль- ной энергии. В ньютоновской механике оба способа ис- пользуют одинаково широко. Однако первый способ об- ладает несколько большей общностью, ибо он применим и к таким силам, для которых нельзя ввести потенциаль- ную энергию (например, к силам трения). Второй же спо- соб применим только в случае консервативных сил. Наша задача — установить связь между потенциальной энергией и силой поля, точнее, определить поле сил F(r) по заданной потенциальной энергии U(г) как функции положения частицы в поле. Мы уже знаем, что при перемещении частицы из од- ной точки стационарного поля консервативных сил в дру- гую работа, которую производят силы поля, может быть представлена как убыль потенциальной энергии части- цы, т. е. Ai2=U\—f/2 = —AU. Это относится и к элемен- тарному перемещению dr, а именно: 8А=—df/, или Fdr= — 6U. D.14) Имея в виду, что Fdr=/7sds, где ds= |dr| —элементарный путь, Fs — проекция вектора F на перемещение dr, пере- пишем уравнение D.14) в форме 93
где — AU есть убыль потенциальной энергии в направле- нии перемещения dr. Отсюда D.15) т. е. проекция силы поля — вектора F—в данной точке на направление перемещения dr равна с обратным зна- ком производной потенциальной энергии U по данному направлению. Символ d/ds — частной производной — под- черкивает, что производная берется по определенному на- правлению. Перемещение dr можно взять в любом направлении, в частности вдоль координатных осей х, у, z. Если пере- мещение dr, например, параллельно оси х, то его можно представить так: dr = idx, где i — орт оси х, Ах— прира- щение координаты х. Тогда работа силы F на перемеще- нии dr, параллельном оси х, где Fx — проекция вектора F на орт i (а не на перемеще- ние dr, как в случае Fs). Подставив последнее выражение в уравнение D.14), получим Fx=-dU/dx, где символ частной производной означает, что U(x, у, z) при дифференцировании должна рассматриваться как функция одного аргумента х, остальные же аргументы дол- жны оставаться при этом постоянными. Ясно, что для про- екций Fy и Fz уравнения будут аналогичны уравнению для Fx. Итак, взяв с обратными знаками частные производ- ные функции U по х, у и z, мы найдем проекции Fx, Fu и Fz вектора F на орты i, j и к. Отсюда легко найти и сам вектор: F=Fxi + Fy} + FzK или ' ^М. D.16) дх ~ ду ' ' дг Величину, стоящую в скобках, называют градиентом скалярной функции U и обозначают grad U или V U- Мы будем пользоваться вторым, более удобным, обозначением, где у(«набла») означает символический ректор или оператор y>ir+lif+ki- DЛ7) 94
Поэтому у U формально можно рассматривать как про- изведение символического вектора V7 на скаляр U. Таким образом, связь между силой поля и потенци- альной энергией как функцией координат можно предста- вить в следующем компактном виде: D.18) т. е. сила поля F равна со знаком минус градиенту по- тенциальной энергии частицы в данной точке поля. По- следняя формула дает возможность, зная функцию U(г), восстановить поле сил F(r). Пример. Потенциальная энергия частицы в некотором поле име- ет вид: а) U(х, у) =—аху, где а — постоянная; б) ?/(г)=аг, где а — постоянный вектор, г— радиус-вектор точ- ки поля. Найдем соответствующее каждому случаю поле сил: а) F= — -— i +— j )=a(yi + xj); \ дх ду j б) представим функцию U в виде U=axx+ayy-{-azz; тогда Смысл градиента станет нагляднее и яснее, если ввести понятие эквипотенциальной поверхности- поверхности, во всех точках которой потенциальная энер- гия U имеет одно и то же значение. Ясно, что каждому значению U соответствует своя эквипотенциальная по- верхность. Из формулы D.15) следует, что проекция вектора F на любое направление, касательное к эквипотенциальной поверхности в данной точке, равна нулю. Это значит, что вектор F нормален эквипотенциальной поверхности в дан- ной точке. Далее, возьмем перемещение ds по нормали к эквипотенциальной поверхности в сторону уменьше- ния U, тогда д?/<0 и, согласно D.15), Fs>0, т. е. век- тор F направлен в сторону уменьшения U. А так как F противоположен по направлению вектору V U, то мы при- ходим к выводу, что градиент U — это вектор, направ- ленный по нормали к эквипотенциальной поверхности в сторону возрастания потенциальной энергии U. Сказанное поясняет рис. 4.8, относящийся к двумер- ному случаю. На нем изображены система эквипотенциа- лей (U\<iU2<Uz<iUi), а также градиент потенциальной 95
w Рис. 4.8 энергий "\U и соответствующий вектор силы F в Точке А поля. Полезно подумать, какими будут векторы этих двух величин, например, в точке В данного поля. В заключение заметим, что можно говорить о гради- енте не только функции U, но и любой другой скалярной функции координат. Понятие градиента широко исполь- зуется в самых различных разделах физики. Понятие поля. Опыт показывает, что в случае гравита- ционных и электростатических взаимодействий сила F, действующая на интересующую нас частицу со стороны окружаю- щих тел, пропорциональна массе (или заряду) частицы, причем си- ла F может быть представлена в виде произведения двух величин, например в случае тяготения F=mG, D.19) где т — масса частицы, G — неко- торый вектор, зависящий как от положения частицы, так и от свойств окружающих тел. Это открывает возможность иной физической интер- претации взаимодействия, связанной с понятием поля. А именно: говорят, что интересующая нас частица нахо- дится в поле, создаваемом окружающими ее телами и характеризуемом вектором G(r). Или, иначе, считают, что в каждой точке пространства вокруг этих тел (источ- ников поля) создаются гакие условия (вектор G), при ко- торых частица, помещенная в эти точки, испытывает дей- ствие силы D.19), причем считают, что поле, характери- зуемое G(r), существует безотносительно к тому, есть в нем частица или нет *. Вектор G называют напряженностью поля. Заметим, что напряженность электрического поля обо- значают вектором Е, а сила F, действующая на точечный заряд q в электростатическом поле, имеет вид, аналогич- ный D.19), т. е. F=<7E. Далее в этом параграфе почти всюду мы будем поль- зоваться величинами т и G, т. е. рассматривать гравита- * Пока мы остаемся в рамках статики, понятие поля может рас- сматриваться как чисто условное (формальное), введенное лишь для удобства описания явлений. Однако при переходе к переменным по- лям выясняется, что понятие поля имеет глубокий физический смысл; поле есть физическая реальность. 96
ционное поле. Чтобы получить соответствующие соотно- шения для электростатического поля, достаточно заме- нить в формулах т и G на q и Е. Одно из важнейших свойств полей заключается в том, что поле, образованное несколькими источниками, равно сумме полей, созданных каждым из них. Точнее, напря- женность G результирующего поля в произвольной точке где G; — напряженность поля t-ro источника в этой же точке. Эта формула выражает так называемый прин- цип суперпозиции (или наложения) полей. Обратимся к потенциальной энергии частицы. Соглас- но D.19), формулу D.14) можно записать так: /nGdr = = —сШ. Поделив обе части на т и обозначив отношение и/т через ф, получим Gdr= — dcp, D.21) или jGdr = ?!-%. D.22) i . Функцию ф(г) называют потенциалом поля в точ- ке с радиусом-вектором г. Формула D.22) дает возможность найти потенциал любого гравитационного и электростатического поля. Для этого достаточно вычислить интеграл / Gdr по произ- вольному пути между точками 1 и 2 и представить затем полученное выражение в виде убыли некоторой функции, которая и есть потенциал ф(г). Так, потенциалы гравита- ционного поля точечной массы т и кулоновского поля то- чечного заряда q определяются, согласно D.12), форму- лами ?гр = - Т т/г, Ткул = kq/r. D.23) Заметим, что потенциал ф, как и потенциальная энер- гия, может быть определен только с точностью до неко- торой произвольной постоянной, также совершенно не- существенной. Поэтому ее обычно опускают. Итак, поле можно описывать или в векторном виде G(r), или в скалярном ф(г). Оба способа адекватны. Практически же оказывается, что второй способ описа- ния поля (с помощью потенциала ф) в большинстве слу- чаев значительно удобнее, и вот почему. 07
1. Зная ф(г), можно немедленно вычислить потенци- альную энергию U и работу сил поля А: U=m<?, Ап=т («рх - <fe). D.24) 2. Вместо трех компонент векторной функции G(r) проще задавать скалярную функцию ср(г). 3. Когда поле создается многими источниками, потен- циал ф рассчитывать легче, чем вектор G: потенциалы — скаляры, их можно просто складывать, не заботясь о на- правлении сил. Действительно, согласно D.20) и D.21), Gdr= 2G,dr = —2СЧ>» = —с12ф< = —с1ф, т. е. , (г), D.25) где ф; — потенциал, создаваемый t-й частицей в данной точке поля. 4. И наконец, зная функцию ф(г), можно легко восста- новить поле G(r) —как G=-v?. D.26) Эта формула непосредственно следует из D.18). § 4.3. Механическая энергия частицы в поле Кинетическая энергия. Пусть частица массы т дви- жется под действием некоторой силы F (в общем случае сила F может быть результирующей нескольких сил). Найдем элементарную работу, которую совершает эта сила на элементарном перемещении dr. Имея в виду, что F=mdv/dt и dr = vd?, запишем Скалярное произведение vdv = u(dv)v, rAe(dv)T— проек- ция вектора dv на направление вектора v. Эта проекция равна do — приращению модуля вектора скорости. По- этому vdv = udu и элементарная работа ЬА=mv dv = d (mv2/2). Отсюда видно, что работа результирующей силы F идет на приращение некоторой величины (стоящей в скобках), которую называют кинетической энер- гией: D.27) 98
Таким образом, приращение кинетической энергии части- цы при элементарном перемещении равно <1Г=8Д D.28) а при конечном перемещении из точки / в точку 2 Т2 — Т1—л12) D.29) т. е. приращение кинетической энергии частицы на неко- тором перемещении равно алгебраической сумме работ всех сил, действующих на частицу на том же перемеще- нии. Если Л]2>0, то Т%>Т\, т. е. кинетическая энергия частицы увеличивается; если же <4i2<0, то кинетическая энергия уменьшается. Уравнения D.28) и D.29) справедливы в инерциаль- ных и неинерциальных системах отсчета. В последних кроме сил, действующих на рассматриваемую частицу со стороны каких-то тел (сил взаимодействия), необхо- димо учитывать и силы инерции. Поэтому под работой в этих уравнениях надо понимать алгебраическую сум- му работ как сил взаимодействия, так и сил инерции. Полная механическая энергия частицы. Согласно D.28), приращение кинетической энергии частицы рав- но элементарной работе результирующей F всех сил, действующих на частицу. Что это за силы? Если части- ца находится в интересующем нас стационарном поле консервативных сил, то на нее действует консервативная сила Fkohc со стороны этого поля. Кроме того, на части- цу могут действовать и другие силы, имеющие иное про- исхождение. Назовем их сторонними силами FCTop- Таким образом, результирующая F всех сил, дейст- вующих на частицу, может быть представлена в виде F = Fkohc+ FCTOp. Работа всех этих сил идет на прираще- ние кинетической энергии частицы: ^Т = Лконс -(- ЛС10р. Согласно D.10), работа сил поля равна убыли потенци- альной энергии частицы: ЛКонс =—At/. Подставив это выражение в предыдущее и перенеся величину AU влево, получим Отсюда видно, что работа сторонних сил идет на прира- щение величины T+U. Эту величину — сумму кинетичес- кой и потенциальной энергий — называют полной ме- 99
ханической энергией частицы в поле и обозначают Е: D.30) Заметим, что полная механическая энергия Е, как и по- тенциальная U, определяется с точностью до произволь- ной постоянной. Итак, из предыдущих двух уравнений следует, что приращение полной механической энергии частицы в стационарном поле консервативных сил при перемеще- нии ее из точки / в точку 2 можно записать в виде D.31) т. е. приращение полной механической энергии частицы на некотором пути равно алгебраической сумме работ всех сторонних сил, действующих на частицу на том же пути. Если ЛСТОр>0, то полная механическая энергия частицы увеличивается, если же Лстор<0, то уменьша- ется. Пример. Тело массы m бросили со скоростью Vo с обрыва высо- той h над поверхностью воды. Найдем работу, которую совершила сила сопротивления со стороны воздуха, при условии, что тело упало на поверхность воды со скоростью v. Если рассматривать движение тела в поле сил тяжести, то сила сопротивления со стороны воздуха будет сторонней и, согласно урав- нению D.31), искомая работа АСОпр = Е2—?i = mt>2/2—(mvo2/2 +mgh) или Интересно, что полученная величина может оказаться не только от- рицательной, но и положительной (это зависит, например, от харак- тера ветра в процессе падения тела). Итак, мы установили, что полная механическая энер- гия частицы может измениться под действием только сто- ронних сил. Отсюда непосредственно вытекает закон сохранения механической энергии части- ц ы: если сторонние силы отсутствуют или таковы, что не со- вершают работы в течение интересующего нас времени, то полная механическая энергия частицы в стационар- ном поле консервативных сил остается постоянной за это время, т. е. D.32) 100
•a о r\ Уже в такой простейшей форме закон сохранения энергии позволяет достаточно легко получать ответы на ряд важных вопросов без привлечения уравнений движе- ния, что часто сопряжено с проведением громоздких и утомительных расчетов. Именно это обстоятельство и превращает законы сохранения в весьма действенный ин- струмент исследования. Проиллюстрируем возможности и преимущества, ко- торые дает применение закона сохранения D.32), на следующем примере. Пример. Пусть частица дви- жется в одномерном стационарном поле, где ее потенциальная энергия U(x) имеет вид, как на рис. 4.9. Если сторонние силы отсутствуют, то полная механическая энергия частицы в данном поле, т. е. Е, не меняется в процессе движения и мы можем просто решить, напри- мер, такие вопросы, как: 1. Определить, не решая ос- новного уравнения динамики, ско- рость частицы в зависимости от ее координаты. Для этого достаточно знать, согласно уравнению D.32), конкретный вид потенциаль- ной кривой 0(х) и значение полной энергии Е. 2. Установить область изменения координаты х частицы, в ко- торой она может находиться при данном значении полной энергии Е. Ясно, что в область, где U>E, частица попасть не может, поскольку потенциальная энергия U частицы не должна превышать ее полную энергию. Отсюда сразу следует, что при Е=Е\ (рис. 4.9) частица будет двигаться или в области между координатами xt и х2 (совер- шает колебания) или правее координаты хз. Перейти же из первой области во вторую (или обратно) частица не может: этому препятст- вует потенциальный барьер, разделяющий обе эти области. Заметим, что когда частица движется в ограниченной области поля, то говорят, что она заперта в потенциальной яме (в нашем случае — между Х\ и х2). Иначе ведет себя частица при ? —?2 (рис. 4.9): для нее доступ- на вся область правее координаты ха. Если в начальный момент час- тица находилась в точке х0, то в дальнейшем она будет двигаться вправо. Полезно самостоятельно проследить, как будет меняться при этом кинетическая энергия частицы в зависимости от ее координа* ты х. Рис. 4.9 § 4.4. Потенциальная энергия системы До сих пор мы ограничивались рассмотрением пове- дения одной частицы с энергетической точки зрения. Те- перь перейдем к системе частиц. Это может быть любое тело, газ, какой-то механизм, Солнечная система и т. д. 101
Собственная потенциальная энергия системы. Рас- смотрим систему, между частицами которой действуют одни лишь центральные силы, т. е. силы, зависящие при данном характере взаимодействия только от расстояния между частицами и направленные по прямой, проходя- щей через эти частицы. Покажем, что независимо от системы отсчета работа всех этих внутренних сил при переходе системы частиц из одного положения в другое может быть представлена как убыль некоторой функции, зависящей при данном ха- рактере взаимодействия только от относительного рас- положения частиц системы, т. е. от ее конфигурации. Эту функцию называют собственной потенциаль- ной энергией системы (в отличие от внешней потен- циальной энергии, характеризующей взаимодействие данной системы с другими телами). Сначала возьмем систему из двух частиц 1 и 2. Оп- ределим алгебраическую сумму элементарных работ сил Fj и F2, с которыми эти частицы взаимодействуют. Пусть в произвольной /(-системе отсчета за время dt частицы совершили перемещения dn и dr2. Тогда соответствую- щая сумма работ этих сил Учитывая, что F2 =—Fi (согласно третьему закону Нью- тона), перепишем предыдущее уравнение так: SA,2=Fi(dri-dr2). Величина, стоящая в скобках, представляет собой не что иное, как перемещение частицы / относительно частицы 2, точнее, перемещение частицы / в /('-системе отсчета, жестко связанной с частицей 2 и перемещающейся вмес- те с ней поступательно относительно исходной /(-систе- мы отсчета. Действительно, перемещение dri частицы / в /(-системе отсчета может быть представлено как пере- мещение dr2 /('-системы отсчета (связанной с частицей 2) плюс перемещение dr/ частицы / относительно этой К.'- dr,=d Р системы, т. е. drj = dr2+dri". Отсюда dri—dr2=< bAh2=F1ur'l. Полученный таким образом результат весьма замеча- телен: алгебраическая сумма элементарных работ пары сил взаимодействия в произвольной /(-системе отсчета оказывается всегда равной элементарной работе, кото- 102
рую совершает сила, действующая на одну частицу, в системе отсчета, где другая частица покоится. Иначе го- воря, работа 6^1,2 не зависит от выбора исходной К-сн- стемы отсчета. Сила Fb действующая на частицу / со стороны час- тицы 2, является центральной, а значит и консервативной. Поэтому работа данной силы на перемещении dn' мо- жет быть представлена, согласно D.10), как убыль по- тенциальной энергии частицы / в поле частицы 2 или как убыль потенциальной энергии взаимодействия рас- сматриваемой пары частиц: где U\2 — функция, зависящая только от расстояния ме- жду этими частицами. При конечном же перемещении Рассмотрим теперь систему из трех частиц. (Получен- ный в этом случае результат легко обобщить и на сис- тему из произвольного числа частиц.) Работа, которую совершают все силы взаимодействия при перемещении всех частиц, может быть представлена как алгебраичес- кая сумма работ всех трех пар сил взаимодействий, т. е. ^=^1,2+^1,3+^2,3- Но для каждой пары этих сил, как только что было показано, Aih = —AUik, поэтому где функция 1/соб — собственная потенциаль- ная энергия данной системы частиц: Так как каждое слагаемое этой суммы зависит от рас- стояния между соответствующими частицами, то очевид- но, что собственная потенциальная энергия данной сис- темы зависит от относительного расположения частиц (в один и тот же момент), или, другими словами, от кон- фигурации системы. Ясно, что подобные рассуждения справедливы и для системы из любого числа частиц. Поэтому можно ут- верждать, что каждой конфигурации системы частиц присуще свое значение собственной потенциальной энер- гии и работа всех внутренних центральных (консерва- тивных) сил при изменении этой конфигурации равна убыли собственной потенциальной энергии системы: 103
— ^1соб ^гсоб — D.33) где f/ico6 и U2co6 — значения собственной потенциальной энергии системы в начальном и конечном состояниях. Мы видим, таким образом, что суммарная работа вну- тренних центральных сил не зависит от того, как конк- ретно система переходит от конфигурации 1 к конфигу- рации 2. Данная работа определяется исключительно са- мими конфигурациями системы. Все это позволяет дать более общее определение консервативных сил: консерва- тивными называют силы, зависящие только от конфигу- рации системы и суммарная работа которых не зависит от «пути» перехода, а определяется только начальной и конечной конфигурациями системы. Собственная потенциальная энергия системы — вели- чина не аддитивная, т. е. она не равна в общем случае сумме собственных потенциальных энергий ее частей. Не- обходимо учесть еще потенциальную энергию взаимодей- ствия UB3 отдельных частей системы: ^соб=У^ + ^вз, D.34) где Un — собственная потенциальная энергия л-й части системы. Следует также иметь в виду, что собственная потен- циальная энергия системы, как и потенциальная энергия взаимодействия каждой пары частиц, определяется с то- чностью до произвольной постоянной. В заключение приведем полезные формулы для расчета собст- венной потенциальной энергии системы Прежде всего покажем, что эта энергия может быть представлена как D 35) где U, — потенциальная энергия взаимодействия i-й частицы со все- ми остальными частицами системы. Здесь сумма берется по всем частицам системы. Убедимся в справедливости этой формулы сначала для системы из трех частиц. Выше было показано, что собственная потенциальная энергия данной системы Uco5 = Ui2 + Ui3 + U23. Преобразуем эту сум* му следующим образом Представим каждое слагаемое Иги в сим- метричном виде: игк={игк + икгI2, ибо ясно, что игк = икг. Тогда Uco6 = — {1Гп + U2X + СА13 + Uzl + СГ23 + Сгруппируем члены с одинаковым первым индексом: 104
?Л:об = — [(^12 + Uo) + {U2y + U2i) + (С/ц + f/32)] • Каждая сумма в круглых скобках представляет собой потенциаль- ную энергию Ut взаимодействия i-й частицы с остальными двумя. Поэтому последнее выражение можно переписать так: з ^соб = Y {Ui + U2 + С/3) = y ' что полностью соответствует формуле D.35). Обобщение полученного результата на произвольную систему очевидно, ибо ясно, что подобные рассуждения совершенно не зави- сят от числа частиц, составляющих систему. Для системы, взаимодействие между частицами которой носит гравитационный или кулоновский характер, формулу D.35) можно преобразовать и к другому виду, воспользовавшись понятием потен- циала. Заменим в D.35) потенциальную энергию i-й частицы выра- жением Ui = nii<pi, где я; — масса (заряд) j-й частицы, а <р*—-по- тенциал, создаваемый всеми остальными частицами системы в точка нахождения i-й частицы. Тогда Если массы (заряды) распределены в системе непрерывно, то сум- мирование сводится к интегрированию: {<??dV, D.37) где р — объемная плотность массы (заряда), dV — элемент объема. Здесь интегрирование проводится по всему объему, занимаемому массами (зарядами). «Внешняя» потенциальная энергия системы. Рассмот- рим случай, когда система находится во внешнем ста- ционарном поле консервативных сил. В этом случае ка- ждая частица системы будет характеризоваться своим значением потенциальной энергии Ui в данном поле, а вся система — величиной Эту величину мы и будем называть «внешней» по- тенциальной энергией системы в отличие от ?Л?об — собственной потенциальной энергии, зависящей только от взаимодействия частиц системы между собой. Согласно D.10), убыль потенциальной энергии каж- дой частицы во внешнем поле равна работе силы данно- го поля на соответствующем перемещении, поэтому убыль 105
всей системы равна АШ1ет — алгебраической сум- ме работ всех сил внешнего поля, действующих на все частицы системы: л ¦ внеш D.39) Получим полезную формулу для вычисления внеш- ней потенциальной энергии системы, находящейся в од- нородном силовом поле. Пусть, например, это будет поле тяжести, где на i'-ю частицу системы дейст- вует сила rtiig. В этом случае потенциальная энергия данной частицы, согласно D.13), есть niigZi, где z\—¦ вертикальная координата частицы, отсчитанная от не- которого произвольного уровня О. Тогда потенциальная энергия всей системы во внешнем однородном поле (соб- ственная потенциальная энергия нас сейчас не интере- сует) может быть записана так: Сумма, стоящая в скобках, в соответствии с C.8) есть не что иное, как произведение массы т всей системы на вертикальную координату zc центра масс данной систе- мы, т. e.^niiZi^mzc. Поэтому выражение для ?/ВНеш можно переписать в окончательном виде: D.40) т. е. потенциальная энергия системы во внешнем одно- родном поле тяжести равна произведению массы т си- стемы на g и на вертикальную координату Zc ее центра масс. Приращение величины ?/Внеш при перемещении систе- мы определяется формулой D.41) где Azc — приращение вертикальной координаты цент- ра масс данной системы частиц. § 4.5. Закон сохранения механической энергии системы Диссипативные силы. Помимо разделения всех сил на внешние и внутренние (в зависимости от выбора си- стемы частиц), силы, как мы уже знаем, принято подраз- делять на консервативные и неконсерва- тивные (в зависимости от их природы). 106
К неконсервативным силам относятся так называемые диссипативные силы — это силы трения и сопро- тивления. Любая диссипативная сила может быть пред- ставлена в виде F= — k(v)v, D.42) где v — скорость данного тела относительно другого тела (или среды), с которым оно взаимодействует; k(v) — по- ложительный коэффициент, зависящий в общем случае от скорости v. Сила F всегда направлена противополож- но вектору v. В зависимости от выбора системы отсчета работа этой силы может быть как положительной, так и отрица- тельной. Однако, как мы сейчас покажем и что будет ва- жно для дальнейшего, суммарная работа всех внутрен- них диссипативных сил в системе —• величина всегда отри- цательная независимо от системы отсчета: внугр 0. D.43) Переходя к доказательству, отметим прежде всего, что внутренние диссипативные силы в данной системе бу- дут встречаться попарно, причем в каждой паре, соглас- но третьему закону Ньютона, они одинаковы по модулю и противоположны по направлению. Найдем элементар- ную работу произвольной пары диссипативных сил вза- имодействия между телами / и 2 в системе отсчета, где скорости этих тел в данный момент равны vi и v2: Учтем, что F2—Fb vi—V2=v — скорость тела / отно- сительно тела 2, а также то, что Fi = —kv. Тогда выра- жение для работы преобразуется так: 8ЛДИС = ?х (vx — v2) & = — k vv dt = — Ы- dt. Отсюда видно, что работа произвольной пары внутрен- них диссипативных сил взаимодействия всегда отрица- тельна, а значит, и суммарная работа всех пар внутрен- них диссипативных сил также всегда отрицательна, при- чем в любой системе отсчета. Кинетическая энергия системы. Согласно D.28), при- ращение кинетической энергии каждой частицы равно работе всех сил, действующих на частицу: ДГ,=Л,. По- этому работу А, которую совершают все силы, действую- щие на все частицы системы, при изменении ее состояния, 107
можно записать так: А= 2^i = SA^i = A ZiTt, или А = &Т, T=^Th D.44) где Г—суммарная кинетическая энергия системы. Итак, приращение кинетической энергии системы рав- но работе, которую совершают все силы, действующие на в се частицы системы: Тг-Т1=А. D.45) Заметим попутно, что кинетическая энергия системы — величина аддитивная: она равна сумме кинетических энергий отдельных частей системы независимо от того, взаимодействуют они между собой или нет. Уравнение D.45) справедливо как в инерциальных, так и в неинерциальных системах отсчета. Следует толь- ко помнить, что в неинерциальных системах отсчета кро- ме работ сил взаимодействия необходимо учитывать и работу сил инерции. Полная механическая энергия системы. Только что было показано, что приращение AT кинетической энер- гии системы равно работе, которую совершают все си- лы, действующие на все частицы системы. Разделим эти силы на внутренние и внешние, а внутренние, в свою очередь, — на консервативные и диссипативные. Тогда предыдущее утверждение можно записать так: А'Г=А 4- А —дконс 1/1 дис _|_ А "¦' -^вну гр Г ^внеш — внутр I внутр • -^внеш1 Учтем, что работа внутренних консервативных сил рав- на, согласно D.33), убыли собственной потенциальной энергии системы: Лвк™р =—AUC06. Тогда предыдущее выражение примет вид 5) = ЛД„% + Лнеш- D.46) Введем понятие полной механической эне- ргии системы, или, короче, механической энергии как сумму кинетической и собственной по- тенциальной энергии системы: ¦=1 "Г^сов D.47) Очевидно, энергия Е зависит от скоростей частиц сис- темы, характера взаимодействия между ними и конфигу- 108
рации системы. Кроме того, энергия Е, как и потенци- альная энергия f/соб. определяется с точностью до при- бавления несущественной произвольной постоянной и является величиной неаддитивной, т. е. энергия Е систе- мы йе равна в общем случае сумме энергий ее отдель- ных частей. В соответствии с D.47) D.48) где Еп — механическая энергия п-й части системы, ?/вз—¦ потенциальная энергия взаимодействия ее отдельных ча- стей. Вернемся к уравнению D.46). Перепишем его с уче- том D.47) в виде D.49) — приращение механической энергии системы равно ал- гебраической сумме работ всех внутренних диссипатив- ных сил и всех внешних сил. Уравнение D.49) справедливо как в инерциальной, так и в неинерциальной системах отсчета. Следует толь- ко иметь в виду, что в неинерциальной системе отсчета необходимо учитывать и работу сил инерции, играющих роль внешних сил, т. е. под Лвпеш надо понимать алгебра- ическую сумму работ внешних сил взаимодействия и ра- боту сил инерции. Закон сохранения механической энергии. Этот закон непосредственно вытекает из последнего уравнения и формулируется так: механическая энергия замкнутой системы частиц, в которой нет диссипативных сил, сохраняется в процес- се движения, т. е. D.50) Такую систему называют консервативной*. Заме- тим, что при движении замкнутой консервативной си- стемы сохраняется именно полная механическая энергия, кинетическая же и потенциальная в общем случае изме- няются. Однако эти изменения происходят всегда так, что приращение одной из них в точности равно убыли * С достаточно хорошим приближением замкнутой консерватив- ной системой можно считать Солнечную систему. 109
другой, т. е. АТ=—Д?/Соб. Это положение справедливо только в инерциальных системах отсчета. Далее, из уравнения D.49) следует, что если замк- нутая система не консервативна, т. е. в ней имеются дис- сипативные силы, то механическая энергия такой систе- мы, согласно D.43), убывает: Можно сказать: уменьшение механической энергии обус- ловлено тем, что она расходуется на работу против дис- сипативных сил, действующих в системе. Однако такое объяснение является формальным, поскольку оно не рас- крывает физической природы диссипативных сил. Более глубокое осмысливание этого вопроса привело к фундаментальному выводу о существовании в природе универсального закона сохранения энер- гии: энергия никогда не создается и не уничтожается, она мо- жет только переходить из одной формы в другую или об- мениваться между отдельными частями материи. При этом понятие энергии пришлось расширить введением понятий о новых формах ее (помимо механической) —• энергия электромагнитного поля, химическая энергия, ядерная и др. Универсальный закон сохранения энергии охватыва- ет, таким образом, и те физические явления, на которые законы Ньютона не распространяются. Поэтому он не может быть выведен из этих законов, а должен рассмат- риваться как самостоятельный закон, представляющий собой одно из наиболее широких обобщений опытных фактов. Возвращаясь к уравнению D.51), можно сказать: при уменьшении механической энергии замкнутой системы всегда возникает эквивалентное количество энергии дру- гих видов, не связанных с видимым движением. В этом смысле уравнение D.49) можно рассматривать как более общую формулировку закона сохранения энергии, в которой указана причина изменения механической энер- гии у незамкнутой системы. В частности, механическая энергия может сохранять- ся у незамкнутых систем, но это происходит лишь в тех случаях, когда, согласно уравнению D.49), уменьшение этой энергии за счет работы против внутренних диссипа- тивных сил компенсируется поступлением энергии за счет работы внешних сил. ПО
Механическая энергия системы во внешнем поле. Если интересу- ющая нас система частиц находится во внешнем стационарном пола консервативных сил, то часто бывает удобно пользоваться другим выражением для полной механической энергии Е этой системы, от- личным от D.47). В данном случае внешние силы, действующие на частицы систе- мы, можно разделить на силы со стороны внешнего поля (внешние силы поля) и все остальные внешние силы, не относящиеся к данному внешнему полю (внешние сторонние силы). Со- ответственно работа Лвнеш внешних сил может быть представлена как алгебраическая сумма работ внешних сил поля и внешних сто- ронних сил: Л — Лиоля а- /3СТ°Р ¦^внещ — -^внеш "+" -^внеш • Но работа внешних сил поля, в свою очередь, может быть представ- лена как убыль внешней потенциальной энергии, а именно А ™еш == = —Д1/Внсш. Тогда -^ннеш := А^внещ -(- -^внеш " Подставив последнее выражение в формулу D.46), получим Д (Т + UQ06 + ^внеш) = А%*ш + А™ . D.52) Величину, стоящую слева в скобках, называют полной м е х а н и- ческой энергией Е' системы во внешнем стацио- нарном поле консервативных сил: D.53) В отличие от выражения D.47) эта полная механическая энергия включает в себя помимо суммарной кинетической и собственной по< тенциальной энергии еще и потенциальную энергию системы во внеш< нем поле ивпеш. С учетом D.53) уравнение D.52) можно переписать так: '2 С1 — ^внеш "г ¦ 1ДНС 'внутр D.54) Из этого уравнения вытекает закон сохранения полной механической энергии системы, находящейся во внеш- нем стационарном поле консервативных сил: если на интересующую нас систему частиц не действуют внешние сторонние силы и нет внутренних диссипативных сил, то полная ме- ханическая энергия такой системы остается постоянной: D.55) Е' = Т + Uco6 + ивнеш = const. Простейшим примером подобной системы могут служить два небольших тела, соединенные друг с другом пружинкой. Если эта система движется в поле тяжести в отсутствие сопротивления воз- духа (т. е. нет внешних сторонних сил), то меняются ее кинетическая энергия Т, собственная потенциальная энергия (/СОб и внешняя по- 111
тенциальная энергия ?/Внеш, однако алгебраическая сумма этих трех величин будет оставаться постоянной. Другой пример — это система Земля — Луна в поле тяготения Солнца. В процессе движения этой системы также меняются Т, Исая и l/внсш, но их алгебраическая сумма сохраняется неизменной. В заключение остается отметить, что уравнение D.54) выполня- ется как в инерциальной, так и в неинерциальной системах отсчета, закон же сохранения механической энергии D.55) — только в инер- циальной. Связь между энергиями в К- и /^-системах отсчета. Прежде всего установим эту связь для кинетических энергий системы. Пусть в /(-системе отсчета кинетичес- кая энергия интересующей нас системы частиц равна Т. Скорость 1-й частицы можно представить как Vi=Vj+Vc, где V; — скорость этой частицы в //-системе, a Vc — ско- рость //-системы относительно /(-системы отсчета. Тог- да для кинетической энергии Т системы можно записать гс,-(у,-+ УсJ _ 2 Так как в //-системе центр масс покоится, значит, согла- сно C.9), 2m,-v/=0 и предыдущее выражение примет вид D.56) где T = ll2^im^i2 — суммарная кинетическая энергия час- тиц в //-системе, m — масса всей системы. Таким образом, кинетическая энергия системы частиц складывается из суммарной кинетической энергии Т в Ц-системе и кинетической энергии, связанной с движе- нием системы частиц как целого. Это важный вывод, и он неоднократно будет использоваться в дальнейшем (в частности, при изучении динамики твердого тела). Из формулы D.56) следует, что кинетическая энер- гия системы частиц минимальна в Ц-системе — в этом еще одна особенность //-системы. Действительно, в Ц- системе Vc = 0, поэтому в D.56) остается только Т. Перейдем к полной механической энергии Е системы. Так как собственная потенциальная энергия системы t/соб зависит только от конфигурации системы, то зна- чение ?/соб одинаково во всех системах отсчета. Добавив исоб в левую и правую части равенства D.56), получим 112
формулу преобразования полной механической энергии Е при переходе от К- к Д-системе: mVl, D.57) где E = T + L\o6. Эту энергию называют внутренней механической энергией системы. Пример. На гладкой горизонтальной плоскости лежат две не- большие шайбы, каждая массы т, которые соединены между собой невесомой пружинкой. Одной из шайб сообщили начальную скорость vo, как показано на рис. 4.10 (вид сверху). Най- дем внутреннюю механическую энергию Е этой системы в процессе движения. Поскольку плоскость гладкая, система в про- цессе движения будет вести себя как замкнутая. Поэтому ее полная механическая энергия Е и ско- рость Vc центра масс будут сохраняться, оста- ваясь равными тем значениям, которые они име- ли в начальный момент: ? = т.ио2/2 и Vc = u0/2. Подставив эти значения в формулу D.57), полу- чим ' = ? — Рис. 4.10 = 1/4 mvr где учтено, что масса системы равна 2т. Нетруд- но сообразить, что внутренняя энергия Е связана с вращением и колебанием данной системы, при- чем в начальный момент Е была равна только энергии вращательно- го движения. Если система частиц замкнута и в ней происходят процессы, связанные с изменением полной механической энергии, то из D.57) следует, что АЕ = АЕ, т. е. прира- щение полной механической энергии относительно про- извольной инерциальной системы отсчета равно прира- щению внутренней механической энергии. При этом кинетическая энергия, обусловленная движением систе- мы частиц как целого, не меняется, ибо для замкнутой системы Vc=const. В частности, если замкнутая система консерва- тивна, то ее полная механическая энергия сохраняется во всех инерциальных системах отсчета. Этот вывод на- ходится в полном соответствии с принципом относитель- ности Галилея. § 4.6. Столкновение двух частиц Предварительные сведения. В этом параграфе мы рассмотрим различные случаи столкновения двух частиц, 113
используя в качестве инструмента исследования только законы сохранения импульса и энергии. При этом мы уви- дим, что законы сохранения позволяют сделать ряд ве- сьма общих и существенных заключений о свойствах дан- ного процесса вне какой-либо зависимости от конкретно- го закона взаимодействия частиц. Попутно покажем, какие преимущества дает Д-систе- ма, использование которой, как будет видно, значитель- но упрощает анализ процесса и многие расчеты. Хотя мы будем говорить о столкновении частиц, необходимо сразу же оговорить, что все последующие рассуждения и выводы в равной степени относятся и к столкновению любых тел. Надо только иметь в ви- ду, что вместо скорости частицы следует брать скорость центра масс каждого тела, а вместо кинетической энер- гии частицы — ту часть кинетической энергии каждого тела, которая характеризует его движение как целого. Прежде чем переходить к рассмотрению теории стол- кновений, приведем несколько важных и полезных со- отношений для системы из двух частиц в ее Д-системе отсчета. Ранее (в конце §3.4) были получены выражения C.12) для импульса каждой частицы в Д-системе. За- пишем эти выражения в такой форме: Pi=f*(Vi-v2)," pa=!*(va —vj, D.58) где vi и V2 — скорости частиц в исходной системе отсче- та, \х — так называемая приведенная масса си- стемы, /TJj OT2 D.59) ni\ и т2 — массы частиц. Из формул D.58) видно, что импульсы обеих частиц в Д-системе одинаковы по модулю и противоположны по направлению, причем модуль импульса каждой час- тицы можно записать как D.60) здесь yOTH=|vi—V21 —скорость одной частицы «относи- тельно другой». Теперь обратимся к кинетической энергии. Суммар- ная кинетическая энергия обеих частиц в Д-системе 114
т„ Так как, согласно D.59), l/mi + l/m2= 1/ц, то выражение для Т примет следующий вид: М-^отн D.61) Если частицы взаимодействуют друг с другом, то пол- ная механическая энергия частиц в Д-системе где U — потенциальная энергия взаимодействия данных частиц. В дальнейшем при рассмотрении столкновений частиц будем считать: 1) исходная .К-система отсчета инерциальная, 2) система из двух частиц замкнутая, 3) импульсы (и скорости) частиц до и после столкно- вения соответствуют достаточно большим расстояниям между ними; при этом потенциальной энергией взаимо- действия можно просто пренебречь. Кроме того, величины, относящиеся к системе после столкновения, будем отмечать штрихом, а величины в Ц- системе — значком ~ (тильда) сверху. Перейдем к существу вопроса. Различают три типа столкновения частиц: абсолютно неупругое, абсолютно упругое и промежуточный случай — неупругое. Абсолютно неупругое столкновение. Это такое столк- новение, в результате которого обе частицы «слипают- ся» и далее движутся как единое целое. Пусть две час- тицы, массы которых ту и т2, имеют до столкновения скорости vi и v2 (в i^-системе). После столкновения об- разуется частица с массой m.i + m2, что прямо следует из аддитивности массы в ньютоновской механике. Скорость v' образовавшейся частицы можно найти сразу из зако- на сохранения импульса: (щ -(- щ) v' =m1\1-\- тг v2. Ясно, что скорость v' равна скорости центра масс систе- мы. В //-системе этот процесс выглядит наиболее просто: до столкновения обе частицы движутся навстречу друг другу с одинаковыми импульсами р, а после столкнове- ния образовавшаяся частица оказывается неподвижной. 115
При этом суммарная кинетическая энергия Т частиц це- ликом переходит во внутреннюю энергию Q образовав- шейся частицы, т. е. T = Q. Отсюда с учетом формулы D.61) найдем j^4h_=J M?_(v V2J. D.62) 2 2 mi + tu2 Таким образом, величина Q для данной пары частиц за- висит только от их относительной скорости. Абсолютно упругое столкновение. Это такое столкно- вение, в результате которого внутренняя энергия частиц не меняется, а поэтому не меняется и кинетическая энер- гия системы. Рассмотрим два частных случая; лобовое и нелобовое упругие р, рг - * столкновения. *- ^* "° 1. Лобовое стол- -"*—~т ji *~ жле кновение — обе час- тицы до и после столк- Рис 4.11 новения движутся по од- ной и той же прямой. Пусть до столкновения скорости частиц в Д'-системе от- счета равны vi и v2 (частицы движутся или навстречу друг другу, или одна частица догоняет другую). Каковы скорости этих частиц после столкновения? Рассмотрим этот процесс сначала в Д-системе, где до и после столкновения обе частицы имеют одинаковые по модулю и противоположные по направлению импульсы (рис. 4.11). Более того, так как суммарная кинетическая энергия частиц до и после столкновения одинакова, как и их приведенная масса, то, согласно D.61), импульс каждой частицы в результате столкновения изменит то- лько направление на противоположное, не меняясь при этом по модулю, т. е. р/=—р,-, где i=\, 2. Последнее от- носится и к скорости каждой частицы в Д-системе: Теперь найдем скорость каждой частицы после столкно- вения в i^-системе отсчета. Для этого используем форму- лы преобразования скоростей при переходе от Ц- к К- системе, а также предыдущее равенство. Тогда где Vc — скорость центра масс (Д-системы) в .К-системе отсчета; эта,скорость определяется формулой C.9). Итак, 116
скорость i'-й частицы в Л*-системе после столкновения есть v'c=2Vc-v,, D.63) где 1=1, 2. В проекциях на произвольную ось х это ра- венство имеет вид ¦v'lx._=2VCx-vix. D.64) В частности, если массы частиц одинаковы, то легко убе- диться, что частицы в результате столкновения просто обмениваются скоростями, т. е. vi' = V2 и \2—\\. т,- Рг -mz Рис. 4.12 О р, Рис. 4.13 2. Нелобовое столкновение. Ограничимся случаем, когда одна из частиц покоится до столкновения. Пусть в i^-системе отсчета частица массы гп\ с импуль- сом pi испытала упругое нелобовое столкновение с по- коившейся частицей массы т2. Каковы возможные им- пульсы этих частиц после столкновения? Рассмотрим этот процесс также сначала в Д-системе. Здесь, как и в предыдущем случае, обе частицы в любой момент времени до и после столкновения имеют одина- ковые по модулю и противоположные по направлению импульсы. Кроме того, импульс каждой частицы не изме- нится по модулю в результате столкновения, т. е. Однако направление разлета частиц теперь будет иным. Оно будет составлять с первоначальным направлением движения некоторый угол 0 (рис. 4.12), зависящий от за- кона взаимодействия частиц и их взаимного расположе- ния в процессе столкновения. Найдем импульс каждой частицы в i^-системе отсче- та после столкновения. С помощью формул преобразо- 117
вания скоростей при переходе от Ц- к /(-системе полу- чим: '2, где Vc — скорость Д-системы относительно ^-системы отсчета. Сложив отдельно левые и правые части этих равенств с учетом того, что pi' = —рг', получим как и должно быть в соответствии с законом сохранения импульса. Построим теперь так называемую векторную ди- аграмму импульсов. Сначала изобразим вектор pi отрезком АВ (рис. 4.13), затем векторы р/ и рг', ка- ждый из которых представляет собой, согласно D.65), сумму двух векторов. Заметим, что это построение справедливо вне зави- симости от угла ¦п. Отсюда следует, что точка С (рис. 4.13) может находиться только на окружности радиуса р с центром в точке О, которая делит отрезок АВ на две части в отношении АО: ОВ = гп\:т2. Более того, в рас- сматриваемом случае (частица массы т2 покоится до столкновения) эта окружность проходит через точку В — конец вектора рь ибо отрезок ОВ—р. Действительно, • ТП-2 где V\ — скорость налетающей частицы. А так как в на- шем случае Vi = vOtn, то, согласно D.59) и D.60), Таким образом, для построения векторной диаграммы импульсов, соответствующей упругому столкновению двух частиц (одна из которых первоначально покоилась) необходимо: 1) изобразить отрезок АВ, равный импульсу р\ на- летающей частицы; 2) через точку В — конец вектора pi — провести ок- ружность радиуса 118
центр которой — точка О — делит отрезок АВ на две ча- сти в отношении АО : ОВ — т\: т2. Эта окружность есть геометрическое место точек всех возможных положений вершины С треугольника им- пульсов ABC, стороны АС и СВ которого и представля- ют собой возможные импульсы частиц после столкнове- ния (в i^-системе отсчета). В зависимости от соотношения масс частиц точка А— начало вектора pi — может находиться внутри данной окружности, на ней или снаружи (рис. 4.14, а, б, в). При этом во всех трех случаях угол й может принимать все " - ч - . «' Рис. 4.14 значения от 0 до я. Возможные же значения угла рассе- яния налетающей частицы ¦&{ и угла разлета частиц 0 будут такими: а) б) т1=т2 е>я/2 в) /п1>тя2 0<&1<91макс 0<я/2 Здесь ¦&! макс — предельный угол. Он определя- ется формулой s'm^lMaKC = m2/m1, D.67) которая непосредственно следует из рис. 4.14, в: sin $1ткс=ОС'/АО =ОВ/АО = т2/т1. Кроме того, обнаруживается еще один интересный факт. В последнем случае (mi>m2) под одним и тем же углом "&i возможно рассеяние частицы mi как с импуль- сом АС, так и с импульсом AD (рис. 4.14, в), т. е. в этом случае решение неоднозначно. Аналогично обстоит дело и с частицей т2. И наконец, из той же векторной диаграммы импуль- сов можно найти связь между углами Oi и fh 119
Этим исчерпываются сведения, которые можно полу- чить о данном процессе, исходя из одних только зако- нов сохранения импульса и энергии. Мы видим, таким образом, что уже сами по себе за- коны сохранения импульса- и энергии действительно поз- воляют сделать ряд важных заключений о свойствах рас- сматриваемого процесса. При этом особенно существен тот факт, что эти свойства имеют общий характер, т. е. совершенно не зависят от рода взаимодействия частиц. Следует, однако, обратить внимание на одно прин- ципиальное обстоятельство. Векторная диаграмма им- пульсов, в основе которой лежат законы сохранения им- пульса и энергии, давая нам полную картину всех воз- можных случаев разлета частиц после столкновения — результат сам по себе весьма существенный, — совершен- но не говорит о том, какой из этих возможных случаев реализуется конкретно. Для установления этого необхо- димо обратиться к более детальному рассмотрению про- цесса столкновения с помощью уравнений движения. При этом выясняется, например, что угол рассеяния di налетающей частицы зависит от характера взаимодей- ствия сталкивающихся частиц и от так называемого прицельного параметра*, неоднозначность же решения в случае mi>m2 объясняется тем, что один и тот же угол рассеяния #1 может реализоваться при двух значениях прицельного параметра, причем независимо от закона взаимодействия частиц. Указанное обстоятельство представляет собой очень характерную и принципиальную черту законов сохране- ния вообще. Законы сохранения никогда не дают и не могут дать однозначного ответа на вопрос о том, что произойдет. Но если, исходя из каких-либо других сооб- ражений, можно указать, что именно должно произойти, го законы сохранения дают ответ на вопрос, как это должно произойти. Неупругое столкновение. Это такое столкновение, в результате которого внутренняя энергия разлетающихся частиц (или одной из них) изменяется, а следовательно изменяется и суммарная кинети- ческая энергия системы. Соответствующее приращение кинетической энергии системы принято обозначать через Q. В зависимости от зна- ка Q неупругое столкновение называют экзоэнергетическим (Q>0) или эндоэнергетическим (Q<0). В первом случае * Прицельный параметр — это расстояние между пря- мой, вдоль которой направлен импульс налетающей частицы, и час- тицей, с которой происходит «столкновение». 120
кинетическая энергия системы увеличивается, во втором — умень- шается. При упругом столкновении, разумеется, Q = 0. Наша задача: найти возможные импульсы частиц после неупру- гого столкновения. Этот вопрос наиболее просто решается в Д-системе. Согласно условию, приращение суммарной кинетической энергии системы в данном процессе f'—T=Q. D.68) Так как в данном случае Т'фТ, то это означает, согласно D.61), что импульсы частиц после столкновения изменятся по модулю. Кмпульс каждой частицы после столкновения р' легко найти, заме- нив Т в D.68) его выражением Т = р'2/2ц. В результате получим У = V2[х (? + (?) • D.69) Теперь рассмотрим тот же вопрос в К-системе отсчета, где час- тица массы nti с импульсом pi ис- пытывает столкновение с покоя* щейся частицей массы пг2. Для определения возможных случаев разлета частиц после столкнове- ния здесь также полезно восполь- зоваться векторной диаграммой импульсов. Ее построение анало- гично тому, как это было сделано для упругого столкновения. Им- пульс налетающей частицы pi= Рис- 4.15 =АВ (рис. 4.15) делят точкой О на две части, пропорциональные массам частиц (АО : ОВ = т.\: яг2). Затем из точки О проводят окружность радиуса р', определяемого формулой D.69). Эта окружность и является геометрическим местом точек всех возможных положений вершины С треугольника импуль- сов ABC, стороны АС и СВ которого равны импульсам соответству- ющих частиц после столкновения. Отметим, что теперь в отличие от упругого столкновения точка В — конец вектора pi — не лежит на окружности, а именно: при Q>0 эта точка находится внутри окружности, а при Q<0 — вне ее. Рис. 4.15 соответствует последнему случаю — эндоэнергетическому столкновению. Порог. Существует много неупругих столкновений, в которых внутренняя энергия частиц способна изменяться только на совершенно определенную величину, зависящую от свойств самих частиц (тако- вы, например, неупругие столкновения атомов и молекул). Несмотря на это, экзоэнергетические столкновения (Q>0) могут происходить при сколь угодно малой кинетической энергии налетающей частицы. Эндоэнергетические же процессы (Q<0) в таких случаях обладают порогом. Порогом называют минимальную кинетическую энергию налетающей частицы, начиная с которой данный процесс становится энергетически возможным. Итак, пусть нам необходимо осуществить такое эндоэнергетиче- ское столкновение, в котором внутренняя энергия частиц способна получить приращение не меньше некоторого значения \Q\. При ка- ком условии такой процесс окажется возможным? Этот вопрос наиболее просто решается также в //-системе, где ясно, что суммарная кинетическая энергия Г частиц до столкновения 121
Рассмотрим во всяком случае должна быть не меньше \Q\, т. е. f^|Q|. Отсюда следует, что существует минимальное значение TMXn=\Q\, при ко- тором кинетическая энергия системы целиком пойдет на увеличение внутренней энергии частиц, и частицы после столкновения остановят- ся в Д-системе. этот же вопрос в К-системе отсчета, где частица массы гп\ налетает на по- коящуюся частицу массы т2. Так как в /(-системе при Тмин частицы после столк- новения останавливаются, то это значит, что в К-системе при соответствующей пороговой ки- I нетической энергии 7\Пор нале- тающей частицы обе частицы после столкновения будут дви- гаться как единое целое, при- чем с суммарным импульсом, равным импульсу pi налетаю- щей частицы, и кинетической энергией p12/2(m1+m2). По- этому А так как получим Рис. 4.16 то, исключив pi2 из этих двух уравнений, * 1пор —' D.70) Это и есть та пороговая кинетическая энергия налетающей частицы, начиная с которой данный эндоэнергетический процесс становится энергетически возможным. Заметим, что формула D.70) играет большую роль особенно в атомной и ядерной физике. С помощью ее определяют как порог различных эндоэнергетических процессов, так и соответствующее им значение энергии Щ\. Задачи ф4.1. Работа и мощность. Камень массы m бросили с поверхно- сти Земли под углом а к горизонту с начальной скоростью Vo. Пре- небрегая сопротивлением воздуха, найти мощность силы тяжести через t секунд после начала движения, а также работу этой силы за первые t секунд движения. Решение. Скорость камня через t секунд после начала дви- жения v=vo + g^. Мощность, развиваемая силой тяжести в этот мо- мент, N = mgv = m{ поэтому 1 а мощность Лг<0, а при В нашем случае gvo—gvo cos (я/2 4-а) =—, Отсюда видно, что при /</о = t>U, наоборот, Лг>0. 122
Работа силы тяжести за первые t секунд t А= f N At = mgt (gt/2 — v0 sin a). 6 Графики зависимостей N(t) и A(t) показаны на рис. 4.16. ф 4.2. Консервативность сил поля. Имеются два стационарных силовых поля: 1) F=ay\; 2) ?=ах\ + Ьу], где i, j — орты осей х и у; а и 6 •— постоянные. Консервативны ли силы этих полей? Решение. Найдем работу силы каждого поля на пути от неко- торой точки 1 (xi, y\) до некоторой точки 2 (х2, г/г): х, 1) ЬА = Fdr = ау i dr = ау dx, А=а\ у их; Xi 2) ЬА = {ах i + by j) dr — ах dx + by dy, Xi У2 A = a \ xdx + b 1 у dy. В первом случае интеграл зависит от вида функции у (х), т. е. от пути, поэтому первая сила неконсервативная. Во втором же слу- чае оба интеграла не зависят от пути: они зависят только от коорди- нат начальной и конечной точек пути, следовательно, вторая сила консервативная. ф 4.3. Потенциальная энергия частицы в поле. Сила, действую- щая на частицу в некотором поле консервативных сил, имеет вид F=a(y\+x\), где а — постоянная, i и j — орты осей хну. Найти по- тенциальную энергию U(x, у) частицы в этом поле. Решение. Вычислим элементарную работу силы F на переме- щении dr и представим ее, согласно D.14), в виде убыли некоторой функции U. Эта функция и есть потенциальная энергия частицы в данном поле. Итак, ЬА = F dr = а (у dx + х dy) = — d (— аху). Отсюда U(x, у)=—аху + const. # 4.4. О разных подходах к решению. Шарик массы т подве- сили на упругой невесомой нити, жесткость которой х. Затем шарик подняли так, чтобы нить оказалась в недеформированном состоянии, и без толчка отпустили. Найти максимальное удлинение хт нити в процессе движения шарика. Решение. Рассмотрим три эквивалентных способа решения, основанных на энергетических соображениях. 1. Исходим из уравнения D.29): приращение кинетической энер- гии шарика должно быть равно алгебраической сумме работ всех сил, действующих на него. В нашем случае это сила тяжести mg и упругая сила со стороны нити FYnV = HX, где х — удлинение нити. В начальном и конечном положениях шарика его кинетическая энер- гия равна нулю (ясно, что при максимальном растяжении нити ша- 123
рик остановится), поэтому, согласно D.29), сумма работ +ЛУпр = 0, или mgxm + j (— %х) dx = mgxm — t.x\j2 = 0. о Отсюда jKm = 2/rag 2. Можно рассматривать шарик в поле тяжести Земли. При та- ком подходе следует говорить о полной механической энергии шари- ка в поле тяжести Земли. Приращение этой энергии, согласно D.31), равно работе сторонних сил. В данном случае сторонней силой надо считать силу упругости, приращение же полной механической энер- гии шарика равно приращению только его потенциальной энергии в поле тяжести Земли. Поэтому = \ ( — %x)dx=—' Отсюда тот же результат для хт. Заметим, что можно было бы поступить и наоборот, т. е. рас- сматривать шарик в поле упругой силы, тогда роль сторонней силы играла бы сила тяжести. Полезно убедиться, что и в этом случае ре- зультат будет тем же. 3. И наконец, можно рассматривать шарик в поле, образованном совместным действием и силы тяжести, и упругой силы. Тогда сто- ронние силы отсутствуют и полная механическая энергия шарика в таком поле остается постоянной в процессе движения, т. е. Д?= =<AT+AU=0. При переходе шарика из начального положения в ко- нечное (нижнее) ДГ = 0, а следовательно, и Д?/=Д[/ТЯш+Д?/упр = = 0, или Результат опять тот же. ф 4.5. Небольшое тело массы т поднимается без начальной ско- рости с поверхности Земли под действием двух сил: силы F, меняю- щейся с высотой подъема у по закону F=—2mg(l—ay), где а — по- ложительная постоянная, и силы тяжести mg. Найти работу силы F на первой половине пути подъема и соответствующее приращение потенциальной энергии тела в поле тяжести Земли. (Поле тяжести предполагается однородным.). Решение. Сначала найдем весь путь подъема. В начале и кон- це пути скорость тела равна нулю, поэтому равно нулю и прираще- ние кинетической энергии тела. Согласно же D.29), Д7" равно алгебра- ической сумме работ А силы F и силы тяжести. А так как ДГ=0, то, значит, и А = 0. Учитывая, что положительное направление оси у взято вверх, запишем Л h А= [{Fy — mg)Ay = mg \ (\-2ay) dy =mgh(\ — ah)=0. $ $ Отсюда h=l/a. 124
Работа силы F на первой половине пути подъема ft/2 й/2 ¦ f (I — ay) йу = 3mg/ia. о Соответствующее приращение потенциальной энергии Ш = mgh/2 = mg/2a. % 4.6. Потенциал и напряженность поля. Найти потенциал и на- пряженность гравитационного поля, созданного однородным шаром массы М и радиуса R, в зависимости от расстояния г до его центра. Решение. Сначала определим потенциал поля, создаваемого тонким однородным сферическим слоем вещества массы т и радиуса а. Для этого найдем потенциал dqj в точке Р(г>а), который создает элементарный пояс dS данного слоя (рис. 4.17, а). Если масса этого Р О Рис. 4.17 пояса dm и его точки находятся на расстоянии х от точки Р, то dcp = —ydm/x. Учитывая, что dm='/2 tn sin d dd, получим 1 d^ = — ¦ sin&d&. (О a. B) Преобразуем A) с помощью B) к виду dq> = У2(ут/аг)йх и проин- тегрируем это уравнение по всему слою. Тогда Далее, из теоремы косинусов (для АО АР) следует, что х2 = —2ar cos d. Взяв дифференциал этого выражения, найдем г -t-ы 1 — ут/г. C) Таким образом, потенциал в точке Р вне тонкого однородного сферического слоя таков, как если бы вся масса этого слоя была со- средоточена в его центре. Если же точка Р находится внутри слоя (г<.а), то предыдущие расчеты остаются в силе вплоть до интегрирования. Теперь пределы интегрирования по х будут от а—г до а+r. В результате 125
<Рвнутрн= — У ml a, D) т. е. потенциал внутри слоя не зависит от положения точки Р, а сле- довательно, он одинаков во всех точках внутри слоя. Напряженность поля в точке Р, согласно D.26), d<f i —упг/г2 вне слоя, дг ' 0 внутри слоя. Графики зависимостей ф(г) и G(r) для тонкого однородного сферического слоя показаны на рис. 4.17, б. М Рис. 4.18 Рис. 4.19 Обобщим полученные результаты на однородный шар массы М и радиуса R. Если точка Р находится вне шара (r>R), то из форму- лы C) сразу следует ?вне= — У.Щг. E) Если же точка Р находится внутри шара (г<Я), то потенциал ф в этой точке можно представить как сумму: где ф! — потенциал от шара радиуса г, ц>г — потенциал от слоя с ра- диусами от г до R. Согласно E), М (г//?K М "( Потенциал же ф2, создаваемый слоем, одинаков во всех точках внут- ри этого слоя. Проще всего фг вычислить для точки, находящейся в центре слоя: /? (Ш 3 уМ где dM = 3(JVJ/i?3) r2dr — масса тонкого слоя между радиусами г и r+dr. В результате ^внутри = ?1 + ?2 = ~ Xh C—/-2//?2). F) 126
Напряженность поля в точке Р, как следует из E) и F), — Y М/r2 при г > R, дг \—yMr/R3 при r^R. Графики зависимостей ц>(г) и G(r) для однородного шара радиу- са R показаны на рис. 4.18. #4.7. Космические скорости. Показать, что кинетическая энер- гия Тг, которую необходимо сообщить телу для удаления его за пре- делы земного притяжения, в два раза превышает кинетическую энергию Tt, необходимую для выведения этого тела на круговую орбиту искусственного спутника Земли (вблизи ее поверхности). Сопротивлением воздуха и вращением Земли пренебречь. Решение. Найдем скорость vi тела, движущегося по круговой орбите. Согласно основному уравнению динамики, где m — масса тела, R — радиус орбиты, приблизительно равный ра- диусу Земли. Отсюда «1 = /г7г = 7,9 км/с. Это первая космическая скорость. Для того чтобы тело могло преодолеть поле тяготения Земли, ему необходимо сообщить вторую космическую скорость v^ Ее можно найти из закона сохранения энергии: кинетическая энергия тела вблизи поверхности Земли должна быть равна глубине потенциальной ямы в этом месте. Последняя равна приращению по- тенциальной энергии тела в поле тяготения Земли между точками rt = R и Г2 = о°. Таким образом, где М — масса Земли. Отсюда v2 = y2y M/R = Y2gR = 11 км/с Следовательно, y2 = yiV2 и Г2 = 27\. % 4.8. Решение в неинерциальной системе отсчета. Плоскую жесткую спираль из гладкой проволоки, расположенную в горизон- тальной плоскости, вращают с постоянной угловой скоростью со во- круг неподвижной вертикальной оси О (рис. 4.19). По спирали сколь- зит небольшая муфта М. Найти ее скорость и' относительно спирали как функцию расстояния р от оси вращения О, если муфта начала двигаться от этой оси со скоростью vo'. Решение. Этот вопрос наиболее целесообразно решать в си- стеме отсчета, связанной со спиралью. Известно, что приращение ки- нетической энергии тела должно быть равно алгебраической сумме работ всех сил, действующих на тело. В нашем случае из всех сил работу будет совершать только центробежная сила инерции. Все остальные силы — сила тяжести, сила реакции со стороны спирали и сила Кориолиса — перпендикулярны скорости v' муфты, поэтому работы не совершают. 127
Согласно уравнению D.29), 1 Tm{v^-v'i)=^ где т — масса муфты, dr — ее элементарное перемещение относи- тельно спирали. Так как скалярное произведение р dr = p(dr)p = = pdp, то интеграл оказывается равным '/г"гсо2р2. Отсюда искомая скорость 0 4.9. Система частиц. Три одинаковые заряженные частицы, каждая массы тис зарядом q, поместили в вершины углов равно- стороннего треугольника со стороной а. Затем частицы одновременно освободили, и они стали симметрично разлетаться под действием ку- лоновских сил отталкивания. Найти: 1) скорость каждой частицы в зависимости от расстояния г меж- ду ними; 2) работу Аи которую совершили кулоновские силы, действую- щие на каждую частицу при разлете их на очень большое расстояния друг от друга. Решение. 1. Данная система замкнутая, поэтому для нее при- ращение кинетической энергии равно убыли потенциальной, т. р Отсюда v = у 2kqi (г — а)/тга . Видно, что при г->оо скорость каждой частицы стремится к предель- ному значению 2. Работа всех сил взаимодействия при изменении конфигурации этой системы равна убыли потенциальной энергии системы: где учтено, что в конечном положении ?/2 = 0. Отсюда искомая ра- бота Замечание. Следует обратить внимание на одну часто встре- чающуюся ошибку при решении подобного рода задач. Рассуждают гак: в начальном положении потенциальная энергия каждой частицы в поле других двух равна 2kq2la, а в конечном — нуль. Отсюда иско- мая работа Ai = 2kqz/a. Как видно, полученный результат отличается вдвое от (ft). Почему? 128
Ошибка здесь в том, что поле, в котором перемещается каждая частица, нестационарное (ведь другие две частицы тоже перемеща- ются относительно друг друга), поэтому работа сил такого поля не иожет быть представлена как убыль потенциальной энергии частицы в поле. #4.10. Внутренняя механическая энергия системы. Система со- стоит из двух шариков с массами т4 и т2, которые соединены меж- ду собой невесомой пружинкой. Шарикам сообщили скорости Vi и V2 соответственно, после чего система начала двигаться в однородном поле сил тяжести Земли. Пренебрегая сопротивлением воздуха и счи- тая, что в начальный момент пружинка не деформирована и vt±V2, найти внутреннюю механическую энергию данной системы в процес- се движения. Решение. Внутренняя механическая энергия системы — это ее энергия Е в //-системе. Здесь //-система движется с ускорением g, поэтому в этой системе отсчета на каждый шарик действуют две внешние силы: сила тяжести m,g и сила инерции, равная —ntig. Ясно, что суммарная работа этих внешних сил равна нулю (в Д-сис- теме), а следовательно, энергия Е меняться не будет. Чтобы ее най- ти, достаточно рассмотреть начальный момент, когда пружинка еще не деформирована и энергия Е равна только суммарной кинетиче- ской энергии f0 в //-системе. Воспользовавшись формулой D.61), получим # 4.11. Столкновение частиц. В /(-системе отсчета частица / массы т\ налетает на покоящуюся частицу 2 массы т^. Заряд каж- дой частицы равен q. Найти минимальное расстояние, на которое они сблизятся при лобовом «соударении», если кинетическая энер- гия частицы / вдали от частицы 2 равна Т\. Решение. Рассмотрим процесс столкновения отдельно в К- и Ц-системах отсчета. 1. В /(-системе в момент наибольшего сближения обе частицы будут двигаться как единое целое со скоростью и, которую можно определить из закона сохранения импульса." Pi = (Щ + Щ) v, где /?i — импульс налетающей частицы, р\ = yr2m\ Tx . Из закона сохранения энергии следует, что где приращение потенциальной энергии системы AU = l Исключив v из этих двух уравнений, найдем 2. В //-системе решение наиболее просто: здесь суммарная кине- тическая энергия частиц идет целиком на приращение потенциальной энергии системы частиц в момент их наибольшего сближения: 129
где, согласно D.61), Т= сюда легко найти гМИн- , Д[/=й(?2//-мин. От- #4.12. Частица массы т( с импульсом р4 испытала упругое столкновение с покоившейся частицей массы /п2. Найти импульс р,' первой частицы после столкновения, если в результате столкновения она рассеялась под углом ¦& к первоначальному направлению дви- жения. Решение. Из закона сохранения импульса (рис. 4.20) находим Рг=р\+Р?— 2pi/>icos», A) где р/ — импульс покоившейся частицы после столкновения. Рис. 4.21 Далее, из закона сохранения энергии следует, что Ti = Ti'+T2', где Т\ и Т% — кинетические энергии первой и второй частиц после столкновения. Преобразуем это равенство с помощью соотношения T=p2l2m к виду Р22 = (/>1-/>12)да2/«1- B) Исключив р2'2 из A) и B), получим cos a ± у > Pi=Pi Если mi<im2, то физический смысл имеет только знак плюс перед корнем. Это следует из того, что при этом условии корень будет больше, чем cos О, а так как р/ — это модуль вектора, то, естест- венно, он не может быть отрицательным. Если же шОтг, то физический смысл имеют оба знака перед корнем — ответ в этом случае неоднозначен: под углом д импульс рассеянной частицы может иметь одно из двух значений (это зави- сит от относительного расположения частиц в момент соударения). Последний случай соответствует векторной диаграмме, показанной на рис. 4.14, в. #4.13. Какую часть ti своей кинетической энергии теряет час- тица массы т.\ при упругом рассеянии под предельным углом на по- коящейся частице массы m2(/ni>m2)? Решение. Пусть Гь pi и Т'\, р\ — кинетическая энергия и им- пульс налетающей частицы до и после рассеяния. Тогда Ч = G*1 — Т[IТ1 = 1 - Т\]ТХ = 1 - to'/Л)8. (О 130
т. е. задача сводится к нахождению отношения р\1р\. Воспользуемся векторной диаграммой импульсов, соответствую- щей предельному углу di пр (рис. 4.21). Из прямоугольного тре- угольника АС О следует, что Р? = (Л - РJ -P2 = P2i- 2рхр, откуда (Pi/Pii2 = 1 - 2Р/Р1 = I - 2Щ/(Щ + mi). B) После подстановки B) в A) получим #4.14. Атом массы от4 испытал неупругое столкновение с по- коившейся молекулой массы т2. После соударения обе частицы раз- летелись под углом Э друг к другу с кинетическими энергиями 7Y и 7Y соответственно, причем молекула оказалась в возбужденном состоянии — ее внутренняя энергия увеличилась на определенную ве- личину Q. Найти Q, а также пороговую кинетическую энергию атома, при которой возможен переход молекулы в данное возбужденное со- стояние. Решение. Из законов сохранения энергии и импульса в этом процессе' следует: р\ = Р\ + Рг + 2Pi Рг cos e > где штрихами отмечены величины после соударения (второе соотно- шение сразу следует из треугольника импульсов согласно теореме ко- синусов). Воспользовавшись формулой р2 = 2тГ, исключим Т, из этих уравнений. В результате получим Q = («2/«l- 1)^2 + 2]/ (m2/m1)T1 T'2cosQ Тх пор #4.15. Распад частицы. Частица с импульсом р0 (в /С-системе) распалась на лету на две частицы с массами nit и тг. При этом вы- делилась энергия Q — энергия распада (она перешла в кинетическую энергию). Построить векторную диаграмму импульсов для этого процесса и найти с помощью нее возможные импульсы pi и р2 воз- никших частиц. Решение. Наиболее просто этот процесс выглядит в Д-систе- ме: здесь распадающаяся частица покоится, а частицы распада раз- летаются в противоположные стороны с одинаковыми по модулю импульсами р\=р2=р. Энергия распада Q целиком переходит в суммарную кинетическую энергию Т возникающих частиц. Поэтому р =1 где ц — приведенная масса системы возникших частиц. 131
Найдем импульсы возникших частиц в /С-системе. Воспользовавшись формулой преобразования скоростей при переходе от Ц- к /(-системе, запишем: Pi = mi vx = m1(Vc Р2 = = т. (Vc = «i VC + = m2 VC + ] причем, согласно закону сохранения импульса, pi+p2=po. С помощью этих фор- мул построим векторную , ^ \ л диаграмму импульсов ^ ^ (рис. 4.22). Изобразим сначала отрезок АВ, рав- аый импульсу р0. Затем радиусом р проведем ок- ружность с центром в точке О, которая делит р0 ' отрезок АВ на две части в отношении mi : m%. Эта Рис. 4.22 окружность и есть гео- метрическое место точек всех возможных положений вершины С треугольника импульсов ABC. Глава 5 ЗАКОН СОХРАНЕНИЯ МОМЕНТА ИМПУЛЬСА § 5.1. Момент импульса частицы. Момент силы Анализ поведения систем показывает, что кроме энер- гии и импульса существует еще одна механическая вели- чина, с которой также связан закон сохранения, — это так называемый момент и м пул ьс а *. Что это за вели- чина и каковы ее свойства? Сначала возьмем одну частицу. Пусть г — радиус-ве- ктор, характеризующий ее положение относительно не- которой точки О выбранной системы отсчета, ар — ее им- пульс в этой системе. Моментом импульса частицы А от- носительно толчки О (рис. 5.1) называют вектор L, рав- ный векторному произведению векторов г и р: < = [гр]. E.1) Из этого определения следует, что L является аксиаль- ным вектором. Его направление выбрано так, что вра- щение вокруг точки О в направлении вектора р и вектор * Используют также названия момент количества дви- жения, углов ой момент или просто момент. 132
L образуют правовинтовую систему. Модуль вектора L равен L = rp sin a = lp, E.2) где а — угол между г и р, / = rsincc — плечо вектора р относительно точки О (рис. 5.1). Уравнение моментов. Выясним, какая механическая величина ответственна за изменение вектора L в данной м Рис. 5 1 Рис. 5.2 системе отсчета. Для этого продифференцируем E.1) по времени: - ' dL/& = [dr/d/, p] + [r, dp/d^j. Так как точка О неподвижна, то вектор dr/dt равен ско- рости v частицы, т. е. совпадает по направлению с векто- ром р, поэтому t, р] = 0. Далее, согласно второму закону Ньютона, dp/d/=F, где F — равнодействующая всех сил, приложенных к частице. Следовательно, Величину, стоящую в правой части этого уравнения, на- зывают моментом силы F относительно точки О (рис. 5.2). Обозначив ее буквой М, запишем L = [rF]. E.3) Вектор М, как и L, является аксиальным. Модуль этого вектора, аналогично E.2), равен М=/ E.4) 133
где I — плечо вектора F относительно точки О (рис. 5.2). Итак, производная по времени от момента импульса L частицы относительно некоторой точки О выбранной системы отсчета равна моменту М равнодействующей си- лы F относительно той же точки О: E.5) Это уравнение называют уравнением моментов. Заметим, что если система отсчета является неинерциаль- Рис. 5.3 Рис. 5.4 ной, то момент силы М включает в себя как момент сил взаимодействия, так и момент сил инерции (относитель- но той же точки О). Из уравнения моментов E.5), в частности, следует, что если М=0, то L = const. Другими словами, если от- носительно некоторой точки О выбранной системы отсче- та момент всех сил, действующих на частицу, равен ну- лю в течение интересующего нас промежутка времени, то относительно этой точки момент импульса частицы ос- тается постоянным в течение этого времени. Пример 1. Некоторая планета А движется в поле тяготения Солнца С (рис. 5.3). Относительно какой точки гелиоцентрической системы отсчета момент импульса данной планеты будет сохраняться во времени? Для ответа на этот вопрос прежде всего необходимо установить, какие силы действуют на планету А. В данном случае это только сила тяготения F со стороны Солнца. Так как при движении планеты направление этой силы все время проходит через центр Солнца, то последний и является той точкой, относительно которой момент силы F все время равен нулю, и момент импульса планеты будет оставать- ся постоянным. Импульс же р планеты при этом будет меняться. Пример 2. Шайба А, двигаясь по гладкой горизонтальной плос- кости, упруго отскакивает от гладкой вертикальной стенки (рис. 5.4, 134
вид сверху). Найдем точку, относительно которой момент импульса шайбы будет оставаться постоянным в этом процессе. На шайбу действуют сила тяжести, сила реакции со стороны горизонтальной плоскости и сила реакции R со стороны стенки в мо- мент удара о нее. Первые две силы уравновешивают друг друга, остается сила R. Ее момент равен нулю относительно любой точки, лежащей на линии действия вектора R, а значит, относительно лю- бой из этих точек момент импульса шайбы будет оставаться постоян- ным в данном процессе. Пример 3. На горизонтальной гладкой плоскости находятся не- подвижный вертикальный цилиндр и шайба А, соединенная с ци- линдром горизонтальной нитью АВ (рис. 5.5, вид сверху). Шайбе со- общили начальную скорость v, как показано на рисунке. Есть ли здесь точка, относительно которой момент импульса шайбы будет ос- таваться постоянным в процессе движения? В данном случае единственная некомпенсированная сила, действу- р g с ющая на шайбу А, — это сила на-' ' ' гяжения F со стороны нити. Не- трудно видеть, что точки, относительно которой момент силы F в процессе движения был бы все время равен нулю, здесь нет. А сле- довательно, нет и точки, относительно которой момент импульса шай- бы оставался бы постоянным. Этот пример показывает, что не всегда существует точка, отно- сительно которой момент импульса частицы оставался бы посто- янным. Уравнение моментов E.5) позволяет получить ответ на два вопроса: 1) найти момент силы Ж относительно интересующей нас точки О в любой момент времени t, если известна за- висимость от времени момента импульса L(t) частицы, относительно той же точки; 2) определить приращение момента импульса части- цы относительно точки О за любой промежуток времени, если известна зависимость от времени момента силы М(^), действующего на эту частицу (относительно той же точки О). Решение первого вопроса сводится к нахождению про- изводной по времени от момента импульса; т. е. dL/dt, которая и равна, согласно E.5), искомому моменту си- лы М. Решение же второго вопроса сводится к интегрирова- нию уравнения E.5). Умножив обе части этого уравнения на d^ получим dL=McU — выражение, которое определя- ет элементарное приращение вектора L. Проинтегриро- вав это выражение по времени, найдем приращение век- тора L за конечный промежуток времени t: 135
E.6) Величину, стоящею в правой части этого уравнения, на- зывают импульсом момента силы. Таким об- разом, приращение момента импульса частицы за любой промежуток времени равно импульсу момента силы за эго же время. Рассмотрим два примера. Рис 5 7 Пример 1 Момент импульса частицы относительно некоторой точки меняется со временем t no закону L(i)=a+b<2, где а и b — некоторые постоянные векторы, причем a_Lb Найдем момент силы М, действующий на частицу, когда угол между векторами М и L ока- жется равным 45° Согласно E 5), M = dL/dif=2bi', т е вектор М все время совпа- дает по направлению с вектором b Изобразим векторы М и L в не- который момент t (рис 5 6) Из этого рисунка видно, что угол а= = 45° в момент t0, когда а = 6/02 Отсюда to=^ajb и М = 2Уа/6 b Пример 2 Камень А массы т бросили под углом к горизонту с начальной скоростью v0 Пренебрегая сопротивлением воздуха, най- дем зависимость от времени момента импульса камня L(t) относи- тельно точки бросания О (рис 5 7) За промежуток времени At момент импульса камня относительно точки О получит приращение dL=Mdif=[r, mg]dt Так как r=\ot + + gt2/2 (см с 12), то dL=[v0, mg]tdt Проинтегрировав это выраже ние с учетом того, что в момент / = 0 L@)=0, получим L(^) = = [vo, mg]t2l2 Отсюда видно, что направление вектора L остается не- изменным в процессе движения (вектор L направлен за плоскость, рис 5 7) Момент импульса и момент силы относительно оси. Возьмем в интересующей нас системе отсчета произ- вольную неподвижную ось z. Пусть относительно неко- торой точки О на оси z момент импульса частицы А равен L, а момент силы, действующий на частицу,— М. Моментом импульса относительно оси z называют про- 136
екцию на эту ось вектора L, определенного относитель- но произвольной точки О данной оси (рис. 5.8). Анало- i Рис. 5.8 Рис. 5.9 гично вводят и понятие момента силы относительно оси. Их обозначают соответственно Lz и Мг. Далее мы уви- дим, что Lz и Мг не зависят от выбора точки О на оси г. Выясним свойства этих величин. Записав уравнение E.5) в проекциях на ось г, полу- чим dLjdt=Mz, E.7) т. е. производная по времени от момента импульса частицы отно- сительно оси z равна моменту си- лы относительно этой оси. В част- ности, если Mz = 0, то Lz = const. Другими словами, если момент силы относительно некоторой не- подвижной оси z равен нулю, то момент импульса частицы отно- сительно этой оси остается посто- янным. При этом сам вектор L может и меняться. Пример. Небольшое тело массы т, подвешенное на нити, равно- мерно движется по горизонтальной окружности (рис. 5.9) под дейст- вием силы тяжести mg и силы натяжения Т со стороны нити. Отно сительно точки О момент импульса тела — вектор L — находится в одной плоскости с осью г и нитью, и при движении тела вектор L под действием момента М силы тяжести все время поворачивается, т. е. меняется. Проекция же Lz остается при этом постоянной, так как вектор М перпендикулярен оси г и Afz = O. 137 Рис. 5.10
Найдем теперь аналитические выражения для Lz и Mz. Нетрудно видеть, что эта задача сводится к нахож- дению проекций на ось z векторных произведений [гр] и [rF]. Воспользуемся цилиндрической системой координат р, ф, z, связав с частицей А (рис. 5.10) орты ер, еф, ez, направленные в сторону возрастания соответствующих координат. В этой системе координат радиус-вектор г и импульс р частицы записывают так: где рр, Ptp, pz — проекции вектора р на соответствующие орты. Из векторной алгебры известно, что векторное про- изведение [гр] можно представить определителем еР е9 ег р 0 z Р? Pf Рг Отсюда сразу видно, что момент импульса частицы от- носительно оси z Lz=?p,, E.8) где р — расстояние частицы от оси z. Преобразуем это выражение к виду, более удобному для практических применений. Имея в виду, что рф = отиф = отрйJ, получим 4 = тер2(«г) E.9) где (oz — проекция угловой скорости to, с которой пово- рачивается радиус-вектор частицы. Аналогично E.8) записывается и момент силы отно- сительно оси z: MZ=PF9, E.10) где Fy — проекция вектора силы F на орт еф. Обратим внимание, что проекции Lz и Mz действи- тельно не зависят от выбора точки О на оси г, относи- тельно которой определены векторы L и М. Кроме того, видно, что Lz и Mz — величины алгебраические, их знаки соответствуют знакам проекций рф и Fv. § 5.2. Закон сохранения момента импульса Выберем произвольную систему частиц. Введем по- нятие момента импульса данной системы как векторную сумму моментов импульсов ее отдельных частиц: 138
l-УЛ/. EЛ1> где все векторы определены относительно одной и той же точки О заданной системы отсчета. Заметим, что мо- мент импульса системы — величина аддитивная: момент импульса системы равен сумме моментов импульсов ее отдельных частей независимо от того, взаимодействуют они между собой или нет. Выясним, какая величина определяет изменение мо- мента импульса системы. Для этого продифференцируем E.11) по времени: dL/cU=2iLj/d?. В предыдущем пара- графе было показано, что производная dLj/ctf равна мо- менту всех сил, действующих на t-ю частицу. Предста- вим этот момент в виде суммы моментов внутренних и внешних сил, т. е. М/ + М*. Тогда Здесь первая сумма — это суммарный момент всех вну- тренних сил относительно точки О, вторая сумма — сум- марный момент всех внешних сил относительно той же точки О. Покажем, что суммарный момент всех внутренних сил относительно любой точки равен нулю. Действительно, внутренние силы — это силы взаимодействия между час- тицами данной системы. По третьему закону Ньютона, эти силы попарно одинаковы по модулю, противополож- ны по направлению и лежат на одной прямой, т. е. имеют одинаковое плечо. Поэтому моменты сил каждой пары взаимодействия равны по модулю и противоположны по направлению, т. е. уравновешивают друг друга, и, значит, суммарный момент всех внутренних сил всегда равен ну- лю. В результате последнее уравнение принимает вид E.12) где Мвнеш — суммарный момент всех внешних сил, Мвнеш = 2М'- Уравнение E.12) утверждает: производная момента импульса системы по времени равна суммарному момен- ту всех внешних сил. Разумеется, оба момента, Lh M, здесь определены относительно одной и той же точки О заданной системы отсчета. 139
Как и в случае одной частицы, из уравнения E.12) следует, что приращение момента импульса системы за конечный промежуток времени t E.13) т. е. приращение момента импульса системы равно им- пульсу суммарного момента всех внешних сил за соот- ветствующий промежуток времени. И здесь, конечно, оба момента, L и МВнеш определены относительно одной и той же точки О выбранной системы отсчета. Уравнения E.12) и E.13) справедливы как в инер- циальной, так и в неинерциальной системах отсчета. Толь- ко в неинерциальной системе отсчета нужно учитывать и действие сил инерции, играющих роль внешних сил, т. е. под МВнеш в этих уравнениях следует понимать сумму Мвз+Мин. где Мвз — суммарный момент внешних сил взаимодействия, МИц — суммарный момент сил инерции (относительно одной и той же точки О системы отсчета). Итак, мы пришли к важному выводу, согласно урав- нению E.12), момент импульса системы может изменять- ся под действием только суммарного момента всех внеш- них сил. Отсюда непосредственно вытекает и другой ва- жный вывод — закон сохранения момента им- пульса: момент импульса замкнутой системы частиц остается по- стоянным, т. е. не меняется со временем, причем это справедливо для момента импульса, взятого относитель- но любой точки инерциальной системы отсчета. Таким образом, в.инерциальной системе отсчета мо- мент импульса замкнутой системы частиц I E.14) При этом моменты импульса отдельных частей или час- тиц замкнутой системы могут изменяться со временем, что и подчеркнуто в последнем выражении. Однако эти изменения всегда происходят так, что приращение мо- мента импульса одной части системы равно убыли мо- мента импульса ее другой части (конечно, относительно одной и той же точки системы отсчета). В этом смысле уравнения E.12) и E.13) можно рас- сматривать как более общую формулировку закона со- 140
хранения момента импульса, формулировку, в которой указана и причина изменения момента импульса интере- сующей нас системы — действие других тел (через момент внешних сил взаимодействия). Сказанное, разумеется, справедливо только по отношению к инерциальным сис- темам отсчета. Подчеркнем еще раз: закон сохранения момента им- пульса имеет место только по отношению к инерциаль- ным системам отсчета. Однако это не исключает случаев, Нить Рис. 5.11 когда момент импульса системы сохраняется и в неинер- циальных системах отсчета. Для этого достаточно, чтобы согласно уравнению E.12) —а оно справедливо и в не- инерциальных системах отсчета — суммарный момент всех внешних сил (включая и силы инерции) был равен нулю. Такие ситуации реализуются довольно редко и со- ответствующие случаи имеют весьма частный характер. Закон сохранения момента импульса играет такую же важную роль, как и законы сохранения энергии и им- пульса. Уже сам по себе он позволяет сделать во многих случаях ряд существенных заключений о свойствах тех или иных процессов, совершенно не вникая в их деталь- ное рассмотрение. Проиллюстрируем сказанное на таком примере. Пример. Два одинаковых шара насажены на гладкий горизон- тальный стержень, по которому они могут скользить (рис. 5.11). Шары сближают и соединяют нитью. Затем всю установку приводят во вращение вокруг вертикальной оси, предоставляют ее самой себе и пережигают нить. Шары, естественно, разлетаются к концам стерж- ня. Угловая скорость установки при этом резко уменьшается. Наблюдаемый эффект является прямым следствием закона со- хранения момента импульса. Данная установка ведет себя, по суще- ству, как замкнутая: внешние силы компенсируют друг друга, силы трения в оси предполагаются пренебрежимо малыми. Для количест- венной оценки изменения угловой скорости будем считать, что масса всей установки практически сосредоточена в шарах, а их размеры 141
достаточно малы. Тогда из равенства моментов импульса шаров от- носительно точки С в начальном и конечном состояниях системы, 2m[riVi]=2m[r2v2], следует Отсюда видно, что с увеличением расстояния г шаров от оси враще- ния угловая скорость установки уменьшается (как 1/г2). И наоборот, если бы расстояние между шарами уменьшалось (под действием каких-либо внутренних сил), угловая скорость установки увеличивалась бы. Этот эф- фект имеет общий характер, и его широко используют, например, фигуристы и гим- насты. Обратим внимание на тот факт, что конечный ре- вультат совершенно не зави- сит от характера внутренних сил (здесь — это силы тре- Рис. 5.12 ния между шарами и стерж- нем). Особый интерес представляют случаи, когда момент импульса L сохраняется для незамкнутых систем, у ко- торых, как известно, импульс р меняется со временем. Если относительно некоторой точки О выбранной систе- мы отсчета, суммарный момент внешних сил МВнеш=0 в течение интересующего нас промежутка времени, то, со- гласно E.12), момент импульса системы относительно точки О сохраняется за это время. В незамкнутых систе- мах такой точки, вообще говоря, может и не быть, что следует прежде всего выяснить для каждого конкретного случая. Пример 1. Система Земля — Луна, движущаяся в поле тяготения Солнца, является незамкнутой. Ее импульс все время меняется под действием сил тяготения со стороны Солнца. Здесь, однако, имеется одна точка, относительно которой момент сил тяготения, действую- щих на данную систему, все время равен нулю, — это центр Солнца. Поэтому можно сразу утверждать, что момент импульса системы Земля — Луна относительно центра Солнца остается постоянным. Пример 2. На гладкой горизонтальной плоскости лежит стержень OS, который может свободно вращаться вокруг неподвижной вер- тикальной оси, проходящей через его конец О (рис. 5.12). В конец В стержня попадает, застревая, шайба А, скользившая по плоскости, и вся система начинает вращаться как единое целое вокруг точки О. Ясно, что система шайба — стержень незамкнутая: кроме сил, уравновешивающих друг друга в вертикальном направлении, со сто- роны оси в процессе удара будет действовать горизонтальная сила, а после того, как стержень начнет вращаться, возникает еще одна сила со стороны оси, благодаря которой центр масс системы будет двигаться по окружности. Но обе силы проходят через точку О, а 142
следовательно, момент этих внешних сил относительно точки О все время равен нулю. Отсюда вывод: момент импульса данной системы будет оставаться постоянным относительно точки О. В более ограниченном случае у незамкнутых систем может сохраняться не сам момент импульса L, а его проекция на некоторую неподвижную ось г. Это бывает тогда, когда проекция суммарного момента Мвнеш всех внешних сил на эту ось z равна нулю. В самом деле, за- писав уравнение E.12) в проекциях на ось z, получим dLj&=Mmmt. E.15) Здесь Lz и МвнеШ2 — момент импульса и суммарный мо- мент внешних сил относительно оси z: *i» EЛ6) где L,z и Mlz — момент импульса и момент внешних сил относительно оси z для t'-й частицы системы. Из уравнения E.15) следует, что если относительно некоторой неподвижной в данной системе отсчета оси г проекция Мвнеш z=О, то момент импульса системы относи- тельно этой оси сохраняется: E.17) При этом сам вектор L, определенный относительно про- извольной точки О на этой оси, может меняться. Напри- мер, если система движется в однородном поле тяжести, то суммарный момент всех сил тяжести относительно любой неподвижной точки О перпендикулярен вертика- ли, а значит, относительно любой вертикальной оси •МВНеш z= 0 и Lz=const, чего нельзя сказать о векторе L. Рассуждения, которые приводят к закону сохранения момента импульса, целиком опираются на справедли- вость законов Ньютона. А как обстоит дело в системах, не подчиняющихся этим законам, например в системах с электромагнитным излучением, в атомах, ядрах и др.? Учитывая громадную роль, которую играет закон со- хранения момента импульса, в физике понятие момента импульса расширяют на немеханические системы (кото- рые не подчиняются законам Ньютона) и постулируют закон сохранения момента импульса для всех физиче- ских процессов. Такой расширенный закон сохранения момента им- пульса уже не является следствием законов Ньютона, 3 представляет собой самостоятельный общий принцип, ИЗ
являющийся обобщением опытных фактов. Наряду с за- конами сохранения энергии и импульса закон сохранения момента импульса является одним из фундаментальных законов природы. § 5.3. Собственный момент импульса В предыдущем параграфе было установлено, что мо- мент импульса L системы изменяется только под дейст- вием суммарного момента М всех внешних сил; именно этот вектор М .определяет поведение вектора L. Те- шерь рассмотрим некоторые наиболее существенные свойства этих величин и те важные выводы, которые из них вытекают. Суммарный момент внешних сил. Как и момент каждой силы, суммар- ный момент сил зависит, вообще Рис. 5 13 говоря, от выбора точки, относитель- но которой его определяют. Пусть М — суммарный мо- мент сил относительно точки О, а М' — относительно точки О', радиус-вектор которой г0 (рис. 5.13). Найдем связь между М и М'. Радиусы-векторы г« и г', точки приложения силы F, связаны соотношением гг = г'(+г0 (рис. 5.13). Поэтому выражение для М можно записать в таком виде: или [ = M' + [r0F], E.18) где F=2^Ft — результирующая всех внешних сил. Из формулы E.18) видно, что если F=0, то суммар- ный момент внешних сил не зависит от выбора точки, относительно которой его определяют. Таков, в частно- сти, случай, когда к системе приложена пара сил. Пример. К телу в точках 1 к 2 приложены две одинаковые по модулю и противоположно направленные силы F, и F2, не действую- щие вдоль одной прямой (пара сил) Пусть ri2—радиус-вектор, про- веденный из точки / в точку 2 Найдем суммарный момент М этой пары сил Здесь результирующая сила F—Fi + F2 = 0, поэтому согласно E 18) момент М этой пары сил не должен зависеть от выбора точки 144
О, относительно которой его определяют. Воспользовавшись этим, выберем в качестве точки О точку 1 (относительно нее момент силы Fi равен нулю), тогда М = [г12Р2]. Модуль вектора М равен, как нетрудно сообразить, M = tF, где / — плечо пары, т. е. расстояние между прямыми, вдоль которых действуют силы, a F — модуль каждой силы. Интересной и важной особенностью в этом отноше- нии обладает Д-система (напомним, что эта система от- счета жестко связана с центром масс системы частиц и перемещается поступательно по отношению к инерци- альным системам). Так как в общем случае Д-система является неинерциальной, то результирующая всех внешних сил должна включать в себя кроме внешних сил взаимодействия FB3 и силы инерции F,1H. С другой стороны, в Д-системе система частиц как целое покоит- ся, а это значит, согласно C.11), что F=FB3 + Fhh=0. Имея в виду E.18), мы приходим к следующему важ- ному выводу: в Ц-системе суммарный момент всех внеш- них сил, включая силы инерции, не зависит от выбора точки О. И другой важный вывод: в Ц-системе суммарный момент сил инерции относитель- но центра масс всегда равен нулю: E.19) М™=0. В самом деле, сила инерции, действующая на каждую частицу системы, F,- = —m,a0,. где а0 — ускорение Д-систе- мы. Поэтому суммарный момент всех этих сил относи- тельно центра масс С ,). а0]'. Согласно C.8),2m«r'='mrc, а так как в нашем слу- чае гс = 0, то и Mc™=0. Собственный момент импульса. Как и момент сил, мо- мент импульса системы зависит, вообще говоря, от вы- бора точки О, относительно которой его определяют. При переносе этой точки на расстояние г0 (рис. 5.13) новые радиусы-векторы частиц г',- связаны со старыми гг фор- мулой Гг = г/г + г0. Поэтому момент импульса системы от- носительно точки О можно представить так: 145
или L=L' + [rop], | E.20) где I/ — момент импульса системы относительно точки О', a p=2pi — полный импульс системы. Из формулы E.20) следует, что если полный импульс системы р = 0, то ее момент импульса не зависит от вы- бора точки О. А этим как раз и отличается /^-система, в которой система частиц как целое покоится. Отсюда мы приходим к третьему важному выводу: в Ц-системе момент импульса системы частиц не зависит от выбора точки, относительно которой его определяют. Этот момент называют собственным момен- том импульса системы и обозначают L. Связь между L и L. Пусть L — момент импульса си- стемы частиц относительно точки О К-системы отсчета. Так как собственный момент импульса L в /^-системе не зависит от выбора точки О', возьмем точку О' совпа- дающей в данный момент с точкой О ^-системы. Тогда радиусы-векторы каждой частицы в обеих системах от- счета будут одинаковы в этот момент (г'; = г,), скорости же частиц связаны формулой v, = v,. + Vc, E.21) где Vc — скорость /^-системы относительно .К-системы. Поэтому можно записать i/[r/V,] = ' Первая сумма в правой части этого равенства есть соб- ственный момент импульса L. Вторую сумму в соответ- ствии с формулой C.8) представим как m[rc\c], или [гср], где m — масса всей системы, гс — радиус-вектор ее центра масс в ^-системе, р — суммарный импульс сис- темы частиц. В результате E.23) т. е. момент импульса L системы частиц складывается из ее собственного момента импульса L и момента [гср], обусловленного движением системы частиц как целого. Возьмем, например, однородный шар, скатывающий- ся по наклонной плоскости. Его момент импульса отно- 146
сительно некоторой точки этой плоскости складывается из момента импульса, связанного с движением центра масс шара, и собственного момента импульса, обуслов- ленного вращением шара вокруг собственной оси. Из формулы E.23), в частности, следует, что если центр масс системы покоится (импульс системы р = 0), то ее момент импульса L — это собственный момент им- пульса. С этим случаем мы уже знакомы. В другом край- нем случае, когда L=0, момент импульса системы отно- сительно некоторой точки определяется только момен- том, связанным с движением системы как целого, т. е. вторым слагаемым E.23). Так, например, ведет себя мо- мент импульса любого твердого тела, совершающего по- ступательное движение. Уравнение моментов в Д-системе. В предыдущем па- раграфе было отмечено, что уравнение E.12) справедли- во в любой системе отсчета. Значит, оно справедливо и в Д-системе. Поэтому сразу можно записать: dL/cU=M, где М — суммарный момент внешних сил в /^-системе. Так как Д-система в общем случае неинерциальная, то в М входит помимо моментов внешних сил взаимодей- ствия и момент сил инерции. С другой стороны, в нача- ле этого параграфа (см. с. 145) было показано, что мо- мент сил М в Д-системе не зависит от выбора точки, от- носительно которой его определяют. Обычно в качестве такой точки берут точку С — центр масс системы. Целе- сообразность выбора именно этой точки в том, что отно- сительно ее суммарный момент сил инерции равен ну- лю, поэтому следует учитывать только суммарный мо- мент внешних сил взаимодействия Мс. Итак, E.24) т. е. производная по времени от собственного момента импульса системы равна суммарному моменту всех внеш- них сил взаимодействия относительно центра масс дан- ной системы. В частности, если Мс = 0, то L= const, т. е. собствен- ный момент импульса системы сохраняется. В проекциях на ось г, проходящую через центр масс системы, уравнение E.24) имеет вид dZ,/d/= MCz, E.25) 147
где Mcz — суммарный момент внешних сил взаимодейст- вия относительно неподвижной в /^-системе оси г, прохо- дящей через центр масс. И здесь если Mcz=0, то L2= = const. § 5.4. Динамика твердого тела Движение твердого тела в общем случае определя- ется двумя векторными уравнениями. Одно из них — уравнение движения центра масс C.11), другое — урав- нение моментов в Д-системе E.24): mdVc/d^ = F; dL/d^=Mc. E.26) Зная законы действующих внешних сил, точки их при- ложения и начальные условия, можно с помощью этих уравнений найти как скорость, так и положение каждой точки твердого тела в любой момент времени, т. е. пол- ностью решить задачу о движении тела. Однако, несмот- ря на кажущуюся простоту уравнений E.26), решение их в общем случае представляет собой весьма трудную задачу. И прежде всего это обусловлено тем обстоятель- ством, что связь между собственным моментом импульса L и скоростями отдельных точек твердого тела в /^-сис- теме оказывается сложной, за исключением немногих частных случаев. Мы не будем рассматривать эту зада- чу в общем виде (она решается в общей теории) и огра- ничимся в дальнейшем только отдельными частными случаями. Но прежде приведем некоторые соображения, пря- мо вытекающие из вида самих уравнений E.26). Если мы будем переносить силы вдоль направления их дей- ствия, то ясно, что не изменяется ни их результирующая F, ни их суммарный момент Мс. При этом уравнения E.26) тоже не изменятся, а следовательно, не изменится и движение твердого тела. Поэтому точки приложе- ния внешних сил можно переносить вдоль направле- ния действия сил — прием, которым постоянно поль- зуются. Равнодействующая сила. В тех случаях, когда сум- марный момент всех внешних сил оказывается перпен- дикулярным результирующей силе, т. е. JVLLF, все внеш- ние силы могут быть сведены к одной силе F, действую- щей вдоль определенной прямой. В самом деле, если относительнб некоторой точки О суммарный момент 148
M_LF, то всегда можно найти такой вектор rO-LM (рис. 5.14), что при заданных М и F M=[r0F]. При этом выбор г0 неоднозначен: прибавление к нему любого вектора г, параллельного F, не изменит послед- него равенства. А это означает, что данное равенство определяет не точку «приложения» силы F, а линию ее действия. Зная модули М я F соответствующих векто- ров, можно найти плечо / силы F (рис. 5.14): l=M/F. Рис. 5.14 Рис. 5.15 Таким образом, если JVLLF, систему сил, действующих на отдельные точки твердого тела, можно заменить од- ной равнодействующей силой — силой, которая равна результирующей F и создает момент, равный сум- марному моменту М всех внешних сил. Таков, в частности, случай однородного силового по- ля, например поля тяжести, в котором действующая на каждую частицу сила имеет вид Fi = mlg. В этом случае суммарный момент сил тяжести относительно любой точки О равен Стоящая в круглых скобках сумма, согласно C.8), рав- на тгс, где т — масса тела, г с— радиус-вектор его цент- ра масс относительно точки О. Поэтому Это значит, что равнодействующая mg сил тяжести про- ходит через центр масс тела. Обычно говорят, что рав- 149
нодействующая сил тяжести «приложена» к центру масс тела или к его центру тяжести. Ясно, что момент этой силы относительно центра масс тела равен нулю. Условия равновесия твердого тела. Тело будет оста- ваться в состоянии покоя, если нет причин, вызывающих его движение. Согласно уравнениям E.26), для этого не- обходимо и достаточно выполнение двух условий: 1) результирующая всех внешних сил, приложенных к телу, должна быть равной нулю, т. е. р—vp о 1 — Лг/внеш —и' 2) суммарный момент внешних сил относительно лю- бой точки тоже должен быть равен нулю, т. е. Эти условия должны быть выполнены в той системе отсчета, где тело покоится. Если система отсчета неинер- циальная, то кроме внешних сил взаимодействия надо учитывать и силы инерции. Это же относится и к мо- ментам сил. Теперь перейдем к рассмотрению четырех частных случаев движения твердого тела: 1) вращение вокруг неподвижной оси, 2) плоское движение, 3) вращение во- круг свободных осей, 4) особый случай движения тела с одной неподвижной точкой (гироскопы). 1. Вращение вокруг неподвижной оси. Найдем сначала выражение для момента импульса твер- дого тела относительно оси вращения 00' (рис. 5.15). Воспользовавшись формулой E.9), запишем где mi и pi — масса и расстояние от оси вращения i-й частицы твердого тела, coz — его угловая скорость. Обо- значив величину, стоящую в круглых скобках, через /, получим E.27) где / — так называемый момент инерции твердого тела относительно оси ОО'\ 150
Момент инерции твердого тела зависит, как нетрудно ви- деть, от распределения масс относительно интересующей нас оси и является величиной аддитивной. Вычисление момента инерции тела проводится по фор- муле где dm и dV— масса и объем элемента тела, находяще- гося на расстоянии г от интересующей нас оси z; р — плотность тела в данной точке. Моменты инерции некоторых однородных твердых тел относительно оси zc, проходящей через центр масс тела, приведены в следующей таблице (здесь т — мас- са тела): Твердое тело Тонкий стержень длины Сплошной цилиндр ра- диуса R Тонкий диск радуса R Шар радиуса R Ось zq Перпендикулярна стержню Совпадает с осью ци- линдра Совпадает с диаметром диска Проходит через центр шара Момент инерции 'Am/?2 4imR2 VbtnR* Вычисление момента инерции твердого тела произ- вольной формы относительно той ли иной оси представ- ляет собой, вообще говоря, довольно кропотливую в ма- тематическом отношении задачу. Однако в некоторых случаях нахождение момента инерции значительно уп- рощается, если воспользоваться теоремой Штей- не р а: момент инерции I относительно произвольной оси z ра- вен моменту инерции 1с относительно оси Zc, параллель- ной данной и проходящей через центр масс С тела, плюс произведение массы m тела на квадрат расстояния а между осями: ' = /с + та*. E.29) Доказательство этой теоремы приведено в приложении 3. Таким образом, если известен момент инерции 1с, то нахождение момента инерции / элементарно. Например, 151
момент инерции тонкого стержня (массы т и длины /) относительно оси, перпендикулярной стержню и прохо- дящей через его конец, равен Г 1 /9 1 I l \2 1 /9 /= ml2-\-m{— = — ml2. 12 т \ 2 / з Уравнение динамики вращения твердого тела (ось вращения неподвижна). Это уравнение легко получить как следствие E.15), если продифференцировать E.27) по времени, тогда E.30) где Mz — суммарный момент всех внешних сил относи- тельно оси вращения. Из этого уравнения, в частности, видно, что момент инерции / определяет инерционные свойства твердого тела при вращении: при одном и том же значении момента сил Mz тело с большим моментом инерции приобретает меньшее угловое ускорение pz. Напомним, что моменты сил относительно оси — вели- чины алгебраические: их знаки зависят как от выбора положительного направления оси z (совпадающей с осью вращения), так и от направления «вращения» соот- ветствующего момента силы. Например, выбрав положи- тельное направление оси г, как показано на рис. 5.16, мы тем самым задаем и положительное направление отсче- та угла ф (оба эти направления связаны правилом правого винта). Далее, если некоторый момент Ми «вращает» в поло- жительном направлении угла ф, то этот момент считается положительным, и на- оборот. А знак суммарного момента М2 в свою очередь определяет знак |3Z— Рис. 5.16 проекции вектора углового ускорения на ось z. Интегрирование уравнения E.30) с учетом начальных условий — значений o)oz и фо в начальный момент време- ни— позволяет полностью решить задачу о вращении твердого тела вокруг неподвижной оси, т. е. найти зави- симость от времени угловой скорости и угла поворота, (t) Иф@- Заметим, что уравнение E.30) справедливо в любой системе отсчета, жестко связанной с осью вращения. Однако если система отсчета неинерциальная, то момент сил Mz включает в себя не только моменты сил взаимо- действия с другими телами, но и моменты сил инерции. 152
Кинетическая энергия вращающегося твердого тела (ось вращения неподвижна). Имея в виду, что скорость 1-й частицы вращающегося твердого тела и, = ргсо, запи- шем или, короче, E.31) где / — момент инерции тела относительно оси враще- ния, со — его угловая скорость. Пример. Диск / (рис 5 17) вращается вокруг гладкой верткаль ной оси с угловой скоростью i»i На него падает диск 2, вращающийся с угловой скоростью о>2 Вследствие трения между ними оба диска через некоторое время на чинают вращаться как единое целое Най дем приращение кинетической энергии вра щения этой системы, если моменты инер ции дисков относительно оси вращения равны соответственно 1\ и /2 Сначала найдем установившуюся уг ловую скорость вращения Из закона со хранения момента импульса системы отно сительно оси z следует, что /iC0iz+^2W2z = откуда (О Рис 5 17 Заметим что ш1г, м2г и м2 — величины алгебраические Если окажет- ся, что coz>O, то это значит, что соответствующий вектор <я совпада ет с положительным направлением оси г, и наоборот Приращение кинетической энергии вращения этой системы ДГ : (Л- ¦ h WL Заменив coz его выражением A), голучим Л /п ДГ = — ¦ B) Знак минус показывает, что кинетическая энергия системы умень- шается Обратим внимание на то, что полученные результаты A) и B) весьма похожи и по форме, и по смыслу на случай абсолютно не упругого столкновения (см § 4 6). Работа внешних сил при вращении твердого тела во- круг неподвижной оси. В соответствии с уравнением 153
D.49) работа всех внешних сил, действующих на твер- дое тело, равна приращению только кинетической энер- гии тела, так как его собственная потенциальная энер- гия при этом не меняется. Таким образом, 8A = dT или, согласно E.31), 6Л = d (/со2/2). Так как ось z совпадает 2 2 ( с осью вращения, то со2=со22 и Но согласно E.30), /dcoz=Mzd^. Подставив это выраже- ние в последнее уравнение для 8А и учтя, что co2d^=d(p, получим &A=Mzd<?. E.32) Работа бЛ — величина алгебраическая: если Mz и dcp имеют одинаковые знаки, то бЛ>0; если же их знаки противоположны, то бЛ<0. Работа внешних сил при повороте твердого тела на конечный угол ф равна E.33) В случае, если Mz=const, последнее выражение упроща- ется: A=Mzq>. Таким образом, работа внешних сил при вращении твердого тела вокруг неподвижной оси определяется действием момента Mz этих сил относительно данной оси. Если силы таковы, что их момент Mz=0, то работы они не производят. 2. Плоское движение твердого тела (см. с. 21). При плоском движении центр масс С твердого те- ла движется в определенной плоскости, неподвижной в данной ^-системе отсчета, а вектор его угловой скорости и все время остается перпендикулярным этой плоскости. Последнее означает, что в Д-системе твердое тело совер- шает чисто вращательное движение вокруг неподвижной в этой системе оси, проходящей через центр масс тела. Вращательное же движение твердого тела определяет- ся уравнением E.30), которое, как было отмечено, спра- ведливо в любой системе отсчета. Таким образом, мы имеем следующие два уравнения, описывающие плоское движение твердого тела: 154
mac=F; E.34) Рис. 5.18 где т —¦ масса тела, F — результирующая всех внешних сил, 1с и Мег — момент инерции и суммарный момент всех внешних сил — оба относительно оси, проходящей через центр масс тела. При этом следует помнить, что момент МСг включает в себя только внешние силы взаимодействия, несмотря на то что Д-система в общем случае является неинерци- альной. Это связано с тем, что суммарный момент сил инерции равен нулю как от- носительно центра масс, так и относительно любой оси, проходящей через эту точку. Поэтому его можно просто не учитывать (см. с. 147). Заметим также, что угло- вое ускорение pz, а следова- тельно Q)z и ф одинаковы в обеих системах отсчета, так как Д-система движется по- ступательно относительно инерциальной ^-системы от- счета. Интегрируя уравнения E.34) с учетом начальных условий, можно найти зависимости rc{t) и q>(t), опреде- ляющие положение твердого тела в любой момент t. При решени задачи о движении несвободного твердо- го тела необходимо использовать еще одно, дополнитель- ное, условие, определяющее ограничения движения име- ющимися связями. Оно дает кинематическую связь меж- ду линейным и угловым ускорениями. Пример. Однородный цилиндр массы m и радиуса г скатывается без скольжения по наклонной плоскости, составляющей угол а с го- ризонтом (рис. 5.18). Найдем уравнения движения цилиндра. Стандартный подход к решению подобных задач со- стоит в следующем. Прежде всего устанавливают силы, действующие на данное тело, и точки их приложения (в нашем случае это mg — сила тяжести, R — нормальная составляющая силы реакции со сто- роны наклонной плоскости и FTp — сила трения покоя). Затем выби- рают положительные направления оси х и угла поворота ср (лучше всего эти направления взять сразу согласованными, так чтобы знаки ускорений аСх и fiz были одинаковы), например, как показано на рис. 5.18 справа. И только после этого записывают уравнения дви- жения E.34) в проекциях на выбранные таким образом положи- тельные направления лиф: = mg since — j 155
Кроме того, условие отсутствия скольжения определяет еще кинема- тическую связь между ускорениями: Совместное решение этих трех уравнений дает возможность найти ускорения ас иР , а также силу FTp. Кинетическая энергия при плоском движении твердо- го тела. Пусть тело совершает плоское движение в неко- торой инерциальной /(-системе отсчета. Чтобы найти его кинетическую энергию Т в этой системе, воспользуемся формулой D.56). Входящая в эту формулу величина Т в данном случае представляет собой кинетическую энер- гию вращения тела в Д-системе вокруг оси, проходящей через центр масс тела. Согласно E.31), Г = /ссо2/2, по- этому сразу можно записать Т - E.35) где 1с — момент инерции тела относительно оси враще- ния, проходящей через его центр масс, со — угловая ско- рость тела, m — его масса, Ус — скорость центра масс тела в К'-системе отсчета. Таким образом, кинетическая энергия твердого тела при плоском движе- нии складывается из энергии вращения в Ц-системе и энергии, связанной с движением центра масс. 3. Свободные оси. Главные оси тела. Ес- ли твердое тело привести во вращение и затем предоста- вить самому себе, то направление оси вращения в про- странстве, вообще говоря, будет меняться. Для того что- бы произвольная ось вращения тела сохраняла свое на- правление неизменным, к ней необходимо приложить определенные силы. Рассмотрим этот вопрос более подробно на следую- щем примере. Пусть середина С однородного стержня жестко скреплена с осью вращения так, что угол между стержнем и осью равен ¦& (рис. 5.19). Найдем момент М внешних сил, которые необходимо приложить к оси вра- щения, чтобы при вращении стержня с угловой скоро- стью и ее направление не менялось. Согласно E.12), этот момент M=dL/d/. Таким образом, чтобы опреде- лить М, сначала надо найти момент импульса стержня L, а затем его производную по времени. Момент импульса L проще всего определить относи- тельно точки С. Мысленно выделим элемент стержня 156
массы 8т, находящейся на расстоянии г от точки С. Его момент импульса относительно этой точки 6L= = [r, Smv], где v — скорость элемента. Легко видеть, что вектор 6L направлен перпендикулярно стержню (рис. 5.19), причем его направление не зависит от выбора элемента Ьт. Поэтому суммарный мо- мент импульса L стержня совпадает по направлению с вектором 6L. Заметим, что в данном случае вектор L не совпадает по направлению с векто- ром о)! При вращении стержня вектор L будет также вра- щаться с угловой скоростью и. За промежуток времени dt вектор L получает прира- щение dh, модуль которого, как видно из рис. 5.19, равен |dL| = Z, sin (л/2 —0) со d/, или в векторном виде dL=[o)L]d^. Поделив обе части по- следнего выражения на d^, получим Рис. 5.19 Таким образом, действительно, для удержания оси вра- щения в неизменном направлении к ней необходимо в данном случае приложить момент М некоторых внешних сил F (они показаны на рис. 5.19). Однако нетрудно ви- деть, что если ¦&=л/2, то вектор L совпадает по направ- лению с вектором и, и в этом случае Ме=0, т. е. направ- ление оси вращения будет оставаться неизменным без внешнего воздействия. Ось вращения, направление которой в пространстве остается неизменным без действия на нее каких-либо сил извне, называют свободной осью тела. В общей теории доказывается, что для любого твер- дого тела существуют три взаимно перпендикулярные оси, проходящие через его центр масс, которые могут служить свободными осями. Их называют главными осями тела. Нахождение главных осей тела произвольной фор- мы— в математическом отношении сложная задача. Од- нако она очень упрощается для тел, обладающих той или 157
иной симметрией, ибо положение центра масс и направ- ление главных осей обладают в этом случае той же сим- метрией. Например, у однородного прямоугольного параллеле- пипеда главные оси проходят через центры противопо- ложных граней. Если тело обладает осью симметрии (однородный цилиндр, конус и Др.), одной из его глав- ных осей является ось симметрии, в качестве же осталь- ных осей могут служить две любые взаимно перпендику- лярные оси, проходящие через центр масс тела и пер- пендикулярные его оси симметрии. Таким образом, у те- ла с осевой симметрией фиксирована только одна из главных осей. У тела же с центральной симметрией (на- пример, у однородного шара) главными осями являются три любые взаимно перпендикулярные оси, проходящие через центр тела,— ни одна из главных осей не фикси- рована относительно тела. Важной особенностью главных осей является то, что при вращении тела вокруг любой из них его момент им- пульса L совпадает по направлению с угловой скоростью и и определяется как L = /g), E.36) где / — момент инерции тела относительно данной глав- ной оси *. Причем L не зависит от выбора точки, относи- тельно которой его определяют (здесь предполагается, что ось вращения неподвижна). Наиболее просто убедиться в справедливости E.36) можно для случая однородного тела с осевой симметри- ей. Действительно, согласно E.27), момент импульса твердого тела относительно оси вращения ?2=/сог (на- помним, что Lz — это проекция вектора L, определенно- го относительно любой точки на этой оси). Но если тело симметрично относительно оси вращения, то из сообра- жения симметрии сразу следует, что вектор L совпада- ет по направлению с вектором и и, значит, Ь=/ю. Еще раз подчеркнем, что в общем случае (ось вра- щения не совпадает ни с одной из главных осей, хотя и проходит через центр масс тела) направление вектора L не совпадает с вектором и и связь между этими векто- рами носит сложный характер. Это обстоятельство явля- * Заметим, что соотношение E.36) справедливо и относительно осей, параллельных главным осям тела и не проходящих через его центр масс. 158
и ется причиной сложного поведения вращающихся твер- дых тел. 4. Гироскопы. Гироскопом называют массивное симметричное тело, вращающееся с большой угловой скоростью вокруг своей оси симметрии. Рассмотрим по- ведение гироскопа на примере волчка. Опыт показыва- ет, что если ось вращающегося волчка наклонена к вер- тикали, то волчок не падает, а совершает так называе- мое п р ецесси он н ое дви- жение (прецессию) — его ось описывает конус вокруг вертикали с некоторой угло- вой скоростью а/, причем оказывается: чем больше уг- ловая скорость со вращения волчка, тем меньше угловая скорость прецессии а/. Такое поведение волчка- гироскопа можно легко объ- яснить с помощью уравнения моментов E.12), если толь- ко принять , что со>-©' (это условие, кстати, поясняет, что имеется в виду под боль- шой угловой скоростью ги- роскопа). Действительно, мо- мент импульса L прецессиру- ющего волчка относительно точки опоры О (рис. 5.20) можно представить в виде суммы момента импульса Ьш, обусловленного вращением волчка вокруг своей оси, и некоторого добавочного момента импульса I/, вызванно- го прецессией волчка вокруг вертикальной оси, т. е. //////////77777/77/. Рис. 5.20 Поскольку ось волчка совпадает с одной из его главных осей, то, согласно E.36), Ьш=/(о, где / — момент инер- ции волчка относительно этой оси. Кроме того, ясно, что чем меньше угловая скорость прецессии, тем меньше и соответствующий момент U. При со^>со' во всех практи- чески интересных случаях La^>L', поэтому результирую- щий момент импульса L почти совпадает с Lm как по ве- личине, так и по направлению, — можно считать, что 159
Зная же поведение вектора L, мы тем самым найдем и характер движения оси волчка-гироскопа. Но поведением вектора L управляет уравнение мо- ментов E.12). Согласно ему, момент импульса L отно- сительно точки О (рис. 5.20) получает за время dt при- ращение E.37) совпадающее по направлению с вектором М — моментом внешних сил относительно той же точки О (в данном случае это момент силы тяжести tng). Из рис. 5.20 вид- но, что dL_LL. В результате вектор L (а следовательно, и ось волчка) будет поворачиваться вместе с вектором М вокруг вертикали, описывая круговой конус с углом полураствора Ф. Волчок-гироскоп будет прецессировать вокруг вертикальной оси с некоторой угловой скоро- стью й/. Найдем связь между векторами М, L и ш'. Согласно рисунку, модуль приращения вектора L за время cU есть ]dL| =Lsin#-d/cU, или в векторном виде dL=[(fl'L]cM. После подстановки этого выражения в E.37) получим [©' L] = M. E.38) Из этого уравнения видно, что момент силы М определя- ет угловую скорость прецессии о/ (а не ускорение!). По- этому мгновенное устранение момента М приводит к мгновенному исчезновению и прецессии. В этом отноше- нии можно сказать, что прецессия не обладает инер- цией. Заметим, что момент сил М, действующий на гиро- скоп, может иметь любую природу. Для обеспечения ре- гулярной прецессии (постоянной угловой скорости ш') важно только, чтобы вектор М, не меняясь по модулю, поворачивался вместе с осью гироскопа. Пример. Найдем угловую скорость прецессии наклонного волчка массы пг, вращающегося с большой угловой скоростью со вокруг своей оси симметрии, относительно которой момент инерции волчка равен /. Центр инерции волчка находится на расстоянии I от точки опоры. Согласно E.38), со'/со sin О=mgl sin О, где О — угол между вер- тикалью и осью волчка (рис. 5.20). Отсюда ш' = mgl/I w. Интересно, что величина со' не зависит от угла наклона О оси волч- ка. Кроме того, полученный результат показывает, что со' обратно пропорциональна со, т. е., действительно, чем больше угловая ско- рость волчка, тем меньше угловая скорость его прецессии. 160
Гироскопический момент. Рассмотрим теперь эффект, возникающий при вынужденном вращении оси гироско- па. Пусть, например, ось гироскопа укреплена в U-об- разной подставке, которую мы будем поворачивать во- круг оси 00', как показано на рис. 5.21. Если момент импульса L гироскопа направлен вправо, то при таком повороте за время dt вектор L получит приращение dL — вектор, направленный за плоскость рисунка. Согласно E.37), это означает, что на гироскоп действует момент сил М, совпадающий по направлению с вектором dL. Рис. 5 21 Рис 5 22 Момент М обусловлен возникновением пары сил F, дей- ствующих на ось гироскопа со стороны подставки. Ось же гироскопа в соответствии с третьим законом Нью- тона будет действовать на подставку с силами F' (рис. 5.21). Эти силы называют гироскопически- ми; они создают гироскопический момент М'=—М. Заметим, что в данном случае гироскоп не об- ладает способностью противодействовать изменению на- правления его оси вращения. Появление гироскопических сил называют гироско- пическим эффектом. Подобный гироскопический эффект, связанный с возникновением гироскопического давления на подшипники, наблюдается, например, у ро- торов турбин на кораблях при поворотах и качке, у вин- товых самолетов при виражах и т. п. 161
Проследим действие гироскопического момента на примере гиро- скопа, ось которого вместе с рамкой (рис. 5 22) может свободно по- ворачиваться вокруг горизонтальной оси 00' U-образной подставки. Если подставке сообщить вынужденное вращение вокруг вертикаль- ной оси, как показано на рисунке вектором <о', то момент импульса L гироскопа получит за время dt приращение dLi — вектор, направ- ленный за рисунок. Это приращение обусловлено моментом Mi пары сил, действующих на ось гироскопа со стороны рамки. Гироскопиче- ские силы, действующие со стороны оси гироскопа на рамку, вызовут поворот последней вокруг горизонтальной оси 00' При этом вектор L получит дополнительное приращение dLa, которое, в свою очередь, обусловлено моментом Мг пары сил, действующих на ось гироскопа со стороны рамки В результате ось гироскопа будет поворачиваться так, что вектор L будет стремиться совпасть по направлению с век- тором (о'. Таким образом, за промежуток времени dt момент импульса L гироскопа получает приращение dL=dL1 + dL2== (Mi + N\.2)dt. При этом на рамку действует гироскопический момент М'= — (Mi + M2). Составляющая этого момента М/=—Mi вызывает поворот рамки вокруг горизонтальной оси 00', другая составляющая М/=—М2 противодействует повороту всей системы вокруг вертикальной оси (в отличие от предыдущего случая). Гироскопический эффект лежит в основе разнообраз- ных применений гироскопов: гирокомпас, гироскопиче- ский успокоитель качки кораблей, гироскопический ста- билизатор и др. Задачи ф 5.1 Законы сохранения момента импульса и энергии. Дока- зать, что полная механическая энергия Е планеты, движущейся во- круг Солнца по эллипсу, зависит только от его большой полуоси а. Найти выражение для Е, если известны массы планеты и Солнца (ш и М), а также большая полуось а эллипса Решение Воспользуемся законами сохранения момента им- пульса и энергии Точка, относительно которой момент импульса планеты сохраняется, — это центр Солнца Поэтому для положений 1 и 2 планеты (рис. 5 23), в которых вектор скорости перпендикуля- рен радиусу-вектору, можно записать (О Из закона сохранения полной энергии ? следует, что для тех же положений планеты mv\ тм mv2 тм ___,_»_ _Y__. B) Решив совместно уравнения A) и B), выразим, например, V\ через г( и гг 2 2уМ л2 162
И наконец, находим формулу для полной энергии Е как Е = Т («О + U (/"О = - Y mMI(rx + г2). Учитывая, что Л1+Л2 = 2а, получим окончательно Е = — у тМ/2а. ф 5.2. Частица / массы пц налетает на частицу 2 массы т^ имея вдали от частицы 2 кинетическую энергию Го и прицельный параметр / ¦— плечо вектора импульса относительно частицы 2 (рис. 5.24). Заряд каждой частицы равен q. Найти наименьшее расстояние, на которое сблизятся частицы, если: 1) mi«Cm2; 2) т\ сравнимо с mi. Решение. 1. Условие т!<Ст2 означает, что частица 2 в про- цессе взаимодействия будет практически оставаться в покое. Вектор силы, действующей на частицу 1, все время проходит через точку, в Рис. 5.24 которой находится частица 2. Поэтому момент импульса частицы 1 относительно покоящейся частицы 2 сохраняется. Отсюда 1 Ро ~ гмт Р. где слева — момент импульса частицы 1 вдали от частицы 2, а спра- ва— в момент наибольшего сближения, когда r_Lp (рис. 5.24). Далее, из закона сохранения энергии следует где Т — кинетическая энергия частицы 1 в момент наибольшего сбли- жения. Решив эти два уравнения (с учетом связи р и Т), получим 27-п A) 2. В данном случае уже нельзя считать, что частица 2 покоится в процессе взаимодействия. Решение наиболее целесообразно про- вести в //-системе, где картина «соударения» выглядит так, как по- казано на рис. 5.25. Система из двух частиц предполагается замкну- той, поэтому ее собственный момент импульса сохраняется: 1 Pw = гып*Ръ B) где учтено, что в момент наибольшего сближения ПгХр. Кроме того, на основании закона сохранения энергии T0=T+kq4rMUH, C) 163
где То и Т — суммарные кинетические энергии частиц в Д-системе, когда частицы находятся далеко друг от друга, и в момент наиболь- шего сближения. Из уравнений B) и C) получим то же выражение A), только в нем вместо То будет стоять То, причем в данном слу- чае (частица 2 первоначально покоилась), согласно D.61), о —' ¦ щ 'То и выражение дтя гм Заметим, что при mi<Cm-2 величина То будет полностью совпадать с A). ф 5.3. Небольшой шарик подвесили к точке О на легкой нерас- тяжимой нити длиной / Затем шарик отвели в сторону так, что нить отклонилась на угол О от вертикали, и сообщили ему начальную ско- Рис. 5 25 Рис. 5.26 рость v0 перпендикулярно вертикальной плоскости, в которой распо- ложена нить При каком значении vo максимальный угол отклонения нити от вертикали окажется равным я/2? Решение. На шарик в процессе движения действуют две силы — сила тяжести и сила натяжения со стороны нити. Нетрудно видеть, что относительно вертикальной оси z, проходящей через точ- ку О, момент этих сил Afz = O. Следовательно, относительно данной оси момент импульса шарика Lr = const, или /sin 9-mw0 = /mv, A) где m — масса шарика, v — его скорость в положении, при котором нить составляет прямой угол с вертикалью Шарих движется в поле тяжести Земли под действием сторон- ней силы — силы натяжения со стороны нити Эта сила все время перпендикулярна вектору скорости шарика и поэтому работы не совершает. Отсюда следует, что согласно уравнению D.31) механи- ческая энергия шарика в поле тяжести Земли сохраняется: ;os 9, B) где правая часть равенства соответствует горизонтальному положе- нию нити. 164
Решив совместно уравнения A) и B), получим - ф 5.4. На жестком проволочном полукольце радиуса Го, которое может свободно вращаться вокруг вертикальной оси АВ (рис. 5.26), находятся две одинаковые небольшие муфточки. Их соединили нитью и установили в положение 1—/. Затем всей установке сообщили угловую скорость coo и, предоставив ее самой себе, пережгли нить в точке А. Считая, что масса установки практически сосредоточена в муфточках, найти ее угловую скорость в момент, когда муфточки соскользнут (без трения) в крайнее нижнее положение 2—2. г - / т Рис. Ь.27 i А" =>' Риг г Р . 5.28 Решение. Пусть в нижнем положении расстояние муфточек от оси вращения г и угловая скорость установки со. Тогда из законов сохранения энергии и момента импульса относительно оси вращения следует, что r2M2_r2M2 =2gh, r2<o = rl<*0, где h — разность высот верхнего и нижнего положений муфточек. Здесь учтено, что в нижнем положении, как и в верхнем, скорость муфточек относительно проволочного полукольца равна нулю. Кроме того, из рис. 5.26 видно, что г\ = г2 + Д2. Решив совместно эти три уравнения, получим # 5.5. Гладкий стержень свободно вращается в горизонтальной плоскости с угловой скоростью соо вокруг неподвижной вертикальной оси О (рис. 5.27), относительно которой его момент инерции равен /. На стержне около оси вращения находится небольшая муфта массы т, соединенная с этой осью нитью. После пережигания нити муфта начинает скользить вдоль стержня. Найти скорость v' муфты отно- сительно стержня в зависимости от ее расстояния г до оси вращения. 165
Решение. У данной системы в процессе движения сохраня- ются кинетическая энергия и момент импульса относительно оси вра- щения. Отсюда следует, что /а>0 = (/ + тг2) о, где у2 = у'2 + со2г2 (рис. 5.27). Из этих уравнений получим v' = ш0 г//1 + mryi . ф 5.6. Горизонтально летевшая пуля А попала, застряв, в верти- кальный однородный стержень массы т и длины 10, верхний конец, которого укреплен в шарнире О (рис. 5.28). Пуля имела импульс р и попала в стержень на расстоянии / от точки О. Пренебрегая ее массой, найти: 1) приращение импульса системы пуля — стержень за время дви- жения пули в стержне; 2) угловую скорость, которую приобретет стержень, с учетом собственного момента импульса пули, равного L и совпадающего по направлению с вектором р (пуля вращается вокруг направления ее движения). Решение. 1. Система пуля — стержень незамкнутая: помимо сил, уравновешивающих друг друга, в процессе движения пули в стержне возникает горизонтальная составляющая силы реакции в точке О со стороны оси. Действие этой составляющей и вызовет при- ращение импульса системы: где Vc — скорость центра стержня после застревания пули. Так как все внешние силы в этом процессе проходят через точку О, то за время движения пули в стержне момент импульса системы будет оставаться постоянным относительно любой оси, проходящей через эту точку. Взяв ось перпендикулярной к плоскости рисунка, за- пишем 1р = /со , где / — момент инерции стержня относительно выбранной оси, а а» — угловая скорость стержня непосредственно после остановки пули в нем. Из этих двух уравнений с учетом того, что Ус = иг, г—расстоя- ние от точки О дв центра стержня, получим Отсюда видно, что знак приращения Ар зависит от отношения III». В частности, при /До=2/3 величина Др=0, т. е. импульс системы не изменяется за время движения пули в стержне. Это значит, что в данном случае горизонтальная составляющая силы реакции в точке О отсутствует. 2. В этом случае момент импульса системы относительно точки О также будет оставаться постоянным за время движения пули в стержне, поэтому, согласно E.23), Слева записан момент импульса пули относительно точки О, а спра- ва— момент импульса стержня (с пулей) непосредственно после 166
остановки пули в стержне (см. рис. 5.29, где все три вектора распо- ложены в горизонтальной плоскости). Найдем вектор L, когда стержень (с пулей) приобретет угло- вую скорость to. Возьмем малый элемент стержня массы dm, нахо- дящийся на расстоянии г от точки О. Его момент импульса относи» тельно точки О равен dL = [r, dm v] = dm-ri to = (m to//0) r2 dr, где v — скорость данного элемента. Проинтегрировав это выражение по всем элементам, получим L = i/з ot/q to. а Рис. 5.29 Рис. 5.30 Таким образом, Из этой формулы, согласно рис. 5.29, получим 3 С помощью того же рисунка можно найти и направление вектора « (угол а). % 5.7. Динамика вращательного движения. Однородный сплош- ной цилиндр массы т0 и радиуса R может без трения вращаться во- круг неподвижной горизонтальной оси О (рис. 5.30). На цилиндр в один ряд плотно намотан тонкий нерастяжимый шнур длины / и мас- сы т. Найти угловое ускорение цилиндра в зависимости от длины х свешивающейся части шнура. Считать, что скольжения нет и центр масс намотанной части шнура находится на оси цилиндра. Решение. Воспользуемся уравнением моментов E.15) относи- тельно оси О. Для этого найдем момент импульса системы относи- тельно данной оси, Z-j, и соответствующий момент сил Мг. Момент импульса 161
= (mo/2 где учтено, что момент инерции цилиндра I=m<,R2/2 и v = a>zR (от- сутствие скольжения шнура). Момент внешних сил тяжести относи- тельно оси О Продифференцировав Lz по времени и подставив &Lzl&t и Мг в урав- нение моментов, получим z lR(mo + 2m) ф 5.8. На гладкой горизонтальной плоскости лежит однородный диск радиуса га. На него осторожно опустили другой такой же диск, предварительно сообщив ему угловую скорость со0. Через сколько времени оба диска будут вращаться с одной и той же угловой ско- ростью, если коэффициент трения между дисками равен к? Решение. Сначала найдем установившуюся угловую скорость вращения со. Из закона сохранения момента импульса следует, что /dig = 2 /(О, где / — момент инерции каждого диска относительно общей оси вра- щения. Отсюда И = Wg/2. Теперь рассмотрим поведение одного из дисков, например нижнего. Его угловая скорость меняется под действием момента М сил тре- ния. Вычислим М. Для этого выделим на верхней поверхности диска элементарное кольцо с радиусами г, г+йг. Момент сил трения, дей- ствующих на данное кольцо, равен Ш = гк (mg/я г\) 2яг Ar = 2k (mg/rl) r^ Ar, где т — масса каждого диска. Проинтегрировав это выражение по г от 0 до Го, получим Согласно уравнению E.30), приращение угловой скорости нижнего диска на величину dco происходит за время At = (I/M) dca = Cro/4*g) d<». Интегрируя это уравнение по со от 0 до соо/2, находим, что искомое время t =3/s rQv>o!kg. # 5.9. Плоское движение твердого тела. Однородный цилиндр находится на горизонтальной доске (рис. 5.31). Коэффициент трения между ними равен к. Доске сообщили ускорение а в горизонтальном направлении перпендикулярно оси цилиндра. Найти: 1) ускорение 168
оси цилиндра в отсутствие скольжения; 2) предельное значение апр, при котором скольжение еще отсутствует. Решение. 1. Выбрав положительные направления «nip, как показано на рис. 5.31, запишем уравнение движения оси цилиндра и уравнение моментов в Д-системе относительно этой оси: где т и I — масса и момент инерции цилиндра относительно его оси, г — радиус цилиндра. Кроме того, условие отсутствия скольжения цилиндра дает кинематическую связь ускорений: а— аг = \ Из этих уравнений находим ас = а/3. Рис. 5.31 Рис. 5.32 2. Определим из предыдущих уравнений значение силы трения FTp, обеспечивающей качение цилиндра без скольжения: FTV = ma/3 Максимально возможное значение этой силы равно kmg. Отсюда ф5.10. Однородный шар радиуса г начинает скатываться без скольжения с вершины сферы радиуса R (рис. 5.32). Найти угловую скорость ш шара после отрыва от поверхности сферы. Решение. Прежде всего заметим, что угловая скорость шара после отрыва от поверхности сферы изменяться не будет. Поэтому задача сводится к нахождению ее значения в момент отрыва. Запишем уравнение движения центра шара в момент отрыва: mv2/(R + r) =mg cos &, где v — скорость центра шара в момент отрыва, О — соответствую- щий угол (рис. 5.32). Скорость v можно найти из закона сохранения энергии: где / — момент инерции шара относительно оси, проходящей через его центр. Кроме того, v = tar, h = (R + л)A — cos»). Из этих четырех уравнений получим (Я + Г)/Г2 . 169
# 5.11. Конический маятник. Тонкий однородный стержень массы т и длины / вращается с постоянной угловой скоростью со вокруг вертикальной оси, проходящей через его точку подвеса О (рис. 5.33). При этом стержень описывает коническую поверхность с некоторым углом полураствора О. Найти угол О, а также модуль и направление силы реакции R в точке О. Решение. Рассмотрим систему отсчета, вращающуюся вокруг вертикальной оси вместе со стержнем. В этой системе на стержень действует кроме силы тяжести mg и силы реакции R еще центро- бежная сила инерции F4j. Так как стер- жень покоится в данной системе отсче- та, т. е. находится в состоянии равно- весия, то это значит, что результирую- щий момент всех сил относительно лю- бой точки и результирующая всех сил равны нулю. Относительно точки О момент соз- дают только сила тяжести и центро- бежные силы инерции. Из равенства моментов этих сил следует = Л*иб- A) тд Вычислим ЛГцб. Элементарный момент сил инерции, который действует на элемент стержня их, находящийся на расстоянии х от точки О, равен i = (m ufi/l) sin » cos »je2,djc. Проинтегрировав это выражение по всей длине стержня, получим B) Из A) и B) следует, что ccs» = 3?/2(*2/. C) Найдем модуль и направление вектора R. В системе отсчета, где стержень вращается с угловой скоростью со, его центр масс (точка С) движется по горизонтальной окружности. Поэтому из уравнения движения центра масс C.11) сразу следует, что вертикальная со- ставляющая вектора R есть Я, — mg, а горизонтальная составляю- щая R±определяется уравнением man=Rx, где ап—нормальное ускорение центра масс С. Отсюда D) Модуль вектора R есть / = 1/2 т ш2 ' 'о>2 1J , а его направление — угол О' между вектором R и вертикалью — определяется формулой cos %'=mglR. Интересно, что 'Ь'ф'Ь, т. е. 170
вектор R не совпадает по направлению со стержнем В этом легко убедиться, выразив cos <K через cos О: Отсюда видно, что cos O'>cos #, т. е. д'<д. Это и показано на рис. 5.33. Заметим, что равнодействующая сил инерции Таб проходит не через точку С, а ниже. Действительно, /7Цб==Лх и определяется фор- мулой D), а результирующий момент Мпв—формулой B). Из этих формул следует, "что плечо вектора FU6 относительно точки О равно 2/s/cosO (рис 5.33). # 5.12. Гироскоп. Волчок массы т, ось которого составляет угол Ь с вер- тикалью, прецессирует вокруг верти- кальной оси, проходящей через точку опоры О. Момент импульса волчка ра- вен L, расстояние от его центра масс до точки О есть /. Найти модуль и направ- ление вектора F — горизонтальной со- ставляющей силы реакции в точке О Решение. Согласно E38), угло- вая скорость прецессии Рис. 5.34 Центр масс волчка движется по окруж- ности; следовательно, вектор F направ- лен так, как показано на рис. 5 34 {этот вектор поворачивается вместе с осью волчка). Из уравнения движения центра масс C.11) получаем В результате F = Заметим, что если бы точка опоры волчка находилась на абсо- лютно гладкой плоскости, то волчок прецессировал бы с той же уг- ловой скоростью, но вокруг вертикальной оси, проходящей через центр масс волчка — точку С на рис. 5.34.
Часть II РЕЛЯТИВИСТСКАЯ МЕХАНИКА Глава 6 КИНЕМАТИКА СПЕЦИАЛЬНОЙ ТЕОРИИ ОТНОСИТЕЛЬНОСТИ § 6.1. Трудности дорелятивистской физики Специальная теория относительности, созданная Эйн- штейном в 1905 г., означала пересмотр всех представле- ний классической физики и главным образом представ- лений о свойствах пространства и времени. Поэтому дан- ная теория по своему основному содержанию может быть названа физическим учением о пространстве и време- ни. Физическим потому, что свойства пространства и времени в этой теории рассматриваются в теснейшей свя- зи с законами совершающихся в них физических явле- ний. Термин «специальная» подчеркивает то обстоятель- ство, что эта теория рассматривает явления только в инерциальных системах отсчета. Мы начнем этот раздел с краткого обзора дореляти- вистской физики и остановимся на истоках тех трудно- стей, которые привели к появлению теории относитель- ности. Основные представления дорелятивистской физики. Напомним сначала те представления о пространстве и времени, которые связаны с законами Ньютона, т. е. ле- жат в основе ньютоновской механики. 1. Пространство, имеющее три измерения, подчиня- ется евклидовой геометрии. 2. Наряду с трехмерным пространством существует независимое от него время (независимое в том смысле, в каком три измерения пространства не зависят друг от друга). Но вместе с тем время связано с пространством законами движения. Действительно, время измеряют ча- сами, в принципе представляющими собой любой при- бор, в котором используется тот или иной периодический процесс, дающий масштаб времени. Поэтому определить 172
время безотносительно к какому-либо периодическому процессу, т. е. вне связи с движением, невозможно. 3. Размеры твердых тел (масштабы) и промежутки времени между данными событиями одинаковы в разных системах отсчета. Это соответствует ньютоновской кон- цепции абсолютности пространства и времени, согласно которой их свойства считаются не зависящими от систе- мы отсчета — пространство и время одинаковы для всех систем отсчета. 4. Признается справедливость закона инерции Гали- лея — Ньютона, согласно ко- торому тело, не подвержен- ное действию со стороны других тел, движется прямо- у линейно и равномерно. Этот закон утверждает существо- вание инерциальных систем отсчета, в которых выполня- п ются законы Ньютона (а также принцип относитель- ности Галилея). 5. Из этих представлений вытекают преобразования Галилея, выражающие прост- ранственно-временную связь любого события в разных инерциальщлх системах отсчета. Если /('-система отсчета движется относительно /(-системы со скоростью V (рис. 6.1) и начало отсчета времени соответствует момен- ту, когда начала координат О' и О обеих систем совпа- Рис. 6.1 дают, то у' = у; t'=i Отсюда следует, что координаты любого события отно- сительны, т. е. имеют разные значения в разных систе- мах отсчета; момент же времени, когда событие про- изошло, одинаков в разных системах. Последнее означа- ет, что время течет одинаковым образом в разных системах отсчета. Это обстоятельство казалось столь очевидным, что даже не оговаривалось как специальный постулат. Из F.1) непосредственно вытекает закон преобразо- вания (сложения) скоростей: * Здесь и в дальнейшем мы ограничимся только двумя прост- ранственными координатами: х и у. Координата z ведет себя во всех отношениях так же, как у. 173--
v' = v-V, F.2) где v' и v—-скорости точки (частицы) в К'- и /(-систе- мах отсчета. 6. Выполняется принцип относительности Галилея: асе инерциальные системы отсчета эквивалентны друг другу в механическом отношении, все законы механики одинаковы в этих системах отсчета, или, другими слова- ми, инвариантны относительно преобразований Галилея. 7. Соблюдается принцип дальнодействия: взаимодей- ствия тел распространяются мгновенно, т. е. с бесконеч- но большой скоростью. Эти представления ньютоновской механики вполне соответствовали всей совокупности экспериментальных данных, имевшихся в то время (заметим, впрочем, что эти данные относились к изучению движения тел со ско- ростями, значительно меньшими скорости света). В их пользу говорило и весьма успешное развитие самой ме- ханики. Поэтому представления ньютоновской механики о свойствах пространства и времени стали считаться на- столько фундаментальными, что никаких сомнений в их истинности ни у кого не возникало. Первому испытанию подвергся принцип относитель- ности Галилея, который, как известно, касался только механики — единственного раздела физики, достигшего к тому времени достаточного развития. По мере разви- тия других разделов физики, в частности оптики и элект- родинамики, возник естественный вопрос: распространя- ется ли принцип относительности и на другие явления? Если нет, то с помощью этих (немеханических) явлений можно в принципе различить инерциальные системы от- счета и в свою очередь поставить вопрос о существова- нии главной, или абсолютной, системы отсчета. Одно из таких явлений, которое, как ожидали, по- разному протекает в разных системах отсчета, — это рас- пространение света. Согласно господствовавшей в то вре- мя волновой теории, световые волны должны распрост- раняться с определенной скоростью по отношению к не- которой гипотетической среде («светоносному эфиру»), о природе которой, правда, не было единого мнения. Но какова бы ни была природа этой среды, она не может, конечно, покоиться во всех инерциальных системах сра- зу. Тем самым выделяется одна из инерциальных сис- тем— абсолютная — та самая, которая неподвижна от- носительно «светоносного эфира». Полагали, что в 474
этой — и только этой — системе отсчета свет распростра- няется с одинаковой скоростью с во всех направлениях. Если некоторая инерциальная система отсчета движется по отношению к эфиру со скоростью V, то в этой сис- теме отсчета скорость света с' должна подчиняться обыч- ному закону сложения скоростей F.2), т. е. с'=с—V. Это предположение оказалось возможным проверить на опыте, который и был осуществлен Майкельсоном (совместно с Морли). Опыт Майкельсона. Цель этого эксперимента заключа- лась в том, чтобы обнаружить «истинное» движение Земли от- носительно эфира. Бытго ис- пользовано движение Земли по ее орбите со скоростью 30 км/с. Идея эксперимента состояла в следующем. Свет от источника S (рис. В.2) посылался в двух взаимно перпендикулярных направле- ниях, отражался от зеркал А и В, находящихся на одинаковом расстоянии / от источника S, и возвращался в точку S. В этом опыте сравнивалось время прохождения светом обоих путей: SAS и SBS. Предположим, что установка вместе с Землей дви- жется так, что ее скорость V относительно эфира направ- лена вдоль SA (в момент проведения опыта). Если ско- рость света подчиняется обычному закону сложения ско- ростей F.2), то на пути SA скорость света относительно установки (Земли) равна с—V, а на обратном пути c+V. Тогда время прохождения пути SAS равно / . / 2/ I ±~ c2 — V~^ c + V~ с 1 — (V/сУ- На пути же SBS скорость света относительно установ- ки равна с'=Ус1— V2 (рис. 6.2), и время прохожде- ния этого пути 21 11 1 Рис. 6.2 Из сравнения выражений для t л и t± видно, что свет должен проходить оба пути за разное время. Измерив 175
разность времен ti~t±, можно определить скорость установки (Земли) относительно эфира. Несмотря на то что ожидаемая разность времен бы- ла чрезвычайно мала, установка была достаточно чувст- вительной, чтобы эту разность надежно обнаружить (это достигалось с помощью очень чувствительного интерфе- ренционного метода). Тем не менее результат опыта оказался отрицатель- ным: разность времен не была обнаружена. Конечно, слу- чайно могло оказаться, что в момент проведения опыта Земля покоилась относительно эфира. Но тогда через полгода, например, скорость Земли относительно эфира достигла бы 60 км/с. Однако повторение опыта через полгода по-прежнему не дало ожидаемого результата. Более точные опыты того же рода, поставленные позд- нее, также подтвердили первоначальный результат. Отрицательный результат опыта Майкельсона про- тиворечил тому, что ожидалось на основании преобразо- ваний Галилея (преобразования скоростей). Он показал также, что нельзя обнаружить движение относительно эфира, что скорость света не зависит от движения источ- ника света (ведь источник движется по-разному относи- тельно эфира в разные времена года). В пользу того, что скорость света не зависит от ско- рости источника, говорят и некоторые астрономические наблюдения (например, над двойными звездами), а так- же другие опыты, поставленные позднее специально с целью проверки этого факта. К началу XX в. в теоретической и экспериментальной физике сложилась своеобразная ситуация. С одной сторо- ны, теоретически были предсказаны различные эффекты, выделяющие из множества инерциальных систем глав- ную (абсолютную). С другой стороны, настойчивые по- пытки обнаружить эти эффекты на опыте неизменно оканчивались неудачей. Опыт неуклонно подтверждал справедливость принципа относительности для всех яв- лений, включая и те, к которым теория считала его за- ведомо неприемлемым. Был сделан целый ряд попыток объяснения отрица- тельного результата опыта Майкельсона и аналогичных ему в рамках ньютоновской механики. Однако все они оказались в конечном счете неудовлетворительными. Кар- динальное решение этой проблемы было дано лишь в теории относительности Эйнштейна. 176
§ 6.2. Постулаты Эйнштейна Глубокий анализ всего экспериментального и теоре- тического материала, имеющегося к началу XX в., при- вел Эйнштейна к пересмотру исходных положений клас- сической физики, прежде всего представлений о свойст- вах пространства и времени. В результате им была соз- дана специальная теория относительности, явившаяся ло- гическим завершением всей классической физики. Эта теория принимает без изменения такие положе- ния ньютоновской механики, как евклидовость простран- ства и закон инерции Галилея — Ньютона. Что же каса- ется утверждения о неизменности размеров твердых тел и промежутков времени в разных системах отсчета, то Эйнштейн обратил внимание на то, что эти представле- ния возникли в результате изучения движений тел с ма- лыми скоростями, поэтому их экстраполяция в область больших скоростей ничем не оправдана, а следователь- но незаконна. Только опыт может дать ответ на вопрос, каковы их истинные свойства. Это же относится к пре- образованиям Галилея и к принципу дальнодействия. В качестве исходных позиций специальной теории от- носительности Эйнштейн принял два постулата, или принципа, в пользу которых говорит весь эксперимен- тальный материал (и в первую очередь опыт Майкельсо- на): 1) принцип относительности, 2) независимость скорости света от скорости источ- ника. Первый постулат представляет собой обобще- ние принципа относительности Галилея на любые физи- ческие процессы: все физические явления протекают одинаковым обра- зом во всех инерциальных системах отсчета; все законы природы и уравнения, их описывающие, инвариантны, т. е. не меняются, при переходе от одной инерциальной системы отсчета к другой. Другими словами, все инерциальные системы отсче- та эквивалентны (неразличимы) по своим физическим свойствам; никаким опытом нельзя в принципе выделить ни одну из них как предпочтительную. Второй постулат утверждает, что скорость света в вакууме не зависит от движения источ- ника света и одинакова во всех направлениях. 177
Это значит, что скорость света в вакууме одинакова во всех инерциальных системах отсчета. Таким образом, скорость света занимает особое положение в природе. В отличие от всех других скоростей, меняющихся при переходе от одной системы отсчета к другой, скорость света в пустоте является инвариантной величиной. Как мы увидим, наличие такой скорости существенно изменя- ет представления о пространстве и времени. Из постулатов Эйнштейна следует также, что скорость света в вакууме является предельной: никакой сигнал, никакое воздействие одного тела на другое не могут рас- пространяться со скоростью, превышающей скорость света в вакууме. Именно предельный характер этой ско- рости и объясняет одинаковость скорости света во всех системах отсчета. В самом деле, согласно принципу от- носительности, законы природы должны быть одинаковы во всех инерциальных системах отсчета. Тот факт, что скорость любого сигнала не может превышать предель- ное значение, есть также закон природы. Следовательно, значение предельной скорости — скорости света в ваку- уме— должно быть одинаково во всех инерциальных системах отсчета: в противном случае эти системы мож- но было бы отличить друг от друга. В частности, наличие предельной скорости автомати- чески предполагает ограничение скорости движения час- тиц величиной с. Иначе эти частицы могли бы осуществ- лять передачу сигналов (или взаимодействий между те- лами) со скоростью, превышающей предельную. Таким образом, согласно постулатам Эйнштейна, значение всех возможных в природе скоростей движения тел и распро- странения взаимодействий ограничено величиной с. Этим самым отвергается принцип дальнодействия ньютонов- ской механики. Все содержание специальной теории относительности вытекает из этих двух ее постулатов. В настоящее время оба постулата Эйнштейна, как и все следствия из них, убедительно подтверждаются всей совокупностью накоп- ленного экспериментального материала. Синхронизация часов. Прежде чем делать какие-либо выводы из этих постулатов, Эйнштейн тщательно про- анализировал представления о способах измерения про- странства и времени. В первую очередь он обратил вни- мание на то, что физической реальностью обладает не точка пространства и не момент времени, когда что-ли- бо произошло, а только само событие. Для описания 178
события в данной системе отсчета нужно указать место, в котором оно происходит, и момент времени, когда оно происходит. Положение точки, в которой происходит событие, мо- жет быть определено с помощью жестких масштабов ме- тодами евклидовой геометрии и выражено в декартовых координатах. Ньютоновская механика в этом отношении пользовалась вполне реальными приемами сравнения из- меряемых величин с образцовыми эталонами. Соответствующий же момент времени можно опреде- лить с помощью часов, помещенных в ту точку системы отсчета, где происходит данное событие. Однако такое определение уже не является удовлетворительным, ког- да нам надо сопоставить друг с другом события, проис- ходящие в различных местах, или, что то же самое, срав- нить время для событий, происходящих в местах, уда- ленных от часов. Действительно, чтобы сравнить время (показания ча- сов) в различных точках системы отсчета, прежде всего необходимо установить способ, как определить общее для всех точек системы отсчета время. Другими слова- ми, надо обеспечить синхронный ход всех часов данной системы отсчета. Ясно, что синхронизировать часы, помещенные в раз- личные точки системы отсчета, можно только с помощью каких-нибудь сигналов. Наиболее быстрые сигналы, при- годные для этой цели, — это световые или радиосигналы, распространяющиеся с известной скоростью с. Выбор именно этих сигналов обусловлен еще и тем, что их ско- рость не зависит от направления в пространстве, а также одинакова во всех инерциальных системах отсчета. Далее можно поступить следующим образом. Наблю- датель, находящийся, например, в начале координат О данной системы отсчета, сообщает по радио: «Передаем сигнал точного времени. Сейчас по моим часам время to». В момент, когда этот сигнал достигнет часов, нахо- дящихся на известном расстоянии г от точки О, их уста- навливают так, чтобы они показывали время t=to+f/c, т. е. с учетом времени запаздывания сигнала. Повторе- ние сигнала через определенные промежутки времени даст возможность каждому наблюдателю установить син- хронный ход его часов с часами в точке О. В результате такой операции можно утверждать, что все часы данной системы отсчета показывают в каждый момент одно и то же общее время. 179
Существенно отметить, что определенное таким обра- зом время относится лишь к той системе отсчета, отно- сительно которой синхронизированные часы покоятся. Соотношения между событиями. Обратимся к вопро- су о пространственных и временных соотношениях меж- ду данными событиями в разных инерциальных системах отсчета. Уже в ньютоновской механике пространственные со- отношения между различными событиями зависят от то- го, к какой системе отсчета они относятся. Например, две последовательные вспышки лампочки в движущемся по- . . езде происходят в одной и А 0 в V той же точке системы от- 1_ I ~~~** счета, связанной с поез- ^^ттттттттттштштшттш^тт д0М) но в разных точках системы отсчета, связан- Рис. 6.3 ной с полотном дороги. Утверждение, что два раз- новременных события происходят в одном и том же месте или на таком-то расстоянии друг от друга, приобретает смысл только тогда, когда указано, к какой системе от- счета это утверждение относится. В противоположность этому временные соотношения между событиями в ньютоновской механике считаются не зависящими от системы отсчета. Это значит, что если какие-нибудь два события происходят одновременно в одной системе отсчета, то они являются одновременны- ми и во всех других системах отсчета. Вообще промежу- ток времени между двумя данными событиями считается одинаковым во всех системах отсчета. Легко, однако, убедиться, что в действительности это не так — одновременность (а следовательно, и течение времени) является понятием относительным, приобрета- ющим смысл только тогда, когда указано, к какой систе- ме отсчета это понятие относится. Покажем с помощью простого рассуждения, что два события, одновременные в одной системе отсчета, в другой системе отсчета ока- зываются неодновременными. Представим себе стержень АВ, движущийся с посто- янной скоростью V относительно /(-системы отсчета. В середине стержня находится лампочка О, по концам — в точках А и В — фотоэлементы (рис. 6.3). Пусть в не- который момент лампочка О дала кратковременную вспышку света. Поскольку скорость распространения света в системе отсчета, связанной со стержнем (как и 180
во всякой инерциальной системе отсчета), равна с в обо- их направлениях, то световые импульсы достигнут рав- ноудаленных от О фотоэлементов Л и В в один и тот же момент времени (в системе отсчета «стержень») и оба фотоэлемента сработают"одновременно. Иначе обстоит дело в /С-системе. В этой системе от- счета скорость световых импульсов в обоих направлени- ях также равна с, однако проходимые ими пути различ- ны. Действительно, пока световые импульсы идут к точ- кам А и В, последние переместятся вправо (рис. 6.3) и, следовательно, фотоэлемент А сработает раньше, чем фо- тоэлемент В. Таким образом, события, одновременные в одной си- стеме отсчета, не являются одновременными в другой системе отсчета, т. е. одновременность в отличие от пред- ставлений ньютоновской механики является понятием от- носительным. А это в свою очередь означает, что время в разных системах отсчета течет неодинаково. Если бы в нашем распоряжении имелись мгновенно распространяющиеся сигналы, то события, одновремен- ные в одной системе отсчета, были бы одновременными и в любой другой системе. Это непосредственно следует из только что рассмотренного примера. В этом случае течение времени не зависело бы от системы отсчета и можно было бы говорить об абсолютном времени, кото- рое фигурирует в преобразованиях Галилея. Таким об- разом, преобразования Галилея, по существу, исходят из предположения, что синхронизация часов осуществляет- ся с помощью мгновенно распространяющихся сигналов. Однако таких сигналов в действительности нет. § 6.3. Замедление времени и сокращение длины В этом параграфе мы рассмотрим три важнейших следствия, которые вытекают из постулатов Эйнштей- на,— это равенство поперечных размеров движущихся тел в разных системах отсчета, замедление хода движу- щихся часов и сокращение продольных размеров дви- жущихся тел, а затем (в § 6.4) обобщим полученные ре- зультаты в виде соответствующих формул преобразова- ния координат и времени. Приступая к решению этих вопросов, напомним преж- де всего, что под системой отсчета подразумевается те- ло отсчета, с которым связаны координатная сетка и ряд неподвижных одинаковых часов, синхронизированных 181
между собой. Далее, предполагается, что во всех инер- циальных системах отсчета координатные сетки и часы проградуированы одинаковым образом. Ясно, что это можно осуществить только с помощью эталонов длины и времени, реализованных также одинаковым образом во всех системах отсчета. Для этого достаточно использовать какой-либо при- родный периодический процесс, дающий естественный масштаб как длины, так и времени, например одну из монохроматических волн, испускаемых определенными атомами, неподвижными в данной системе отсчета. Тогда в этой системе отсчета эталоном длины можно взять дли- ну волны, а эталоном времени — соответствующий пе- риод колебания. С помощью этих эталонов можно по- строить эталон один метр как определенное число дан- ных длин волн и эталон одна секунда как тоже опреде- ленное число периодов данных колебаний (заметим, что в настоящее время так и сделано). Аналогичную операцию можно проделать в каждой инерциальной системе отсчета, используя одну и ту же монохроматическую волну одних и тех же атомов, непо- движных в каждой из этих систем отсчета. Основанием для этого служит то, что, по принципу относительности, физические свойства покоящихся атомов не зависят от того, в какой инерциальной системе отсчета они поко- ятся. Реализовав в каждой системе отсчета эталоны длины. и времени, можно перейти к решению такого фундамен- тального вопроса, как сравнение этих эталонов в разных системах отсчета, или, другими словами, к сравнению размеров тел и течения времени в этих системах. Равенство поперечных размеров тел. Начнем с вопро- са о сравнении поперечных размеров тел в разных инер- циальных системах отсчета. Представим себе две инерци- альные системы отсчета К и К', оси ц и ц' которых па- раллельны друг другу и перпендикулярны направлению движения одной системы относительно другой (рис. 6.4), причем начало отсчета О' /('-системы движется по пря- мой, проходящей через начало отсчета О /(-системы. Установим вдоль осей у и у' стержни ОА и О'А', являю- щиеся эталонами метра в каждой из этих систем отсче- та. Представим себе далее, что в момент совпадения осей у' и у верхний конец левого стержня сделает мет- ку на оси у /(-системы. Совпадет ли эта метка с точкой А — верхним концом правого стержня? 182
Принцип относительности позволяет сразу ответить на этот вопрос: да, совпадет. Если бы это было не так, то с точки зрения обеих систем отсчета один из стержней оказался бы, например, короче другого и, следовательно, имелась бы возможность экспериментально отличить од- ну из инерциальных систем отсчета от другой по более коротким поперечным размерам. Однако это противоре- чит принципу относительности. Отсюда следует, что поперечные размеры тел одина- ковы во всех инерциальных системах отсчета. Это озна- чает также, что при указанном выборе начал отсчета К'- и /(-систем координаты у' и у любой точки или события совпадают, т. е. У'=У. F.3) Это соотношение представляет собой одно из искомых преобразований координат. ,\V Vut/Z О о х,х' Рис 6.4 \ \ z \ А А' А" Рис 6 5 Замедление времени. Наша следующая задача — сравнить течение времени в разных инерциальных систе- мах отсчета. Как уже говорилось, время измеряется ча- сами, причем под часами имеется в виду любой прибор, в котором используется тот или иной периодический про- цесс. Поэтому в теории относительности принято обычно говорить о сравнении хода идентичных часов в разных инерциальных системах отсчета. Наиболее просто этот вопрос можно решить с помо- щью следующего мысленного (т. е. в принципе возмож- ного) эксперимента. Возьмем так называемые свето- 183
вые часы — стержень с зеркалами на обоих концах, между которыми «бегает» короткий световой импульс. Период таких часов равен интервалу времени между дву- мя последовательными моментами, когда световой им- пульс достигает какого-то определенного конца стержня. Далее, представим себе две инерциальные системы от- счета К' и К, движущиеся относительно друг друга со скоростью V. Пусть световые часы АВ неподвижны в /('-системе и ориентированы перпендикулярно направле- нию ее движения относительно ^-системы (рис. 6.5). Про- следим теперь за «ходом» этих часов в обеих системах отсчета: К' и К. В /('-системе часы неподвижны, и их период где / — расстояние между зеркалами, с — скорость све- та. В /(-системе, относительно которой часы движутся, расстояние между зеркалами также /, ибо поперечные размеры тел одинаковы в разных инерциальных системах отсчета. Однако путь светового импульса в этой системе отсчета будет уже иным — зигзагообразным (рис. 6.5): пока световой импульс распространяется от нижнего зеркала к верхнему, последнее переместится на некото- рое расстояние вправо и т. д. Поэтому световой импульс, чтобы вернуться к нижнему зеркалу, проходит в /(-сис- теме больший путь, причем с той же скоростью с. Зна- чит, свету понадобится на это больше времени — больше, чем когда часы неподвижны. Другими словами, период движущихся часов удлинится —с точки зрения /(-систе- мы отсчета они будут идти медленнее. Обозначим период движущихся часов через At в /(-системе. Из прямоугольного треугольника АВ'А' (рис. 6.5) следует, что 12+ (VMI2J= (cAt/2J, откуда Д/=B//с)ДгЬ А так как 2//с=Д^0, то F.4) где р= V/c, V — скорость часов в /(-системе. Отсюда видно, что At>At0, т. е. одни и те же часы в разных инерциальных системах отсчета идут по-раз- ному: в той системе отсчета, относительно которой часы 184
движутся, они идут медленнее, чем в системе отсчета, где они покоятся. Другими словами, движущиеся часы идут медленнее, чем покоящиеся. Это явление называют з а- медлением времени. Время, отсчитываемое по часам, движущимся вместе с телом, в котором происходит какой-либо процесс, на- зывают собственным временем этого тела. Его обозначают ДА)- Как следует из F.4), собственное вре- мя самое короткое. Время Д^ того же процесса в другой системе отсчета зависит от скорости V этой системы от- носительно тела, в котором происходит процесс. Эта за- висимость особенно сильно проявляется для значений скорости V, сравнимых со скоростью света (рис. 6.6). Пример. Часы движутся в К-системе отсчета прямолинейно и равномерно со скоростью v. J3-VJ° В начальный момент ?=0 их по- казания совпадали с часами Х-си- Рис. 6.6 стемы. На сколько секунд отста- нут движущиеся часы за время ^ = 60 мин (это время по часам К-системы), если: a) v = 1800 км/ч (реактивный самолет); б) у = = 4/s с, где с — скорость света в вакууме? Пусть в момент t по часам iC-системы движущиеся часы показы- вали t0, причем, •согласно F.4), to=t V\—(v/cJ> тогда искомое время ? 3,0 2,0 1,0 и г .— f / ) 0,1 0,2 ЯЛ 0,4 Oj OS 0,7 ff,8 0,9 /,0 *_*„ = *(!-у 1_(»/сJ). а) При у<Сс, согласно формуле бинома Ньютона, У — (»/сJ я 1 — 1/2 (i>/eJ и t — to= i/2 (w/cJ* =5-10-9 с. б) <—/о=2/б t=24 мин. Таким образом, в отличие от ньютоновской механики течение времени в действительности зависит от состоя- ния движения. Не существует единого мирового време- ни, и понятие «промежуток времени между двумя дан- ными событиями» оказывается относительным. Утверж- дение, что между двумя данными событиями прошло столько-то секунд, приобретает смысл только тогда, ког- да указано, к какой системе отсчета это утверждение от- носится. 185
Абсолютное время ньютоновской механики является в теории относительности приближенным понятием, спра- ведливым только при малых (по сравнению со скоростью света) относительных скоростях систем отсчета. Это сра- зу следует из F.4) и видно из рис. 6.6: при V<^c А/~ДА). Итак, мы пришли к фундаментальному выводу: время в системе отсчета, движущейся с часами, течет медлен- нее (для наблюдателя, относительно которого данные ча- сы движутся). Это же относится и ко всем процессам, протекающим в движущихся относительно наблюдателя системах отсчета. Естественно, возникает вопрос: заметит ли наблюда- тель в /('-системе, движущейся относительно ^-системы, что его часы идут медленнее, чем часы /(-системы? Нет, не заметит. Это сразу же следует из принципа относи- тельности. Если бы К'-наблюдатель тоже обнаружил за- медление времени в своей системе отсчета, то это озна- чало бы, что для обоих наблюдателей—К.' и К — время течет медленнее в одной из инерциальных систем отсче- та. Из этого они заключили бы, что одна из инерциаль- ных систем отсчета отличается от другой — в противоре- чии с принципом относительности. Отсюда следует, что эффект замедления времени яв- ляется взаимным, симметричным относительно обеих инерциальных систем отсчета К и К'. Иначе говоря, если с точки зрения /С-системы медленнее идут часы К'-сис- темы, то с точки зрения ^'-системы, наоборот, медленнее идут часы /С-системы (причем в том же отношении). Это обстоятельство указывает на то, что явление замедления времени является чисто кинематическим. Оно представ- ляет собой обязательное следствие инвариантности ско- рости света и никак не может быть приписано какому- либо изменению в свойствах часов, обусловленному их движением. Формула F.4) сразу же нашла экспериментальное подтвержде- ние, объяснив «загадочное» на первый взгляд поведение мюонов при прохождении земной атмосферы. Мюоны — это нестабильные части- цы, которые самопроизвольно распадаются в среднем через 2 -10 а с (это время измерено в условиях, когда они неподвижны или дви- жутся с малыми скоростями). Мюоны образуются в верхних слоях атмосферы на высоте 20—30 км. Если бы время жизни мюонов не зависело от их скорости, то, двигаясь даже со скоростью света, они не смогли бы проходить путь больше чем сМ = 3-108 м/с X 2-10-6 с = 600 м. Однако наблюдения показывают, что значительное число мюонов все-таки достигает земной поверхности. 18В
Это объясняв] ся тем, что время 2 • 10~в с — это собственное вре- мя (Д^о), жизни мюонов, т е время по часам, движущимся вместе с мюонами Время же по земным часам должно быть, согласно F 4), гораздо больше (скорость этих частиц близка к скорости света) и оказывается достаточным, чтобы мюоны могли достигнуть поверхно- сти Земли. В заключение несколько слов о так называемом «п а- радоксе часов», или «парадоксе близнецов». Пусть имеются двое одинаковых часов Л и В, из которых часы А неподвижны в некоторой инерциальной системе отсче- та, а часы В сначала удаляются от часов А и затем воз- вращаются к ним. Предполагается, что в начальный мо- мент, когда часы находились вместе, они показывали од- но и то же время. С «точки зрения» часов А движущимися являются ча- сы В, поэтому они идут медленнее и по возвращении от- станут от часов А. С «точки же зрения» часов В, наобо- рот, движутся часы А, поэтому по возвращении отстанут именно они. Явное противоречие — в этом суть «пара- докса». В действительности в этих рассуждениях допущена принципиальная ошибка. Эта ошибка касается рассуж- дения с «точки зрения» часов В, ибо система отсчета, связанная с этими часами, является неинерциальной (она сначала удаляется с ускорением, а затем приближается), и мы не имеем права в данном случае использовать ре- зультаты, относящиеся только к инерциальным системам отсчета. Детальный расчет, выходящий за рамки спе- циальной теории относительности, показывает, что часы, движущиеся с ускорением (в нашем случае часы В), идут медленнее, поэтому при возвращении отстанут имен- но они. Лоренцево сокращение. Пусть стержень АВ движется относительно /(-системы отсчета с постоянной скоростью V (рис. 6.7) и длина стержня равна 10 в системе отсчета К', связанной со стержнем. Наша задача — определить длину / данного стержня в /(-системе. Проделаем для этого следующий мысленный экспери- мент. Сделаем на оси х /(-системы метку М и установим около нее часы. Зафиксируем по этим часам время про- лета Ato стержня мимо метки М. Тогда можно утверж- дать, что искомая длина стержня в /(-системе l = V Ый. Для наблюдателя, связанного со стержнем, время про- лета будет иным. Действительно, для него часы, показав- 187
шие пролетное время At0, движутся со скоростью V, а значит, показывают «чужое» время. «Свое» время проле- та At для этого наблюдателя будет, согласно F.4), боль- ше. Это время он может найти из соотношения Из этих двух уравнений, с учетом F.4) получим или F.5) Рис. 6.7 © где p=V/c. Длину 10, измеренную в системе отсчета, где стержень неподвижен, назы- вают собственной дли- ной. Таким образом, продоль- ный размер движущегося стержня оказывается меньше его собственной длины, т. е. /</о. Это явление называют лоренцев ым сокраще- нием. Заметим, что данное сокращение относится толь- ко к продольным размерам тел (размерам в направле- нии движения), поперечные 1,0 0,9 0,8 0,7 0,5 0,4- 0,3 0,2 0,1 О —— \ \ \ S \ \ 0,1 0,2 flJ 0A 0,5 0,6 0,7 0? 0J 1,3 Рис. 6.8 К О 1 5м Зм Рис. 6.9 же размеры, как было установлено, не меняются. Сравни- тельно с формой тела в системе отсчета, где оно покоит- ся, его форма в движущейся системе отсчета может ха- рактеризоваться как сплющенная в направлении дви- жения. Из формулы F.5) следует, что степень сокращения зависит от скорости V. Эта зависимость особенно суще- ственно проявляется для значений скорости V, сравнимых со скоростью света (рис. 6.8). 188
Рассмотрим несколько примеров, связанных с лорен- цевым сокращением. Пример 1. Стержень, собственная длина которого /0 = 5,0 м, дви- жется в продольном направлении со скоростью V относительно К- системы отсчета. При каком значении V длина стержня в /(-системе будет /=3,0 м (эта ситуация показана на рис. 6.9)? Чтобы наблюдать такое сокращение длины, скорость стержня, согласно F.5), должна быть V = c-^l — (//А)J = 4/5 с. Пример 2. Стержень А движется мимо неподвижного в /(-системе от- ' счета стержня В со скоростью v, как ^ -y—J? ' показано на рис. 6.10. Оба стержня g| , имеют одинаковую собственную дли- i ну 10. Найдем в /(-системе отсчета ° промежуток времени At между мо- рис g jq ментами совпадения левых и правых ' ' концов стержней. Длина движущегося в /(-системе стержня А равна 1= =*/0 У I — (v/с) , и с помощью рис. 6.10 нетрудно сообразить, что искомый промежуток времени Д* = (/„ - t)fV = (I - Vl - (t//CJ ) lo/V. Пример 3. Две частицы, двигавшиеся в Х-системе отсчета по одной прямой с одинаковой скоростью v=4/s с, попали в неподвиж- ную мишень с промежутком времени Д/=5-10~9 с (в данной систе- ме отсчета). Каким было собственное расстояние между частицами до попадания в мишень? Расстояние между частицами в /(-системе отсчета l=vAt. По- этому искомое расстояние, согласно формуле F.5), /0 = vM/yi — (w/cJ = 2 м. Итак, в разных инерциальных системах отсчета длина одного и того же стержня оказывается различной. Ины- ми словами, длина — понятие относительное, имеющее смысл только по отношению той или иной системы отсче- та. Утверждение, что длина тела столько-то метров, не имеет смысла, пока не указано, к какой именно системе отсчета отнесена эта величина. При малых же скоростях (V<^c), как следует из F.5) и видно из рис. 6.8, /«/0 и длина тела приобрета- ет практически абсолютный смысл. Необходимо отметить, что лоренцево сокращение, как и замедление времени, должно быть взаимным. Это зна- чит, что если мы будем сравнивать два движущихся от- носительно друг друга стержня, собственная длина ко- торых одинакова, то с «точки зрения» каждого из этих стержней длина другого стержня будет короче, причем в одинаковом отношении. Если бы это было не так, то имелась бы возможность экспериментально отличить 189
инерциальные системы отсчета, связанные с этими стер- жнями, что, однако, противоречит принципу относитель- ности. Это говорит о том, что лоренцево сокращение являет- ся также чисто кинематическим эффектом — в теле не возникает каких-либо напряжений, вызывающих дефор- мацию. Подчеркнем, что лоренцево сокращение тел в направ- лении их движения, равно как и замедление времени, представляет собой реальный и объективный факт, от- нюдь не связанный с какими-либо иллюзиями наблюда- теля. Все значения размеров данного тела или промежут- ков времени, полученные в разных системах отсчета, яв- ляются равноправными (все они «правильные»). Труд- ность понимания этих утверждений связана исключи- тельно с нашей привычкой, основанной на повседневном опыте, считать понятия длины и промежутков времени абсолютными понятиями, когда в действительности это не так. Понятия длины и промежутка времени столь же относительны, как понятия движения и покоя. § 6.4. Преобразования Лоренца Теперь нам предстоит решить фундаментальный во- прос о формулах преобразования координат и времени (имеются в виду формулы, связывающие координаты и моменты времени одного и того же события в разных инерциальных системах отсчета). Преобразования Галилея? Напомним, что эти преоб- разования основаны на предположениях, что длина тел не зависит от движения и время течет одинаково в раз- личных инерциальных системах отсчета. Однако в преды- дущем параграфе было показано, что в действительно- сти это не так: течение времени и длина тел зависят от системы отсчета — выводы, являющиеся неизбежным следствием постулатов Эйнштейна. Поэтому мы вынуж- дены отказаться от преобразований Галилея, или, говоря точнее, признать, что они — лишь частный случай каких- то более общих преобразований. Возникает задача отыскания таких формул преобра- зования, которые, во-первых, учитывали бы замедление времени и лоренцево сокращение (т. е. были бы в конеч- ном счете следствиями постулатов Эйнштейна), и, во-вто- рых, переходили бы в предельном случае малых скорос- 190
Тей в преобразования Галилея. Перейдем к решению этой задачи. Рассмотрим две инерциальные системы отсчета К и К'. Пусть /('-система движется относительно Л"-снстемы со скоростью V. Направим координатные оси обеих си- стем отсчета так, как показано на рис. 6.11: оси х и х' совпадают и направлены параллельно вектору V, а оси у и у' параллельны друг другу. Установим в разных точ- ках обеих систем отсчета одинаковые часы и синхрони- зируем их — отдельно часы K-системы и отдельно часы к /('-системы. И наконец, возь- мем за начало отсчета време- ни в обеих системах момент, когда начала координат О и О'совпадают (t=? = 0). Предположим теперь, что в момент времени t (в /(-сис- теме) в точке с координата- ми х, у произошло некоторое событие А, например вспых- нула лампочка. Наша задача — найти координаты х', у' и момент времени f этого события в Л"-системе. Вопрос относительно координаты у' был уже решен в начале предыдущего параграфа, где было показано [см. формулу F.3)], что у' = у. Поэтому сразу перейдем к нахождению координаты х' события. Координата х' характеризует собственную длину отрезка О'Р, непод- вижного в /('-системе (рис. 6.11). Длина же этого отрез- ка в /(-системе, где отсчет производится в момент I, рав- на х—Vt. Связь между этими длинами дается фор- мулой F.5), согласно которой x — Vt = x']/rl—р2.От- сюда О О' Р Рис. 6.11 -р. F.6) С другой стороны, координата х характеризует собст- венную длину отрезка ОР, неподвижного в /(-системе. Длина же этого отрезка в /('-системе, где измерение про- водится в момент ?, равна х'-\-У?. Учитывая опять F.5), получим = ^Kl—P2, откуда 'V 1-р. F.5') Полученные формулы позволяют также установить и связь между моментами времени t и ? события А в обеих 191
Системах отсчета. Для этого достаточно исключить из F.6) и F.6') х' или х, после чего найдем: t={t'4-х'V/c*)/VT=jK F.7) Формулы F.3), F.6), F.6') и F.7) называют пре- образованиями Лоренца. Они играют фунда- ментальную роль в теории относительности. По этим фор- мулам осуществляется преобразование координат и вре- мени любого события при переходе от одной инерциаль- ной системы отсчета к другой. Итак, преобразования Лоренца при переходе от К- к /('-системе имеют вид: F.8) а при обратном переходе от /('- к /(-системе — х' +Vt' , , V +х' V/c2 ,„ „. х = _ - ; у —у; t— ¦—, F.9) где fi = V/c, V — скорость /('-системы относительно /(-си- стемы. Следует сразу же обратить внимание на симметрию (одинаковый вид) формул F.8) и F.9),что является след- ствием полного равноправия обеих систем отсчета (раз- личный знак при V в этих формулах обусловлен лишь противоположным направлением движения систем К и К' относительно друг друга). Преобразования Лоренца сильно отличаются от пре- образований Галилея F.1), однако последние могут быть получены из F.8) и F.9), если в них формально поло- жить с = оо. Что это значит? В конце предыдущего параграфа было отмечено, что в основе преобразований Галилея лежит допущение о синхронизации часов с помощью мгновенно распростра- няющихся сигналов. Из этого обстоятельства вытекает, что величина с в преобразованиях Лоренца играет роль скорости тех сигналов, которые используют для синхро- низации часов. Если эта скорость бесконечно велика, то получаются преобразования Галилея; если же она рав- на скорости света, то — преобразования Лоренца. Таким образом, в основе преобразований Лоренца лежит до- 192
пущение о синхронизации часов с помощью световых си- гналов, имеющих предельную скорость. Замечательной особенностью преобразований Лорен- ца является то, что при V<Cc они переходят* в преобра- зования Галилея F.1). Таким образом, в предельном слу- чае V<Cc законы преобразования теории относительно- сти и ньютоновской механики совпадают. Это означает, что теория относительности не отвергает преобразований Галилея как неправильные,. но включает их в истинные законы преобразования как частный случай, справедли- вый при У<Сс. В дальнейшем мы увидим, что это отража- ет общую взаимосвязь между теорией относительности и ньютоновской механикой — законы и соотношения тео- рии относительности переходят в законы ньютоновской механики в предельном случае малых скоростей. Далее, из преобразований Лоренца видно, что при V>c подкоренные выражения становятся отрицательны- ми и формулы теряют физический смысл. Это соответст- вует тому факту, что движение тел со скоростью, боль- шей скорости света в вакууме, невозможно. Нельзя даже пользоваться системой отсчета, движущейся со скоростью V = c; при этом подкоренные выражения обращаются в нуль и формулы также теряют физический смысл. Это значит, что, например, с фотоном, движущимся со скоро- стью с, принципиально не может быть связана система отсчета. Или иначе: не существует такой системы отсче- та, в которой фотон был бы неподвижным. И наконец, необходимо обратить внимание на то, что в формулы преобразования времени входит пространст- венная координата. Это важное обстоятельство указыва- ет на неразрывную связь между пространством и време- нем. Другими словами, речь должна идти не отдельно о пространстве и времени, а о едином пространстве — вре- мени, в котором протекают все физические явления. § 6.5. Следствия из преобразований Лоренца Понятие одновременности. Пусть в К-системе отсчета происходят два каких-то события: Ах (хь уь tx) и М (х2, Уь h). Найдем интервал времени между этими со- * Строго говоря, необходимо еще, чтобы х/с<С^, т. е. чтобы вре- мена распространения световых сигналов на расстояния, фигурирую- щие в рассматриваемых задачах (х[с), были малы по сравнению с интересующими нас промежутками времени. При этом условии мож- но считать, что сигналы распространяются практически мгновенно. 193
бытиями в /('-системе, движущейся со скоростью V вдоль оси х, как показано на рис. 6.11. Согласно формуле пре- образования времени F.8), искомый интервал времени 5) А1 V V Рис. - U- 1 fill} и в , у тН Г f'__ {h-ti)-(x2-Xl)Vlct FЛ0) Отсюда следует, что события, одновременные в /(-систе- ме (*2 = *i), не одновременны в /('-системе (^'—^'#0). Исключением является случай, когда оба события проис- ходят в /(-системе а) одновременно в точ- g' a в ках с о д и н а ко в ы- " — ^ и значениями ко- ординаты х (коорди- ната у может быть различной). Итак, одно- временность — понятие отно- сительное: то, что одновременно в одной системе отсчета, в общем случае не одновремен- но в другой системе отсчета. Говоря об одновременности событий, необходимо указывать систему отсчета, относи- тельно которой эта одновременность имеет место. В про- тивном случае понятие одновременности теряет смысл и могут возникать разного рода недоразумения и «парадок- сы». Пример. «Парадокс» стержня и трубки. Сквозь неподвижную в Л'-системе отсчета трубку АВ длины 10 пролетает стержень А'В', соб- ственная длина которого равна 2U. Скорость стержня такова, что его длина в /С-системе равна длине трубки, 1=10 (рис. 6.12, а), и в не- который момент стержень, пролетая сквозь трубку, целиком в ней уместится. Однако «с точки зрения стержня» лоренцево сокращение вдвое претерпевает трубка (рис. 6.12, б), поэтому ясно, что стержень (длины 2/0) не поместится в трубке (длины /о/2). Есть ли здесь про- тиворечие? Противоречия нет, и вот почему. «С точки зрения трубки» концы пролетающего стержня совместятся с концами трубки одновременно. «С точки же зрения стержня» совпадения концов (А с А', В с В') произойдут не одновременно: сначала совпадут концы В и В' (рис. 6.12,6), а затем, через некоторый промежуток времени, концы А и Л'. Из относительности понятия одновременности следу- ет, что часы /С'-системы, расставленные вдоль оси х' и синхронизированные между собой в этой системе отсче- та, в /(-системе будут показыв 1ть разное время. В самом 194
Деле, возьмем для простоты момент, когда начала коор- динат О и О' обеих систем отсчета совпадают и часы в этих точках показывают одно время: t = t' = O. Тогда в /(-системе в точке с координатой х часы Л"-системы пока- зывают в этот момент время / = 0, часы же /('-системы в этой точке — иное время, V'. Действительно, согласно формуле преобразования времени F.8), Отсюда видно, что в момент /=0 (в /(-системе) часы К'- системы будут показывать разное время, зависящее от координаты х (так называемое местное время). Это показано на рис. 6.13, а. Относительно /С'-системы картина будет обратной (рис. 6.13, б), как и должно быть в соответствии с равноправием обеих инерциальных си- стем отсчета. Далее, из формулы F.10) видно, что для одновремен- ных в /(-системе событий знак разности t2'—// определя- ется знаком выражения —(х2—X\)V. Следовательно, в разных системах отсчета (при разных значениях скоро- сти V) разность t2'—1\ будет различной не только по модулю, но и по знаку. Последнее означает, что порядок событий Л1 и Ач может быть любым (как прямым, так и обратным). Сказанное, однако, не относится к причинно-связан- ным событиям. Порядок следования таких событий (при- чина->-следствие) будет одинаков во всех системах от- счета. В этом легко убедиться из следующего рассужде- ния. Рассмотрим, например, выстрел — событие A\{x\,t{) и попадание пули в мишень — событие А2(х2, t%), пред- полагая, что оба события происходят на оси х. В /(-сис- теме отсчета t%>t\, скорость пули v и пусть для опреде- 195
ЛенносТи *2>*i, причем ясно, 4to Х2—Xi — v(t2—t\). Пос- ле подстановки этого равенства в формулу F.10) полу- чим Величина, стоящая во второй круглой скобке числителя, всегда положительна в связи с тем, что V<c (даже при v = c, когда причинно-следственная связь обусловлена максимально возможной скоростью передачи сигналов или взаимодействий). Отсюда следует, что если t2>t\, то и t2/>ti/, т. е. порядок следования причинно-следствен- ных событий одинаков во всех инерциальных системах отсчета. Лоренцево сокращение. Расположим неподвижный в /('-системе стержень вдоль оси х', т. е. вдоль направления движения этой системы отсчета относительно /(-системы. Пусть длина стержня в /('-системе 1й = х2—х\ (собствен- ная длина). В /(-системе, относительно которой стержень движет- ся, его длину определяют как расстояние I между коорди- натами х2 и х\ его концов, взятыми в один и тот же мо- мент {t2=t\). Воспользовавшись преобразованиями Ло- ренца F.8) для координат х' и х, запишем откуда /=/oj/TH*2. F.11) Таким образом, длина I движущегося стержня оказыва- ется меньше его собственной длины /0, и в разных инер- циальных системах отсчета она будет иметь свое значе- ние. Этот результат полностью согласуется с получен- ным ранее F.5). Из определения длины следует, что относительность длины данного стержня является следствием относитель- ности понятия одновременности. Это же относится и к форме любого тела — его размеры в направлении движе- ния также различны в разных инерциальных системах отсчета. Длительность процессов. Пусть в точке с координатой х' /('-системы отсчета протекает некоторый процесс, дли- тельность которого в этой системе A^o=^/—W (собст- венное время процесса). Найдем длительность данного 196
процесса &t = t2—t\ в /(-системе, относительно которой /('-система движется. Воспользуемся с этой целью преобразованиями Ло- ренца для времени. Так как процесс происходит в точке с фиксированной координатой х' /('-системы, то наиболее удобно использовать формулы F.9): или Д^=Д^0/>/Т^|2. F.12) Отсюда видно, что длительность одного и того же про- цесса различна в разных инерциальных системах отсчета. В /(-системе его длительность больше (At>Ato), а следо- вательно, в этой системе отсчета он протекает медленнее, чем в /('-системе. Это вполне согласуется с результатом, относящимся к ходу одних и тех же часов в разных инер- циальных системах отсчета, — формулой F.4). Интервал. Относительный характер пространственных и временных промежутков отнюдь не означает, что тео- рия относительности вообще отрицает существование ка- ких бы то ни было абсолютных величин. В действитель- ности дело обстоит как раз наоборот. Задача, которую ставит перед собой теория относительности, заключается в нахождении таких величин (и законов), которые не зависели бы от выбора инерциальной системы отсчета. Первой из этих величин является универсальная ско- рость распространения взаимодействий, равная скоро- сти света в вакууме. Другой, также весьма важной инва- риантной величиной является так называемый интер- вал Si2 между событиями 1 и 2, квадрат которого опре- деляется как F.13) где tl2 — промежуток времени между событиями, /i2 — расстояние между двумя точками, в которых происходят данные события (l?2~xu +#i22 +2i2^- В инвариантности интервала можно легко убедиться, вычислив его непосредственно в /С'- и /(-системах отсче- та. Воспользовавшись преобразованиями Лоренца F.8) и учитывая, что y'i2=Hi2 и z'i2 = 212, запишем: 197 12 12 1—02 1 — 02 12
Таким образом, действительно, интервал является вели- чиной инвариантной. Иначе говоря, утверждение «два со- бытия разделены таким-то интервалом s» имеет абсо- лютный характер — оно справедливо во всех инерциаль- ных системах отсчета. Инвариантность интервала играет фундаментальную роль в теории относительности и слу- жит весьма эффективным инструментом при анализе и решении многих вопросов (см., например, задачу 6.4). Типы интервалов. В зависимости от того, какая сос- тавляющая в интервале преобладает, пространственная или временная, соответствующие интервалы называют: пространственноподобными (Ii2>cti2) б Н ЫМ И (C^i2>/l2) ремениподооными (c?i2>/i2J Кроме этих двух типов интервалов существует еще тре- тий— ев ет оподоб ный (c^2 = /i2). Если интервал между двумя событиями пространст- венноподобный, то всегда можно найти такую /('-систе- му отсчета, в которой оба события происходят одновре- менно (t[2 =0): Х2/2 _/2 — _/'2 ° '12 42— 112# Если же интервал времениподобный, то всегда можно найти такую /('-систему отсчета, в которой оба события происходят в одной точке (Z'i2 = 0): Г2{2 _ /2 —г2/'2 0 *12 *12 ь 412- В случае пространственноподобных интервалов 1\%> ~>ct\2, т. е. ни в одной системе отсчета события не могут оказать влияния друг на друга, даже если бы связь меж- ду событиями осуществлялась с предельной скоростью с. Иначе обстоит дело в случае времениподобных или свето- подобных интервалов, для которых /12=^^12- Следова- тельно, события, разделенные времениподобными или светоподобными интервалами, могут быть причинно- связанными друг с другом. Преобразование скорости. Пусть в /(-системе в плос- кости х, у движется частица со скоростью v, проекции которой vx и Vy. Найдем с помощью преобразований Ло- ренца F.8) проекции скорости этой частицы о/ и vy' в ^'-системе, движущейся со скоростью V, как показано на рис. 6.11. Для этого проведем расчет по следующей схеме: , dx' _ dx'/dt , _ dy' dy'/dt x~ At' ~ At'lAt ' u~ At' ~ At'/At 198
Продифференцируем выражения F.8) для х*. у' и V по времени t и результаты подставим в предыдущие форму- лы для и/ и Vy. После несложных преобразований по- лучим ¦V 1) F.14) где р = У/'с. Отсюда скорость частицы в /С-системе -82) — . F.15) V = 1 —! Эти формулы выражают релятивистский за- кон преобразова- о к у'\к If1 tf ния скорости. При '^ малых скоростях (V<^c и »<с) они переходят, как нетрудно убедиться, в фор- мулы преобразования ско- Щ К v { Рис. 6.14 рости ньютоновской механики: Рис. 6.15 или в векторном виде v' = v-V. Обратим внимание на то, что последняя формула ока- зывается справедливой только в ньютоновском прибли- жении; в релятивистской же области она не имеет смыс- ла — здесь нет простого закона сложения скоростей. В этом можно легко убедиться хотя бы на таком примере. Пусть вектор скорости v частицы в /(-системе перпенди- кулярен оси х, т. е. имеет проекции vx = 0 и vy = v. Тогда согласно F.14) проекции скорости этой частицы в К.'- системе: vx=-v\ v'y=vvyT^p, F.i6)
Это значит, что в данном случае (у±оси х) у/-проекциЯ скорости уменьшается при переходе к ^'-системе, и яс- но, что \'=/=v—V (рис. 6.14). Рассмотрим еще один пример использования формул преобразования скорости F.14)—при движении двух частиц (см. также задачу 6.7). Пример. Пусть две релятивистские частицы движутся в /(-систе- ме отсчета навстречу друг другу по одной прямой с одинаковой ско- ростью v. Найдем: 1) скорость сближения частиц в этой системе от- счета; 2) их относительную скорость. Прежде всего необходимо уточнить, что понимается под каждой из этих скоростей. 1. Скорость сближения — это скорость, с которой изменяется (уменьшается) расстояние между частицами в данной системе от- счета. В нашем случае она просто равна 2v, причем эта скорость может быть и больше скорости света — это ничему не противоречит. 2. Под относительной скоростью имеется в виду скорость, с ко- торой одна из частиц движется в системе отсчета, связанной с дру- гой частицей и перемещающейся поступательно по отношению к ис- ходной X-системе. Чтобы найти эту скорость, выберем ось х вдоль направления движения частиц. Свяжем с одной из частиц, например частицей 1, которая движется в положительном направлении оси х, /('-систему отсчета (рис. 6.15). Тогда задача сводится к нахождению скорости частицы 2 в этой системе отсчета. Подставив в формулу F.14) для 1»х-проекции скорости vx = —f, V—v, получим Знак минус означает, что в данном случае частица 2 движется в от- рицательном направлении оси х' /('-системы отсчета. Следует отметить, что даже в том случае, когда обе частицы движутся с максимально возможной скоростью v~c, скорость vx' не может превзойти с — это сразу видно из последней формулы. И наконец, проверим непосредственно, что реляти- вистские формулы преобразования скоростей соответст- вуют утверждению второго постулата Эйнштейна отно- сительно неизменности скорости света с во всех инерци- альных системах отсчета. Пусть вектор с имеет в /С-сис- теме проекции сх и су, т. е. с2=сх2+у2. Воспользуемся формулой F.15), преобразовав в ней подкоренное выра- жение следующим образом: После этого нетрудно получить, что v' = c. При этом, ко- нечно, вектор с' в /('-системе будет иметь в общем слу- чае другое направление. 2Q0
§ 6.6. Геометрическая интерпретация преобразований Лоренца Рассмотрим релятивистские представления о пространстве — вре- мени с помощью геометрического метода, развитого Минковским и помогающего в другом свете представить сущность преобразований Лоренца. Диаграммы Минковского. Пусть имеются две инерциальные си- стемы отсчета: /(-система и /('-система, движущаяся относительно первой со скоростью V. Сначала построим так называемую диаграм- му пространства — времени для К-системы, ограничиваясь для боль- x=ct 1 1 / 1 1- / *,' / 4%' i i / / /с X Рис. 6.16 Рис. 6.17 шей простоты и наглядности одномерным случаем (рис. 6.16). На оси ординат данной диаграммы откладывают обычно не само время /, а величину x—ct, где с — скорость света. Это дает возможность про- градуировать обе оси (Ох и От) в метрах, причем в одном и том же масштабе. Каждая точка диаграммы — ее называют мировойточкой — характеризует некоторое событие А(х, т). Всякой частице (даже не- подвижной) на этой диаграмме соответствует мировая линия. На- пример, ось Ох — это мировая линия частицы, покоящейся в точке л; = 0. Ось Ох изображает совокупность всех событий, одновременных с событием О, независимо от координаты х. Мировая линия, соответствующая распространению света из точ- ки О в положительном направлении оси Ох, представляет собой бис- сектрису ОС прямого угла (точечная прямая на рис. 6.16). Изобразим на этой диаграмме оси Ох' и Ох' /('-системы. Миро- вую линию начала отсчета /('-системы получим, положив в преобра- зованиях Лоренца F.8) х'=0. Тогда x=Vt=$x, где if5=V/c. Это есть уравнение прямой, которая составляет с осью Ох угол О, опре- деляемый формулой tg O=p. Полученная прямая — мировая ли- ния — представляет собой совокупность всех событий, происходящих в начале отсчета /('-системы, т. е. ось Ох'. Ось Ох' /('-системы — это прямая, изображающая все события, одновременные в /('-системе с событием 0- Положив в преобразова- 201
ниях Лоренца F.8) /' = 0, получим ct=xVfc, нли т=Eл:. Отсюда сле- дует, что ось Ох' составляет с осью Ох тот же угол Ь (tg ©=C). Таким образом, оси От и Ох' /('-системы расположены симмет- рично по отношению к мировой линии света ОС и координатная сет- ка /('-системы (т', х') оказывается косоугольной. Чем больше ско- рость V системы К', тем более «сплющенной» будет ее координатная сетка (при V->-c она вырождается в мировую линию света). Последнее, что необходимо сделать на данной диаграмме, — это проградуировать оси От, Ох и От', Ох' обеих систем отсчета. Проще всего это можно сделать, воспользовавшись инвариантностью интер- вала: Отметим на оси Ох /(-системы точку, соответствующую единице вре- мени в /(-системе (т=1, рис. 6.17). Проведем через эту точку ги- перболу Т2 —*2=1, все точки которой отвечают инвариантному интервалу s=l (ибо при х=0 т=1 и s=l). Ее асимптотой является мировая линия све- та. Точка пересечения этой гиперболы оси От' соответствует единице времени в К'-системе. Действительно, т'2—х'2=\, и при х'=0 ко- ордината т'= 1. Аналогично градуируются и оси Ох, Ох': возьмем в /(-системе точку х=\, т=0 и проведем через нее гиперболу х2—т2=1. Тогда точка пересечения ее с осью Ох', где т'=0, дает единицу длины (х'=\) в /('-системе (так как л;'2—т'2=1 и т'=0, то х'=1). Построенная таким образом диаграмма — диаграмма Мин- ковского — соответствует переходу от К- к /('-системе и отвечает преобразованиям Лоренца F.8). В согласии с принципом относи- тельности для обратного перехода от К' к /(-системе диаграмма бу- дет иметь совершенно симметричный вид: у /('-системы координатная сетка будет прямоугольной, а у /(-системы — косоугольной (предо- ставим в этом убедиться самому читателю). Покажем, как просто и наглядно диаграмма Минковского по- зволяет интерпретировать, например, такие релятивистские эффекты, как относительность понятия одновременности, замедление времени и лоренцево сокращение. Относительность понятия одновременности следует непосредст- венно из рис. 6.18. Действительно, события А я В, одновременные в /(-системе, в /('-системе оказываются неодновременными. Событие А произойдет позже события В на время Дт'. Замедление времени. Рассмотрим двое часов, К и К', которые показывали одинаковое время т=т'=0 в момент, когда они нахо- дились в одной точке пространства (х=л;' = 0). Предполагается, что часы К неподвижные в /(-системе, а часы К' — в /('-системе. Пусть по часам К прошла единица времени (т=1); это отвечает событию А на диаграмме (рис. 6.19). Проведем через точку А гипер- болу т2—х2=1 и прямую АВ', характеризующую все события, одно- временные в /(-системе с событием А. Пересечение оси От' (мировой линии часов К') с гиперболой дает точку А' (т'=1), а с прямой ОВ' — точку В- (т'<1). Это значит, что в /('-системе в момент, когда по часам К уже прошла единица времени, по движущимся часам К' единица времени еще не прошла, т. е. часы К' идут замедленно, 203
Убедимся с помощью этой же диаграммы, что эффект замедле- ния времени является обратимым. Проведем прямую ВА', параллель- ную оси Ох,', которая характеризует все события, одновременные в /('-системе с событием Л' (т'=1). Точка пересечения В этой прямой с мировой линией часов К — осью Ох — показывает, что т<1, т. е., в самом деле, по отношению к K'-системе замедленно идущими ока- зываются теперь часы /(. Лоренцево сокращение. Пусть метровый стержень покоится в К- системе (отрезок ОА на рис. 6.20). Мировые линии его концов — это прямые Ох и AD. Чтобы измерить длину этого стержня в /('-системе, Рис. 6.19 надо зафиксировать координаты его концов одновременно в этой системе. Но в /('-системе одновременным с событием О (фиксирова- нием левого конца стержня) является событие В' — точка пересече- ния мировой линии правого конца стержня с линией одновременно- сти Ох'. Из диаграммы видно, что в /('-системе ОВ'<сОА', т. е. дви- жущийся относительно /('-системы стержень будет короче метра. Так же просто можно показать, что и лоренцево сокращение является обратимым. Если метровый стержень покоится в /('-системе (отрезок ОА'), то, проведя мировые линии его концов в этой систе- ме (От и А'В), увидим, что в /(-системе при одновременном измере- нии координат его концов отрезок ОВ<^ОА, т. е. по отношению к К- системе лоренцево сокращение будет испытывать /('-стержень. Задачи 0 6.1. Преобразование длины. В /(-системе отсчета находится не- подвижный стержень длины /=1,00 м, ориентированный под углом 0 = 45° к оси Ох (рис. 6.21). Найти его длину /' и соответствующий угол -0' в /('-системе, движущейся относительно /(-системы со ско- ростью V=c/2 вдоль оси Ох. Решение. Длина стержня в /('-системе ;1_р2) + (ДуJ, где р=У/с. Имея в виду, что Дх=/ cos ¦& и Ay=l sin О, получим 203
/' ^ly Г_р2со«2» = 0,94 м. Угол О' в /('-системе найдем через тангенс: Ах' Аху 1-| VT=\ = 1,155. Отсюда •&' = 49°. Следует обратить внимание на то, что полученные результаты не зависят от направления скорости K'-системы: она мо- жет двигаться или в положительном направлении оси х, или в про- тивоположном. В А(х=1) Рис. 6.20 К. А К х ' О О' х Рнс. 6.21 ф 6.2. Собственная длина. Стержень движется вдоль линейки с некоторой постоянной скоростью. Если зафиксировать положение обоих концов стержня одновременно в системе отсчета, связанной с линейкой, то разность отсчетов по линейке Axt = 4,0 м. Если же по- ложение обоих концов зафиксировать одновременно в системе отсче- та, связанной со стержнем, то разность отсчетов по той же линейке Дл;2 = 9,0 м. Определить собственную длину 10 стержня и его ско- рость v относительно линейки. Решение. Ясно, что в первом случае где C — скорость стержня (в единицах скорости света). Во втором же случае /о — это измеренная в системе отсчета, связанной со стер- жнем, длина участка движущейся линейки, собственный размер ко- торого (участка) равен Дх2. Поэтому Из этих двух формул легко найти, что /0 = уЛДд;1-Дд;2 = 6 м, р = у или i»~0,75 с. 204 «0,75
ф 6.3. Преобразование времени. Две нестабильные частицы дви- жутся в /(-системе отсчета по некоторой прямой в одном направле- нии с одинаковой скоростью v = 0,990 с. Расстояние между частица- ми в этой системе отсчета /=12 м. В некоторый момент обе частицы распались одновременно в /('-системе отсчета, связанной с ними. Найти: 1) промежуток времени между моментами распада обеих частиц в исходной /(-системе отсчета; 2) какая частица распалась позже в /(-системе. Решение. 1. Пусть распад частицы, двигавшейся впереди,— это событие /, а распад частицы, двигавшейся сзади, •— событие 2. Тогда, согласно преобразованиям Лоренца F.9) для времени, (*; - х'2) W/C2 ti — t где учтено, что ti' = W'—по условию. Разность (х±'—х2')—это соб- ственное расстояние /о между частицами. Согласно F.5), оно равно 10 = I/} 1 — (w/cJ • Поэтому Ivlc2 U — h — =2,0 мкс. 1 2 I_(t//CJ 2. Так как ti—^>0, то ti>t2, другими словами, частица, двигав- шаяся впереди, распалась позже. Замечание. Нередко эту задачу решают так: согласно F.8), откуда Полученный результат отличается от приведенного выше (наличием корня в знаменателе) и является неверным. Дело в том, что мы не имеем права разность xi—х2 заменять на I, ибо Х\ и х% — это коор- динаты событий (распадов), происшедшие в /(-системе в разные мо- менты времени. Расстояние же / между частицами в /(-системе равно по определению разности координат частиц, зафиксированных одно- временно. ф 6.4. Найти расстояние, которое пролетела в /(-системе отсчета нестабильная частица от момента ее рождения до распада, если ее время жизни в этой системе отсчета Д^ = 3,0 мкс, а собственное вре- мя жизни Д^о = 2,2 мкс. Решение. Воспользовавшись формулой F.12), найдем ско- рость V частицы и затем искомое расстояние как 1= M.V = Kt-cy/~\ — (Д*о/Д02=0,6 км. Другой способ решения основан на использовании инвариантно- сти интервала: 205
где квадрат интервала записан слева в системе отсчета, связанной с самой частицей, а справа — в /(-системе отсчета. Отсюда получает- ся тот же результат для /. ф 6.5. Эффект Доплера. В /(-системе отсчета находится непод- вижный приемник Р (рис. 6.22). К нему со скоростью V приближа- ется источник 5 световых сигналов. В системе отсчета, связанной с источником, сигналы испускаются периодически с частотой \>о (соб- ственная частота). С какой ча- стотой v будет воспринимать эти сигналы приемник Р? Решение. Промежуток времени между двумя последо- вательными сигналами (импуль- «-—•- сами) в /('-системе, связанной 0 0 х X с источником, равен To=l/vo. Так как эта система движется Рис. 6.22 с0 скоростью V, то соответст- вующий промежуток времени в /(-системе, согласно F.12), будет больше: Расстояние между соседними импульсами в /(-системе Поэтому воспринимаемая приемником частота v=c/\, или Если источник приближается (как в нашем случае), то v>vo, если же удаляется, то v<vo(b этом случае знак E меняется на про- тивоположный). Зависимость v/v0 от E показана на рис. 6.23. Полу- ченная формула B) для частоты v соответствует продольному эффекту Доплера. Как видно из приведенного вывода, эффект Доплера является следствием двух явлений: замедления хода движущихся часов (ко- рень в числителе формулы B)) и «уплотнения» (или разрежения) импульсов, связанных с изменением расстояния между источником и приемником (это учтено в первом равенстве формулы A)). Заметим, что в нерелятивистском случае Т = То, поэтому форму- ла для эффекта Доплера не содержит корня У 1 — р2 (вместо него стоит единица) : Рассмотрим попутно более общий случай: в /(-системе вектор скорости V источника составляет угол а с линией наблюдения, как показано иа рис. 6.24. В этом случае в формуле A) достаточно за- менить V на V cos а. Тогда 206
1 — pcosa ' В частности, при a=я/2 наблюдается поперечный эффект Доплера v = vo/l —р, при котором воспринимаемая приемником частота v оказывается всегда меньше собственной частоты vo. % 2,5 2,0 1,5 1,0 0,5 / у / / 1 / 1 1 х= ct, м -1,0 -US -0,6 -0,4 -0,2 0 0,2 ОА 0,6 Удаление Сближение Рис. 6.23 5 3 2 1 А В С >— V С OS IX ,х,м Рис. 6.24 Рис. 6.25 ф 6.6. Соотношения между событиями. На рис. 6.25 изображе- на диаграмма пространства — времени. Каждая точка этой диаграммы (мировая точка) характеризует некоторое со- бытие — его координату и момент времени, когда оно произошло. Рассмотрим три события, соответствующие мировым точкам А, В к С. Убедиться, что между этими событиями имеют место следующие соотношения: Пара событий АВ АС ВС Тнп интервала Времениподобный Пространственно- подобный Светоподобный Собственное время cAto, м 4 0 расстоя- ние Д*о, м 4 0 Возмож- ность при- чинно-след- ственной связи А~+В Нет С-+В 207
Указание- воспользоваться инвариантностью интервала 0 6 7 Две частицы движутся в /(-системе отсчета под прямым углом друг к другу, причем первая со скоростью vi, а вторая со ско- ростью t»2 Найти скорость одной частицы относительно другой Решение Возьмем оси координат /(-системы, как показано на рис 6 26 Свяжем с частицей / /(' систему, тогда скорость частицы 2 в этой системе отсчета и есть искомая скорость С помощью форму- лы F 15), положив V=vi и и* = 0, получим v2 = Заметим, что, по классическому закону сложения скоростей, 0 6 8 Преобразование направления скорости. Частица движется в К системе со скоростью v под углом ¦& к оси х Найти соответству ющий угол ¦&' в К' системе, движущейся со скоростью V, как пока- зано на рис 6 27 Решение Пусть в К системе проекции вектора v равны vx и vv Тогда для угла ¦& можно записать следующее соотношение В К.' системе с учетом формул F 14) получим tg V = vylv'x = vy YT^pl(vx- \ После подстановки vx — v cos О и t»H = sin0 найдем к ft ~S cos»— V/v Как видно из этой формулы, закон преобразования углов для ско< рости иной, нежели для отрезков (см задачу 6 1) 0 6 9 Стержень, ориентированный параллельно оси х /(-систе- мы отсчета, движется в этой системе со скоростью v в положитель- ном направлении оси у Найти угол ¦&' между стержнем и осью х' К'- системы, перемещающейся со скоростью V относительно /(-системы в положительном направлении ее оси х Оси х и х' совпадают, оси у и у' параллельны друг другу Решение Пусть в некоторый момент концы стержня совпада- ют с осью х в К системе Эти два события, одновременные в /(-систе- ме, будут неодновременными в /('-системе, согласно F 10), они про- изойдут через промежуток времени УТ=Т2 , где Ах — собственная длина стержня За это время правый конец стержня окажется «выше» левого на Д(/' = vyM', где v'y = v /"I — р (см формулу F 16)) Таким образом, в /('-системе данный стержень будет повернут против часовой стрелки на некоторый угол ¦&', кото- рый можно определить по формуле tg »' = Ау'/Л*' = №lc /1 — «2 208
где Ах' = Ддс >/*1 — Р2—проекция стержня на ось *' в К'-сиетеме p=V/c. #6.10. Релятивистское преобразование ускорения. В /(-системе движется частица со скоростью v и ускорением а. Найти ускорение этой частицы в /('-системе, которая перемещается со скоростью V в положительном направлении оси х /(-системы. Рассмотреть случаи, когда частица движется вдоль следующих осей Д'-системы: 1) х; 2) у. Л 0 0' х ..X1 Рис. 6.26 Рис. 6.27 Решение. 1. Запишем каждую проекцию ускорения частицы в /('-системе таким образом: Воспользовавшись первой из формул F.14) и последней из F.8), получим после дифференцирования П_82K/2 , п^ A-р«,;/с)»а" а^°- 2. Аналогичные расчеты приводят к следующим результатам: В этих формулах p=,V/c. Глава 7 РЕЛЯТИВИСТСКАЯ ДИНАМИКА § 7.1. Релятивистский импульс Напомним сначала два основных положения ньюто- новской механики об импульсе: 1) импульс частицы определяется как p = mv, где мас- са т частицы считается не зависящей от ее скорости; 2) импульс замкнутой системы частиц сохраняется во времени в любой инерциальной системе отсчета. 209
Теперь обратимей к релятивистской динамике. Ока- зывается (это будет видно уже из простого примера, ко- торый мы сейчас рассмотрим), для замкнутой системы из релятивистских частиц закон сохранения ньютоновс- кого импульса не выполняется. Возникает альтернатива: отказаться или от ньютоновского определения импульса, или от закона сохранения этой величины. Учитывая громадную роль, которую играют законы сохранения, в теории относительности за фундаменталь- ный принимают именно закон сохранения импульса и уже отсюда находят выражение для самого импульса*. Покажем прежде всего, что требование, чтобы закон сохранения импульса выполнялся в любой инерциальной системе отсчета, и учет релятивистского преобразования скоростей при переходе от одной инерциальной системы отсчета к другой приводят к выводу, что масса частицы должна зависеть от ее скорости (в отличие от ньютонов- ской механики). Для этого рассмотрим абсолютно не- упругое столкновение двух частиц — система предпола- гается замкнутой. Пусть в некоторой инерциальной /(-системе отсчета навстречу друг другу движутся две одинаковые частицы 1 и 2 с одинаковой скоростью v0, но под углом а к оси х (рис. 7.1, а). В этой системе отсчета суммарный импульс обеих частиц, очевидно, сохраняется: до и после столк- новения он равен нулю (образовавшаяся частица, как следует из соображений симметрии, оказывается непод- вижной) . Теперь выясним, как будет обстоять дело в другой инерциальной системе отсчета. Для этого выберем сна- чала две системы отсчета: /(i-систему, движущуюся впра- во со скоростью vix, и Л'2-систему, движущуюся влево со скоростью V2x (рис. 7.1, а). Ясно, что частица 1 в /Ci-си- * Возникает естественный вопрос: как же закон сохранения им- пульса может представлять какую-либо ценность, если импульс опре- деляют именно так, чтобы он сохранялся? Для ответа на этот вопрос представим себе частицу, которая при своем движении сталкивается с другими частицами. Рассмотрев первое столкновение, определим импульс так, чтобы выполнялся его закон сохранения в данном столкновении. Но при последующих столкновениях положение изме- нится: мы уже будем знать импульсы частиц, участвующих в этих столкновениях, и теперь закон сохранения импульса (если он дейст- вительно есть) будет выполняться уже не по определению, а в силу глубинных законов природы. Опыт показывает, что так определенный импульс действительно подчиняется закону сохранения. По крайней мере до сих пор не об- наружено ни одного явления, где бы этот закон нарушался. 210
стеме и частица 2 в /Сг-системе движутся только вдоль оси у, причем с одинаковыми по модулю скоростями, ко- торые мы обозначим и. Рассмотрим картину столкновения в Л^-системе (рис. 7.1, б), где частица 1 имеет скорость и. Найдем «/-состав- ляющую скорости частицы 2 в этой системе отсчета, обо- значив ее и'. Эта частица, как было сказано, движется со скоростью и вдоль оси у в /С2-системе и, кроме того, вместе с /^-системой перемещается влево со скоростью к1 -системе V 3 к- системе i ¦* i * .-'la г Рис. 7.1 V относительно /Ci-системы. Поэтому, согласно F.16), у- составляющая скорости частицы 2 в /Сгсистеме равна u' = uVl-0/fc?. G.1) Запишем теперь «/-составляющие импульсов обеих ча- стиц в /и-системе: m\U и т2и'. Согласно G.1), и'<и, по- этому легко видеть, что закон сохранения импульса в его обычной (ньютоновской) формулировке не выполня- ется. Действительно, в нашем случае mi = m2 (частицы одинаковые) и, следовательно, «/-составляющая суммар- ного импульса частиц до столкновения отлична от нуля, а после столкновения равна нулю (образовавшаяся час- тица будет двигаться только вдоль оси х). Потребуем, однако, чтобы закон сохранения импуль- са выполнялся и в ./^-системе, т. е. положим, что m\U — Отсюда с учетом G.1) получим При «->-0 (рис. 7.1) и-у-0 и тх представляет собой массу покоящейся частицы; ее обозначают т0 и называ- ют массой покоя. Скорость же V при этом усло- вии оказывается равной v — скорости частицы 2 относи- 211
тельно частицы /. Поэтому последнюю формулу можно переписать так: G.2) где т — масса движущейся частицы (напомним, что обе частицы одинаковые). Массу т называют реляти- вистской. Последняя, как видно из G.2), больше мас- сы покоя и зависит от скорости частицы (рис. 7.2). т > т. с к Рис. 7.2 Рис. 7.3 Таким образом, мы пришли к важному выводу: реля- тивистская масса частицы зависит от ее скорости. Други- ми словами, масса одной и той же частицы различна в разных инерциальных системах отсчета. В отличие от релятивистской массы масса покоя ча- стицы ш0 — величина инвариантная, т. е. одинаковая во всех системах отсчета. По этой причине можно утверж- дать, что именно масса покоя является характеристикой частицы. В дальнейшем, однако, мы часто будем исполь- зовать релятивистскую массу пг, что продиктовано толь- ко стремлением упростить ряд выводов, рассуждений и расчетов. Теперь сделаем последний шаг — напишем выражение для импульса релятивистской частицы. С учетом G.2) этот импульс записывают в виде У 1 - (»/С)8 G.3) Это и есть так называемый релятивистский им- пульс частицы. Опыт подтверждает, что так опреде- ленный импульс действительно подчиняется закону со- хранения независимо от выбора инерциальной системы отсчета. 212
Отметим, что при у<Сс из G.3) следует ньютоновское определение импульса: p=mov, где т0 не зависит от ско- рости v. На рис. 7.3 показаны для сравнения графики за- висимостей релятивистского ррел и ньютоновского рп им- пульсов частицы от ее скорости. Как видно, различие между обоими импульсами становится весьма значитель- ным по' мере приближения скорости частицы к скорости света. Рассмотрим два примера на применение формул G.2) и G.3). Пример 1. В современных гигантских ускорителях протоны уско- ряются до скоростей, отличающихся от скорости света на 0,0003%. Найдем, во сколько раз релятивистская масса таких протонов пре- вышает их массу покоя. Согласно G.2), m/mo=l/V I—р, где p* = i»/c. Так как р* мало отличается от единицы в данном случае, то подкоренное выражение следует представить в виде 1-ря = A + »A-Р)я2A-р). Тогда искомое отношение т/щ « 1/У 2 A — Р) » 4-102. Пример 2. Выясним, при каких значениях скорости частицы ее ньютоновский импульс отличается от релятивистского на 1%? на Ю%? Из условия т]=(р—рн)/р=1—V 1—(у/с2) получим ./-То ; f0'14 ПРИ -1=0,01, v с =у -п B—¦*])={ ' К |; \0,45 при т)=0,10. Таким образом, использование нерелятивистской формулы для им- пульса гарантирует точность не хуже 1% при а/с<0,14 и не хуже 10% при i;/c<0,45. § 7.2. Основное уравнение релятивистской динамики Согласно принципу относительности Эйнштейна, все законы природы должны быть инвариантны по отноше- нию к инерциальным системам отсчета. Другими слова- ми, математические формулировки законов должны иметь один и тот же вид во всех этих системах отсчета. В част- ности, это относится и к законам динамики. Однако, как показывает более детальное рассмотре- ние, уже основное уравнение динамики Ньютона ma = F не удовлетворяет принципу относительности Эйн- штейна. Преобразования Лоренца при переходе к дру- гой инерциальной системе придают ему совершенно иную форму. 213
Чтобы удовлетворить требованиям принципа относи- тельности, основное уравнение динамики должно иметь другой вид и лишь при и<Сс переходить в ньютоновское уравнение. Этим требованиям, как доказывается в тео- рии относительности, удовлетворяет уравнение dp/6t=F, G.4) где F-*- сила, действующая на частицу. Данное уравнение по виду полностью совпадает с основным уравнением ньютоновской динамики C.1). Однако физический смысл здесь уже другой: слева стоит производная по времени от релятивистского импульса, определяемого формулой G.3). Подставив G.3) в G.4), запишем последнее урав- нение так: mov G.5) Это и есть основное уравнение релятивист- ской динамики. Нетрудно видеть, что именно в таком виде уравнение динамики приводит к сохранению импульса для свобод- ной частицы и при малых скоростях (v<^c) принимает форму основного уравнения ньютоновской динамики (ma = F). Кроме того, именно в таком виде основное уравнение динамики оказывается инвариантным по отношению к преобразованиям Лоренца и, следовательно, удовлетво- ряет принципу относительности Эйнштейна. Не останав- ливаясь на способе доказательства этого, отметим толь- ко, что при переходе от одной инерциальной системы от- счета к другой необходимо принять, что сила F преобра- зуется по определенным законам. Другими словами, си- ла F в теории относительности — величина неинвариант- ная, в разных системах отсчета ее числовое значение и направление будут различны*. Из основного уравнения релятивистской динамики следует неожиданный вывод: вектор ускорения а части- цы в общем случае не совпадает по направлению с век- * В отличие от ньютоновской механики, где силы абсолютны, в теории относительности проекции силы, перпендикулярные направ- лению вектора относительной скорости систем отсчета, различны в разных системах. Эти проекции имеют максимальные значения в той системе отсчета, где частица в данный момент покоится: 214
тором силы F. Чтобы это показать, запишем G.5) в та- кой форме: где т — релятивистская масса частицы. Выполнив диф- ференцирование по времени, получим Это выражение графически представлено на рис. 7.4. Та- ким образом, действительно, век- тор ускорения а в общем случае не коллинеарен вектору силы F. Вектор ускорения а совпадает по у , , направлению с вектором F только в ' двух случаях: 1) если Flv (поперечная си- ла). При этом вектор скорости v по мо- дулю не меняется, т. е. n=const, и урав- нение G.5) принимает вид т0 а/У1 — (w/cJ = F, Рис. 7.4 откуда ускорение ' а = (F/m0) /I - (i//cJ ; 2) если F||v (продольная сила). В данном случае уравне- ние G.5) можно записать в скалярном виде. Выполнив в его левой части дифференцирование по времени, получим I то , откуда ускорение (в векторном виде) есть Нетрудно заметить, что при одинаковых в обоих случаях значе- ниях силы F и скорости v поперечная сила сообщает частице боль- шее ускорение, чем продольная сила. Основное уравнение релятивистской динамики позво- ляет найти закон действующей на частицу силы F, если известна зависимость от времени релятивистского им- пульса p(t), а с другой стороны, найти уравнение движе- ния частицы r(t), если известны действующая сила и на- чальные условия — скорость Vo и положение г0 частицы в начальный момент времени. В качестве примеров на применение уравнения G.5) могут служить задачи 7.1—7.3. 215
§ 7.3. Закон взаимосвязи массы и энергии Кинетическая энергия релятивистской частицы. Оп- пределим эту величину таким же путем, как и в ньюто- новской механике, т. е. как величину, приращение кото- рой равно работе действующей на частицу силы. Сна- чала найдем приращение кинетической энергии AT час- тицы под действием силы F на элементарном пути dr = At Согласно основному уравнению релятивистской динами- ки G.4), Fdt=d(m\) = dm-\ + md\, где т — релятивист- ская масса. Поэтому йТ=л где учтено, что vdv—udu (см. с. 98). Это выражение мо- жно упростить, используя формулу G.2) зависимости массы от скорости. Возведем эту формулу в квадрат и приведем ее к виду m2c2=m2v2\m\c2. Найдем дифференциал этого выражения, имея в виду, что т0 и с — постоянные величины: 2тс2 dm=2mv2 dm -f- 2m2 vdv. Если теперь разделить это равенство на 2т, то его пра- вая часть совпадет с выражением для йТ. Отсюда следу- ет dT=c2dm. G.7) Таким образом, приращение кинетической энергии ча- стицы пропорционально приращению ее релятивистской массы. Кинетическая энергия покоящейся частицы равна нулю, а ее масса равна массе покоя т0. Поэтому, проин- тегрировав G.7), получим Т=(т-то)с2, G.8) или G.9) где р = у/с. Это и есть выражение для релятивист- ской кинетической энергии частицы. Как видно, оно сильно отличается от ньютоновского mQv2/2. Убедим- 216
ся, однако, что при малых скоростях (C<С1) выражение G.9) переходит в ньютоновское. Для этого воспользуем- ся формулой бинома Ньютона, согласно которой 1 При p<Cl можно ограничиться первыми двумя членами этого ряда и тогда 1,0 ~~~' Таким образом, при больших скоростях кинетическая энергия частицы определяет- ся релятивистской формулой J-™-0 ,G.9), отличной от m0v2/2. За- метим, что G.9) нельзя представить и в виде mv2/2, где т — релятивистская мас- са. На рис. 7.5 показаны для сравнения графики зависи- мостей от C релятивистской Грел и ньютоновской Тп кине- тических энергий. Их разли- чие особенно сильно прояв- ляется в области скоростей, сравнимых со скоростью света. Пример 1 Частица с массой покоя /По движется со скоростью, при которой ее релятивистская кинетическая энергия Г в п раз пре- вышает значение кинетической энергии Ти, вычисленное по нереляти- вистской формуле. Найдем Т. Введем для упрощения записей обозначение т—T/nioC2. Тогда заданное условие Г=гапоСг/2 можно записать так: — — -4- 1 : —¦ JssS ! / \У^1 0,1 0,2 0,5 0,4- 0,5 О? 01 OS 0,9 1,0 '/ Рис 7 5 где C = 1»/с. Имея в виду, что >C связано с Т формулой G 9), найдем из нее Исключив Р2 из этих двух уравнений, получим 2ц2 + D— п)х — 2 (л— 1) = 0. Корень этого уравнения т= 1/4 (я — 4 +/л (л+ 8)). Знак минус перед корнем физического смысла не имеет (т не может быть отрицательным), поэтому он опущен. 217
Приведем четыре значения т, вычисленные по последней форму- ле для следующих значений п: п = Т/Тн: 1,01 1,1 1,5 2,0 0,0067 0,065 0,32 0,62 Отсюда видно, что, например, при Г/тоС2<О,ОО67 использование не- релятивистской формулы для кинетической энергии гарантирует точ- ность не хуже 1 %. Пример 2. Какую работу необходимо совершить, чтобы увели- чить скорость частицы с массой покоя то от 0,6 до 0,8 с? Сравним полученный результат со значением, вычисленным по нерелятивист- ской формуле. Искомая работа в соответствии с формулой G.9) равна А = Т2 - Т1 = т0 Ф- [ ' — ' ^ =0,42 т0 с*. Соответствующая же работа, по нерелятивистской формуле, Как видно, различие между обоими результатами весьма значи- тельное. Закон взаимосвязи массы и энергии. Из формулы G.7) следует, что приращение кинетической энергии частицы сопровождается пропорциональным приращением ее ре- лятивистской массы. Вместе с тем известно, что при про- текании различных процессов в природе одни виды энер- гии могут преобразовываться в другие. Например, кине- тическая энергия сталкивающихся частиц может преоб- разоваться во внутреннюю энергию образовавшейся час- тицы. Поэтому естественно ожидать, что масса тела бу- дет возрастать не только при сообщении ему кинетичес- кой энергии, но и вообще при любом, увеличении общего запаса энергии тела независимо от того, за счет какого конкретного вида энергии это увеличение происходит. Отсюда Эйнштейн пришел к следующему фундамен- тальному выводу: общая энергия тела (или системы тел), из каких бы видов энергии она ни состояла (кинетичес- кой, электрической, химической и т. д.), связана с мас- сой этого тела соотношением G.10) Эта формула выражает один из наиболее фундаменталь- ных законов природы — закон взаимосвязи (пропорцио- нальности) массы ти полной энергии Е тела. Во избежание недоразумений обратим внимание на то, 218
что в полную энергию Е не включена потенциальная энергия тела во внешнем поле, если таковое действует на тело. Соотношение G.10) можно записать и в другой фор- ме, если учесть формулу G.8). Тогда полная энергия тела где т0 — масса покоя тела, Т—его кинетическая энер- гия. Отсюда непосредственно следует, что покоящееся тело (Т = 0) также обладает энергией Е0 = т0с*. G.11) Эту энергию называют энергией покоя или соб- ственной энергией. Мы видим, что масса тела, которая в нерелятивистс- кой механике выступала как мера инертности (во вто- ром законе Ньютона) или как мера гравитационного дей- ствия (в законе всемирного тяготения), теперь выступа- ет в новой функции — как мера энергосодержания тела. Даже покоящееся тело, согласно теории относительно- сти, обладает запасом энергии — энергией покоя. Изменение полной энергии тела (системы) сопровож- дается эквивалентным изменением его массы Ат=АЕ/с2, и наоборот. При обычных макроскопических процессах изменение массы тел оказывается чрезвычайно малым, недоступным для измерений. Это можно проиллюстриро- вать на следующих примерах. Пример 1. Для выведения спутника массы от=100 кг на орбиту вокруг Земли ему сообщили скорость и = 8 км/с. Это значит, что его энергия увеличивается на hE = mv2l2 (здесь учтено, что и<Сс). Соот- ветствующее увеличение массы спутника Am = Д?/с2 = /?ш2/2с2 = з,5- 10-8LKr. Пример 2. При нагревании 1 л воды от 0 до 100° С ей сообщают энергию AE=mcp&.t, где ср = 4,2 Дж/(г-К)—теплоемкость воды, Д^ — разность температур. Соответствующее увеличение массы воды '¦ =0,47- 10-ю кг. Пример 3. Пружину с коэффициентом жесткости и=103 Н/см сжали на Ы= 1 см. При этом пружина приобрела энергию \Е= =и(Д/J/2. Эквивалентное приращение массы ее Am = Д?/с2 = 0,5-10-16 кг. Нетрудно видеть, что во всех трех случаях изменение массы лежит далеко за пределами точности эксперимен- та. 219
Однако уже в астрономических явлениях, связанных, например, с излучением звезд, изменение массы пред- ставляет собой весьма внушительную величину. В этом можно убедиться на примере излучения Солнца. Пример. Из астрономических наблюдений установлено, что коли- чество энергии, которое приносит на Землю солнечное излучение за 1 с на площадку 1 м2, перпендикулярную солнечным лучам, состав- ляет около 1,4-103 Дж/(с-м2). Это дает возможность вычислить сум- марную энергию, излучаемую Солнцем за 1 с: ?2=4-102б Дж/с, где R — расстояние от Земли до Солнца. Следовательно, Солнце ежесекундно теряет массу = 4,4-109 «г/с. Величина грандиозная с точки зрения земных масштабов, однако по сравнению с массой Солнца эта потеря ничтожно мала: Ып{т = = 2-10-21 с-1. Совершенно иначе обстоит дело в ядерной физике. Именно здесь впервые оказалось возможным эксперимен- тально проверить и подтвердить закон взаимосвязи мас- сы и энергии. Это обусловлено тем, что ядерные процессы и процессы превращения элементарных частиц сопрово- ждаются весьма большими изменениями энергии, срав- нимыми с энергией покоя самих частиц. Но к этому во- просу мы еще вернемся в § 7.5. § 7.4. Связь между энергией и импульсом частицы Ясно, что как энергия Е, так и импульс р частицы имеют различные значения в разных системах отсчета. Оказывается, однако, что существует величина — неко- торая комбинация Е и р, которая является инвариант- ной, т. е. имеет одно и то же значение в разных системах отсчета. Эта величина есть Е2—р2с2. Убедимся, что это так. Воспользовавшись формулами Е — тс2 и p = mv, за- пишем пС. с4 Е2 — р2 с* = т2 с* — т*ъ2с2= I~(t//CJ 1 или после сокращения Е2 — р2^=т2ас4. G.12) 220
Тот факт, что скорость v в правой части сократилась, означает независимость величины Е2—р2с2 от скорости частицы, а следовательно, и от системы отсчета. Други- ми словами, величина Е2—р2с2 действительно является инвариантом и имеет одно и то же значение то2с4 во всех инерциальных системах отсчета: G.13) Этот вывод чрезвычайно важен: он позволяет, как будет видно из дальнейшего, во многих случаях резко упростить анализ и решение различных вопросов. Приведем еще два полезных соотношения, с которы- ми приходится часто встречаться. Первое: G.14) и второе — связь между импульсом и кинетической энер- гией Т частицы; его легко получить, подставив в G.12) Е = т0с2 + Т, тогда G.15) Последнее соотношение при Т<^т0с2 переходит в ньюто- новское: p = ^2mQT, а при Г>т0с2 приобретает вид р = Г/с. Пример. Считая, что энергия покоя электрона равна 0,51 МэВ, вычислим: 1) импульс * электрона с кинетической энергией, равной его энер- гии покоя; 2) кинетическую энергию электрона с импульсом 0,51 МэВ/с, где с — скорость света. 1. Согласно G.15), при Т=тос2 получим р=уЗ/Иос=О,9 МэВ/с. 2 Этот вопрос можно решить также с помощью G.15). Но мож- но и проще, воспользовавшись G 12): Т =? — 1 — мо"с2= 0,21 МэВ. Рассмотрим попутно весьма интересный вопрос о воз- можности существования частиц с нулевой массой покоя (то = О). Из формул * Заметим, что в настоящее время импульсы релятивистских час- тиц выражают в единицах энергия/с (с — скорость света). Например, если энергия выражается в МэВ A МэВ=1,6-10-6 эрг), то импульс— в МэВ/с. Использование такой единицы импульса заметно упрощает многие расчеты. 221
/l-(tV<02 следует, что частица с массой покоя то = О может иметь энергию и импульс в том и только в том случае, если она движется со скоростью света с. При этом обе последние формулы принимают вид 0/0. Однако это не означает не- определенности энергии и импульса такой частицы. Де- ло в том, что обе эти величины, оказывается, не зависят от скорости, причем связь между импульсом р и энерги- ей Е дается формулой G.14), где v = c, т. е. р=Е/с. G.16) Таким образом, согласно теории относительности, сущест- вование частиц с нулевой массой покоя возможно, при- чем эти частицы могут двигаться только со скоростью с. Это движение не есть результат предшествующего уско- рения, а вообще единственное состояние, в котором та- кие частицы могут существовать. Остановка подобной частицы равносильна ее поглощению (исчезновению). Как сейчас известно, такими частицами являются фотон и, по-видимому, нейтрино. Преобразования импульса и энергии. Пусть частица движется со скоростью v = dl/dl в iC-системе отсчета. Из формулы F.13) следует, что элементарный интервал между двумя событиями, которые про- исходят с частицей, есть ds = /i " Имея в виду это выражение, представим проекции импульса и энер- гии частицы в следующем виде: т0 cdx dx dy Е== щ& Из инвариантности т0, с и интервала As сразу следует, что при пере- ходе к другой инерциальной системе отсчета рх и ру преобразуются подобно dx и dy, т. е. подобно координатам х и у, а энергия Е — по- добно времени t. Поскольку координаты и время входят в преобразования Лорен- ца F.8) линейно, мы выделили в предыдущих выражениях для р и Е одинаковую часть (т^с). Тогда можно сделать следующее сопо- ставление: Делая эту замену в преобразованиях Лоренца F.8), получим сразу искомые преобразования импульса и энергии: 222
Py=Py. E' =¦ G.17) где V — скорость /('-системы относительно /(-системы. Эти формулы выражают закон преобразования проекций импуль- са и энергии частицы при переходе от К- и /('-системе. Запись формул в более компактном виде. В настоящее время все формулы релятивистской механики принято записывать в более компактном виде с помощью использования следующих сокращен- ных обозначений: 1) величины тс2 и рс обозначают просто шири соответственно выражают в энергетических единицах (например, в МэВ); 2) все скорости выражают в единицах скорости света и обозна- чают fi: p = w/c; G.18) 3) часто встречающийся множитель 1/]/1—р так называемый лоренц-фактор: обозначают у — -V- G.19) Эти обозначения резко упрощают как вид самих формул, так и все преобразования и расчеты. Приведем основные формулы релятивист- ской динамики в этих обозначениях: релятивистский импульс G.3) р=- «оР = y щ Р; кинетическая G.9) и полная G.10) энергии: = т° ( /ПГр — !) = «о (Y — 0. Т =1 Е = m = m0 + Т = y m0; связь между энергией и импульсом G.12) — G.15): ?2._ р2 = т1 = inv р = /7- (Т + 2/в0) I преобразования импульса и энергии G.17): G.20) G.21) G.22) G.23) G.24) G.25) J>y = РУ' G.26) 223
§ 7.5. Система релятивистских частиц Об энергии и импульсе системы. До сих пор мы огра- ничивались рассмотрением поведения одной частицы. В отличие от динамики одной частицы построение динами- ки системы частиц в теории относительности является гораздо более сложной задачей. Тем не менее и в этом случае можно установить ряд важных общих законов. Если нас интересует движение системы как целого, то, отвлекаясь от внутренних процессов в системе и пре- небрегая ее пространственной протяженностью, систему можно считать одной материальной точкой — частицей. Поскольку это так, систему релятивистских частиц как целое можно характеризовать полной энергией Е, им- пульсом р, массой покоя Мо и утверждать, что получен- ные ранее выражения справедливы и для системы час- тиц как целого. Остается выяснить, что следует понимать под полной энергией Е, импульсом р и массой покоя Мо системы как целого. В общем случае, если система состоит из взаимо- действующих релятивистских частиц, ее полная энергия hct+W, G.27) где niid1 — полная энергия t'-й частицы (напомним, что в эту величину не включается энергия взаимодействия с другими частицами); W —суммарная энергия взаимо- действия всех частиц системы. В ньютоновской механике W представляет собой по- тенциальную энергию взаимодействия частиц системы — величину, зависящую при данном характере взаимодей- ствий только от конфигурации системы. В релятивистс- кой же динамике, оказывается, не существует понятия потенциальной энергии взаимодействия частиц. Это обу- словлено тем обстоятельством, что само понятие потен- циальной энергии тесно связано с представлением ' о дальнодействии (мгновенной передаче взаимодействий). Являясь функцией конфигурации системы, потенциаль- ная энергия в каждый момент времени определяется от- носительным расположением частиц системы в этот мо- мент. Изменение конфигурации системы должно мгно- венно вызвать изменение и потенциальной энергии. Так как в действительности этого нет (взаимодействия пере- даются с конечной скоростью), то для системы реляти- вистских частиц понятие потенциальной энергии взаимо- действия не может быть введено. 224
В общем случае написать выражение для энергии взаимодействия W, а следовательно, и для полной энер- гии Е системы взаимодействующих релятивистских час- тиц не представляется возможным. Это же относится и к импульсу системы, так как в релятивистской динамике импульс це является величиной, независимой от энер- гии Е. Так же сложно обстоит дело и с массой покоя уИ0 системы, о которой в общем случае можно сказать толь- ко одно: это масса в системе отсчета, где данная меха- ническая система как целое покоится (т. е. в //-системе). Вследствие указанных трудностей построение дина- мики системы релятивистских частиц ограничено срав- нительно немногими простейшими случаями, на двух из которых мы и остановимся. Это система из невзаимодей- ствующих релятивистских частиц и важный в практиче- ском отношении случай столкновения двух частиц. Система невзаимодействующих частиц. В этом случае полная энергия Е и импульс р обладают аддитивными свойствами и для системы их можно представить в виде G.28) где mi и р,- — релятивистская масса и импульс ?-и части- цы системы. Так как взаимодействий в данном случае нет, то скорости всех частиц постоянны, а следовательно постоянны во времени полная энергия и импульс всей системы. Введем понятие энергии покоя Ео системы частиц как полную энергию ее в //-системе, где суммарный импульс р = 2Р' — 0, и система как целое покоится. Таким обра- зом, Ео= где Et — полная энергия t'-й частицы в //-системе. Это значит, что в энергию покоя входит кроме энергии покоя каждой частицы и их кинетическая энергия Ti в //-сис- теме: Ei = m0iC2 + Ti. Это же относится, очевидно, и к массе покоя системы: М0 = Е0/с2. G.30) Отсюда, в частности, следует, что масса покоя системы не равна сумме масс покоя отдельных частиц, а именно: 225
Введение энергии и массы покоя системы (Ео и Мо) позволяет рассматривать систему невзаимодействующих релятивистских частиц как одну частицу с полной энер- гией E^^niiC2, импульсом Р=2 Р«' массой покоя Мо = = Eujc2 и утверждать, что выражения G.12) и G.14) справедливы и для системы частиц: 2c4 = inv, G.31) р = Е\/с\ G.32) где V — скорость системы частиц как целого, т. е. ско- рость Д-системы. Эту скорость, согласно G.32), можно представить в таком виде: где пц— релятивистская масса i-й частицы системы. За- метим, что G.33) по форме совпадает с соответствующим нерелятивистским выражением D.9) для скорости цент- ра масс системы. Столкновение двух частиц. Рассмотрим процесс столк- новения, происходящим в два этапа: сначала образова- ние некоторой составной частицы Л* и затем ее распад на какие-то в общем случае другие частицы: В процессе сближения частиц А\ и Л2 взаимодействие между ними может становиться не малым, и формулы G.28) теряют свою применимость. Однако после того, как возникшие частицы разойдутся на большое расстоя- ние друг от друга, эти формулы опять применимы. В данном случае можно показать, что сумма полных энергий двух исходных частиц (когда они находятся на- столько далеко друг от друга, что их взаимодействие пренебрежимо мало) равна полной энергии составной частицы. Это же относится и ко второй стадии процес- са— распаду. Другими словами, можно показать, что для этого процесса оказывается справедливым закон со- хранения полной энергии в таком виде: Я1+Я2=?*=?'з + ?4+---- G.34) Убедимся, что это именно так, на следующем простом примере. 226
Представим себе столкновение двух одинаковых час- тиц 1 и 2, в результате которого образуется некоторая составная частица. Пусть частицы до столкновения дви- жутся навстречу друг другу в /(-системе с одинаковыми скоростями и, как показано на рис. 7.6. Рассмотрим те- перь этот процесс в /('-системе, движущейся влево со скоростью V относительно К- системы. Так как в /(-систе- ме скорость каждой частицы перпендикулярна вектору V, то, согласно F.14), обе час- тицы в /('-системе имеют ^-компоненту скорости, рав- ную V. Такую же скорость в /('-системе будет иметь и об- разовавшаяся частица, ре- лятивистскую массу которой обозначим М. Из закона со- хранения импульса до и по- рис 7б еле столкновения получим (для ^-составляющей импульса) 2m(v')V=MV, где v'— скорость каждой исходной частицы в /('-системе. Отсюда т. е. сумма релятивистских масс исходных частиц равна релятивистской массе образовавшейся частицы. Анало- гично дело обстоит и в /(-системе. Действительно, при очень малом значении скорости V скорость v' практиче- ски равна v, а масса М — массе покоя Мо образовавшей- ся частицы, так что в /(-системе 2m(v)=M0. Отсюда видно, что масса покоя образовавшейся частицы больше суммы масс покоя исходных частиц. Кинетичес- кая энергия исходных частиц претерпела превращение, в результате которого масса покоя образовавшейся ча- стицы превысила сумму масс покоя исходных частиц. Итак, мы показали, что вследствие сохранения им- пульса системы сумма релятивистских масс исходных частиц равна релятивистской массе образовавшейся час- тицы. Это же, очевидно, относится и к полной энергии. Поэтому можно утверждать, что сохранение полной энер- гии в форме G.34) действительно имеет место для рас- сматриваемых стадий этого процесса. 227
Применение закона сохранения энергии к ядерным процессам позволило, как уже говорилось в конце § 7.3, экспериментально проверить справедливость одного из фундаментальных законов теории относительности — закона взаимосвязи массы и энергии. Рассмотрим при- меры. Пример 1. Энергетический выход ядерных реакций. Возьмем ядерную реакцию типа где слева — исходные ядра, справа — ядра — продукты реакции. При- меним к этой реакции закон сохранения полной энергии: Е\ + ^2 = ?з + ^4- Имея в виду, что полная энергия каждой частицы может быть пред- ставлена как ?=ОТос2 + 7', где то — масса покоя соответствующего ядра, Т — его кинетическая энергия, перепишем предыдущее равен- ство так: (/И! + гп2) & + Тп = (т3 + пц) С2 + Тм, где Т\2 и Г34 — суммарные кинетические энергии ядер до и после реакции. Отсюда Ти—т\2~(Щ +т2) с2 — (/и3 + /и4)с2. Левая часть этого равенства есть приращение суммарной кинетиче- ской энергии ядер данной системы — то, что называют энергети- ческим выходом ядерной реакции и обозначают Q. Итак, Q = [(«1 + Щ) — (/Пз + Ш4)] С2. Эта величина может иметь любой знак — в зависимости от характе- ра той или иной ядерной реакции. Таким образом, энергетический выход ядерной реакции определяется разностью суммарных масс покоя ядер до и после реакции. Все величины, входящие в это соот- ношение, могут быть экспериментально измерены с достаточно высо- кой точностью, тем самым можно проверить и само равенство. Рассмотрим конкретную ядерную реакцию: 7Li + Щ -> 24Не Измеренные массы покоя этих ядер (в атомных единицах массы а. е. м.) равны соответственно 7,0160, 1,0078 и 4,0024 а. е. м. Отсюда нетрудно подсчитать, что сумма масс покоя ядер в результате ядер- ной реакции уменьшилась на 0,019 а. е. м. Учитывая, что 1 а. е. м. соответствует энергии 931,4 МэВ, найдем Q = 0,019-931,4 МэВ = = 17,7 МэВ. Этот результат с большой точностью совпадает с дан- ными эксперимента. Пример 2. Распад частицы. Пусть покоящаяся частица Л( само- произвольно распадается на частицы Л2 и Аз. Согласно закону со- хранения полной энергии, ^1 — ?2 + ?3- Так как полная энергия каждой частицы Е = тас2 + Т, то предыду- щее равенство примет вид 228
где Г2з — суммарная кинетическая энергия образовавшихся частиц. Эту энергию называют энергией распада Q. Таким образом, Q = [Щ — (т2 + п»з)] с2. Поскольку Q — величина существенно положительная, самопроиз- вольный распад частицы возможен только при условии т. е. если масса покоя первичной частицы больше суммы масс покоя возникающих частиц. В противном случае самопроизвольный распад невозможен. Эксперимент полностью подтверждает этот вывод. Рассмотрим, например, распад я-мезона. Экспериментально уста- новлено, что заряженные я-мезоны распадаются на мюон и нейтри- но: я—>-n+v. Согласно табличным данным, массы покоя этих частиц (в единицах массы покоя электрона) равны соответственно 273,2, 206,8 и 0. Отсюда следует, что масса покоя в результате распада уменьшается на 66,4 электронной массы. Так как массе покоя элек- трона соответствует энергия 0,51 МэВ, то энергия данного распада Q = 66,4-0,51 МэВ = 34 МэВ, что находится в точном соответствии с результатами эксперимента. Тот факт, что в результате столкновения частиц и по- следующего затем распада составной частицы полная энергия системы (а значит, и ее импульс) не меняется, приводит к другому важному выводу: величина Е2—р2с2 для системы будет инвариантной не только по отноше- нию к разным инерциальным системам отсчета, но и для указанных выше стадий процесса столкновения. Пусть, например, две релятивистские частицы испы- тали столкновение, в результате которого образовалась новая частица с массой покоя Mq. Если в /(-системе от- счета полные энергии частиц до столкновения равны Е\ и Е2, а их импульсы — соответственно pi и рг, то мы сразу можем записать, что при переходе от /(-системы (до столкновения) к//-системе (после столкновения) бу- дет выполняться следующее равенство: Ч^=М\ с\ G.35) /(-система Д-сисмема где учтено, что в //-системе образовавшаяся частица по- коится. Инвариантность величины Е2—р2с2 дает нам незаме- нимый инструмент при изучении различных процессов распада и столкновения релятивистских частиц, с по- мощью которого чрезвычайно упрощается как анализ са- мих процессов, так и соответствующие расчеты. 229
Пример. В /(-системе отсчета частица с массой покоя т0 и кине- тической энергией Т налетает на другую, покоящуюся, частицу с той же массой покоя. Найдем массу покоя Мо и скорость V составной частицы, образовавшейся в результате столкновения. Воспользовавшись инвариантностью величины Е2—р2с2, запишем где левая часть равенства относится к /(-системе отсчета (до столк- новения), а правая — к //-системе (после столкновения). В данном случае Е=Т + 2т0с2. Кроме того, согласно G.15), р2с2 = Т(Т+2тос2). Поэтому (Т + 2m0 c2J -T(T + 2т0 &) = М\ с*. Отсюда ЛТ0 =— У 2т0 (Т ¦ с Скорость образовавшейся частицы — это скорость //-системы. Со- гласно G.32), V-. Задачи Внимание! В задачах 7.4—7.11 использованы сокращенные обозначения, приведенные в конце § 7.4 (например, р и т0— это со- кращенные записи величин рс и т0с2). 0 7.1. Движение под действием продольной силы. Частица с мас- сой покоя ,/По начала двигаться под действием постоянной силы F. Найти зависимость скорости частицы от времени. Решение. Умножим обе части уравнения G.5) на dt, тогда Проинтегрировав это выражение с учетом того, что в начальный мо- мент а = 0, получим moWyi — (vjcJ = Ft. Отсюда Сравним полученное выражение с ньютоновским. Согласно вто- рому закону Ньютона, a=F/m0 и скорость Vn=FtImu, поэтому пре- дыдущее выражение для скорости v (t) можно представить так: Отсюда видно, что о<аш т. е. действительная скорость v частицы растет со временем медленнее, чем vB, причем при t-^f-oo скорость v-*-c (рис. 7.7). Интересно, что импульс частицы при этом будет расти линейно со временем: из уравнения dp/cU=F следует, что p—Ft. В этом ха- 230
If рактерная особенность релятивистского движения: в то время как скорость частицы стремится к определенному пределу (т. е. практи- чески устанавливается), импульс частицы продолжает расти. 0 7.2. Движение под действием поперечной силы. Релятивист- ская частица с массой покоя т0 и зарядом q движется в постоянном однородном магнитном поле, индукция которого В. Движение про- исходит по окружности радиуса р в плоскости, перпендикулярной вектору В. Найти импульс и круговую частоту обращения частицы по окружности. Решение. В данном случае частица движется под действием силы Лоренца F= = g[vB], где v — скорость ча- стицы. Так как FJLv, то модуль скорости частицы v = const и уравнение G.5) принимает вид та. = q [vB], где т — релятивистская масса частицы. Имея в ви- ду, что а представляет со- бой нормальное ускорение, равное по модулю fz/p, пе- репишем предыдущее урав- нение так: mv2/p — qvB. От- сюда импульс частицы Рис. 7.7 р = mv = q p В. A) Видно, что произведение рВ может служить мерой релятивистского импульса частицы. Период обращения частицы по окружности T=2npjv, откуда круговая частота обращения ы = 2п/Т=ю/р. Учитывая A), получим ы = qB/m. Значит, круговая частота w зависит от скорости частицы: чем боль- ше скорость частицы, а следовательно, и ее релятивистская масса т, тем меньше частота ш. Однако при малых скоростях (а<с) т-+-т0 и <о = qB/rriQ = const, т. е. при нерелятивистских скоростях частота со практически не зави- сит от скорости. О 7.3. Релятивистский протон с импульсом ро влетел в момент t=Q в область, где имеется поперечное однородное электрическое поле с напряженностью Е, причем po-LE. Найти зависимость от вре- мени угла О, на который протон будет отклоняться от первоначаль- ного направления движения. Решение. Выбрав оси координат (х — вдоль вектора ро> У — вдоль вектора Е), запишем уравнение G.4) в проекциях на эти оси: где е — заряд протона. Из этих уравнений следует, что рх- = eEt, или 231
ITlQVy = Л>, = eEt. A) V I - (V /CJ Взяв отношение последних двух равенств, найдем Интересно отметить, что в отличие от нерелятивистского случая здесь vx уменьшается со временем. Чтобы в этом убедиться, возве- дем оба равенства A) в квадрат и затем сложим отдельно их левые и правые части: Заметив, что vx2 + vyi = v2, получим Подставив это выражение в первое из A), найдем с Vx /1+(т0с/Л)J+ (eEt/PoJ ' т. е. действительно, vx уменьшается с ростом t. ф 7.4. Симметричное упругое рассеяние. Релятивистский протон с кинетической энергией Т испытал упругое столкновение с покоив- шимся протоном, в результате чего оба протона разлетелись симмет- рично относительно первоначального направления движения. Найти угол между направлениями разлета протонов после столкновения. Решение. При симметричном разлете протонов их импульсы и энергии должны быть одинаковы по модулю. Это сразу видно из треугольника импульсов (рис. 7.8), выражающего закон сохранения импульсов. Из этого треугольника, согласно теореме косинусов, сле- дует, что р2 = 2р'2-\-2р'2 cos 6, откуда cos 8 = /?2/2//2_ 1. Воспользовавшись формулой G.25) и учтя, что Т=2Т', где Т — кинетическая энергия каждого протона после столкновения, найдем = 4- р'2 Г (Т' + 2ш0) где Ото — масса покоя протона. После подстановки этого выражения в формулу для cos 0 получим cos 6 =Т/(Т +4ш0). Заметим, что в отличие от нерелятивистского случая, когда 8 = я/2, здесь 9<я/2. ф 7.5. Рассеяние фотона на электроне. Фотон с энергией е испы- тал рассеяние на покоившемся свободном электроне. Найти энергию е' рассеянного фотона, если угол между направлениями движения рассеянного и налетающего фотонов равен #. 232
Решение. Воспользуемся законами сохранения энергии и им- пульса. В данном процессе Те — е — е', ре = р — р', где Те и ре — кинетическая энергия и импульс электрона отдачи, р и р' —¦ импульсы налетающего и рассеянного фотонов. Из треугольника импульсов (рис. 7.9), согласно теореме косинусов, следует, что Подставив сюда р=е, р'=е' и Ре = VTe (Те + 2те) = i — s') (s — е' + 2те) Рис. 7.8 где тяе — масса покоя электрона, получим после несложных преобра- зований ф 7.6. К методу встречных пучков. Два протона движутся на- встречу друг другу с одинаковыми кинетическими энергиями Т (в /(-системе отсчета). Найти кинетическую энергию 7" одчого протона в /('-системе отсчета, где другой протон покоится. Решение. Воспользуемся инвариантностью величины Е2—р2, записав ее в /(-системе (она здесь является одновременно и Я-систе- мой), а также в /('-системе: [2 G1 + яю)]2 = G1' + 2тоJ - Т' {Т> + 2т0), где то — масса покоя протона. Отсюда Например, для протонов (ото~1 ГэВ) при Г=50 ГэВ величина 7"' = 5-103 ГэВ. Возможность получения такого большого «выигры- ша» в энергии лежит в основе метода встречных пучков. 0 7.7. Энергетическая схема ядерной реакции. Частица At с ки- нетической энергией 7\ налетает на покоящееся ядро Лг (в К-систе- ме). В результате реакции образуются ядра Аз и Ас Массы покоя частиц равны соответственно т\, т^ т3, т^. Изобра- зить энергетическую схему ядерной реакции для двух случаев а) (т1 + тг)Хтз+мп); б) (mi + m2)<(тз+пи). 233
Найти для второго случая пороговую кинетическую энергию 7\ Пор налетающей частицы в К-системе отсчета. Решение. Из закона сохранения полной энергии следует, что в Д-системе 7x2 + Щ -\- Ш2 = 7^34 + тЗ + т4> где Тп и ?з4 — суммарные кинетические энергии частиц до и после реакции. Обозначив приращение кинетической энергии Ти.—?J через Q, запишем предыдущее выражение так: Q = (щ + щ) — (от3 + щ). Величину Q называют энергетическим выходом ядерной реакции или, короче, энергией реакции. Рис. 7.10 Энергетическая схема ядерной реакции показана на рис. 7.10. В случае а эффект будет положительным, Q>0: суммарная кинети- ческая энергия увеличивается за счет уменьшения суммы масс покоя частиц системы; в случае б — наоборот. В последнем случае, как видно из рис. 7.10, б, ядерная реакция возможна лишь при условии ?i2^|QI, где знак равенства соответст- вует пороговому значению энергии Ti2. При нерелятивистских ско- ростях, согласно D.16), Т 12 = 1/2Ц"отн или 12 = /7^2 Отсюда, имея в виду, что Tlz=\Q\ и Т1 = ТщоР, получим f7l2 ф 7.8. Пороговая энергия. Релятивистская частица с массой по- коя imo налетает на покоящуюся частицу с массой покоя Мц. В ре- зультате столкновения возникают частицы с массами покоя mi, /nz,... по схеме Щ + Мо -* mi + m2 + ... . Найти пороговую (минимальную) кинетическую энергию ГПор нале- тающей частицы, необходимую для осуществления данного процесса. 234
Решение. Прежде всего ясно, что о пороговой энергии может идти речь только в том случае, когда сумма масс покоя возникших час- тиц превышает сумму масс покоя первичных частиц. Чтобы найти ГПОр, воспользуемся инвариантностью величины Е2—р2. Запишем эту вели- чину до столкновения при Г=Гпор в системе отсчета, где частица М„ покоилась, и после столкновения — в //-системе: Е2—р2 — Е2, или (Гпор + т0 + М0J - Тпор (Гпор + 2mQ) = 1 Здесь учтено, что в //-системе кинетическая энергия возникших час- тиц равна нулю на пороге реакции, поэтому их полная энергия равна просто сумме масс покоя отдельных частиц. Из последнего уравнения находим (m1 + w2+...J—(wo + AfoJ пор ~ 2М0 # 7.9. Найти пороговую энергию фотона для рождения пары электрон — позитрон в поле покоящегося протона, если массы покоя электрона и позитрона равны то, а протона — Ма. Решение. Воспользуемся инвариатностью величины Ег—рг и запишем ее до взаимодействия в системе отсчета, где протон покоит- ся, а после взаимодействия — в //-системе. При пороговом значении энергии е налетающего фотона Кор + Л*оJ- е'ор = М + 2«0J- Отсюда епор=2т0A +щ/М0). Видно, что для рождения пары необходимо, чтобы энергия фотона была больше 2«о (этого требует закон сохранения импульса). ф 7.10. Энергии частиц в Д-системе. Фотон с энергией е в лабо- раторной системе отсчета налетает на неподвижную частицу А, мас- са покоя которой равна /п0. Найти: 1) скорость //-системы этих двух частиц; 2) энергию фотона и частицы А в данной Д-системе. Решение I. Согласно формуле G.32), скорость //-системы 2. Из преобразования G.26) для энергии следует, что в //-системе энергия фотона где рс—скорость //-системы. Подставив сюда р=е и выражение для рс из предыдущего пункта, получим Частица А движется в //-системе со скоростью р=Рс поэтому ее полная энергия в //-системе 235
~ Щ Щ + ' hi = В правильности полученных формул можно убедиться, воспользовав- шись инвариантностью величины Е2—р2 при переходе от лаборатор- ной к Д-системе отсчета: О 7.11. Распад движущейся частицы. Релятивистский л°-мезон с массой покоя т0 распался на лету на два 1уФ0Т0На с энергиями Ei и 8г (в /(-системе отсчета). Найти угол 0 между направлениями разлета этих фотонов. Решение. Исходя из инвариантности величины Е2—р2, запи- шем ее до распада в Lf-системе, а после распада — в ./(-системе: ml = (е, + s2J - (Pi где pi и р2 — импульсы фотонов. Преобразуем правую часть этого уравнения, учитывая, что pi = ei и рг = гг. Тогда /и^ = 2е1е2—2p1p2 = 2s1e2(l— cos 6). Отсюда . _Jj_ щ 2 ~~ 2УчЧ
ПРИЛОЖЕНИЯ 1. Движение точки в полярных координатах В полярных координатах р, ф положение точки А на плоскости определено, если заданы ее расстояние р от начала отсчета О и угол Ф между радиусом-вектором р точки и выбранным направлением 00' — началом отсчета угловой координаты ф (рис. 1, а). Введем единичные векторы — орты е р и е , связанные с движу- щейся точкой А и направленные в сторону возрастания соответству- Рис. 1 ющих координат р и ф, как показано на рис. 1, а. В отличие от ортов декартовой системы координат, орты е р и е ? — подвижные (при движении точки А они меняют свое направление). Найдем сра- зу же их производные по времени — они понадобятся ниже. При дви- жении точки А за промежуток времени dt оба орта повернутся в одну сторону на один и тот же угол с!ф (рис. 1, б) и получат прира- щения: (-ер)- Поделив оба выражения на dt, получим (О где точка сверху над буквой означает дифференцирование по вре- мени. Теперь найдем скорость и ускорение точки А, записав ее радиус- вектор р в виде Р = Рер. B) 237
A): Скорость точки v. Продифференцируем B) по времени с учетом C) Отсюда видно, что проекции вектора v на подвижные орты е р и равны: UP = P> v =?'<?, I а модуль вектора скорости v=V р2 + р2ф2. Ускорение точки а. Продифференцировав C) еще раз по време- ни, получим d . а = р ер + р е р + — (?<р) е? + р <f ef . Рис. 2 Учтя A), после несложных преобразований найдем а = (Р — Р <Р2) ef + Bр у + р?) ef , E) т. е. проекции вектора а на орты е и е имеют вид вр = р — Р<Р2, а? = 2р f + р'<р = Основное уравнение динамики в полярных координатах. Основ,- ное уравнение динамики ma=F в проекциях на подвижные орты е р и е легко получить сразу, воспользовавшись формулами F): G) (рис. 2). На где F и F —проекции вектора F на орты е и е этом рисунке Fp<0, a F(p>0. 238
2. О задаче Кеплера В задаче Кеплера рассматривается вопрос о движении частицы в центральном поле сил, убывающих обратно пропорционально ква- драту расстояния от центра поля. Этому закону подчиняются силы гравитационного притяжения между материальными точками (или телами, обладающими сферической симметрией), а также кулонов- ские силы между точечными зарядами. В таком поле потенциальная энергия частицы U=—а/р, где а— постоянная, р — расстояние от центра поля. Рассмотрим случай, ког- да а>0, т. е. сила, действующая на частицу массы т, направлена к центру поля (притяжение). Какой вид будет иметь траектория час- тицы в полярных координатах р(<р), если при ф = 0 р(О)=ро, а ско- рость частицы перпендикулярна радиусу-вектору и равна v0 (рис. 3)? Для решения этой задачи обычно используют законы сохранения энергии и момента импульса. В полярных координатах р, ф из этих законов следует: 1/2/и (р2 + р2;^2) _ а/р=Е, mfii = L, где Е и L — полная механическая энергия и момент импульса части- цы относительно точки О — центра поля. Обе эти величины легко найти из начальных условий. Решение данных уравнений проводят следующим образом. Сна- чала в первом уравнении переходят от дифференцирования по вре- мени к дифференцированию по ф — это можно сделать с помощью второго уравнения: At= (mp2/L)dq>. Затем разделяют переменные р и ф, т. е. приводят полученное выражение к виду dq) = f (p) dp. И на- конец, интегрируют это уравнение с учетом начальных условий. Ре- зультат интегрирования и дает искомое решение р(ф). Мы не будем здесь подробно воспроизводить довольно громозд- кий ход решения этих уравнений (при желании его можно найти почти в любом курсе теоретической физики или механики). Ограни- чимся лишь анализом полученного решения, которое имеет вид Р° . (О где a=ia/mpot>o2. Из математики известно, что уравнение A) определяет кривую второго порядка. В зависимости от значения параметра это может быть эллипс (окружность), парабола или гипербола. 1. Сразу видно, что при а=1 величина р не зависит от ф, т. е. траекторией является окружность. Такую траекторию частица будет иметь при скорости va, равной vl=VaJmJ0. B) 2. Для всех значений параметра а, при которых р конечно вплоть до ф = я, траектория будет иметь форму эллипса. Как следует из A), при ф=я р(я) = ро/Bя-1). Отсюда видно, что р(я) будет конечным лишь при 2а>1, т. е. при скорости va<vu, где 1/„ =У2а//вр0. C) 239
3. Если же 2а=1, т. е. vo = vu, то эллипс вырождается в пара- болу — частица обратно не вернется. 4. При Vo>vu траектория будет иметь форму гиперболы. Все эти случаи показаны на рис. 4. Следует обратить внимание на то, что для эллиптических орбит центр поля совпадает с одним из фокусов эллипса: при vo<ui — с задним фокусом, а при t>o>yi — с передним. Заметим, что уравнение A) описывает, например, траектории пла- нет Солнечной системы (при этом а=утМ, М — масса Солнца). Применительно к движению космических аппаратов скорости vi и уц Рис. 4 Рис. 5 являются соответственно первой и второй космическими скоростями. Ясно, что их значения зависят от массы тела, являющегося источни- ком поля. 3. Доказательство теоремы Штейнера Теорема: момент инерции / твердого тела относительно произ- вольной оси О равен моменту инерции /с этого тела относительно оси С, параллельной данной и проходящей через центр масс тела, плюс произведение массы т тела на квадрат расстояния а между осями: Доказательство. Пусть положение 1-го элемента твердого тела относительно осей О я С характеризуется векторами р,- и р(, а по- ложение оси С относительно оси О — вектором а (рис. 5, плоскость которого перпендикулярна осям О и С). Воспользовавшись связью между этими векторами (?,- = ?,- +а), преобразуем выражение для момента инерции тела относительно оси О следующим образом." 1=%гт?'12 + 2и mi p'{ + ¦.<#. 240
В правой части этого равенства первая сумма представляет собой момент инерции тела /с относительно оси С, а последняя сумма просто равна та2. Остается показать, что средняя сумма равна нулю. Пусть IV — радиус-вектор i'-го элемента тела относительно цент- ра масс, тогда относительно последнего суммарный вектор ?mirl' = = 0, согласно C.8). Но р( — это составляющая вектора г/, перпен- дикулярная осям О и С. Отсюда ясно, что если суммарный вектор равен нулю, то и сумма его составляющих в плоскости, перпендику- лярной осям О и С, также равна нулю, т. е. ?niip'~0. Теорема, таким образом, доказана. 4. Греческий алфавит А, в, Г, А, Е, Z, н, в, а Р Y б е ¦п 9, — альфа — бета — гамма — дельта — эпсилон — дзэта — эта ¦§ — тэта I, К Л, М N, О, п, 1- X к V 1 о я - йота — каппа —ламбда — мю — ню — кси — омикрон — пи Р, 2, т, Ф, х, Q, Р а т V Ф X со — ро — сигма — тау — ипсилон — фи — хи — пси — омега 5. Основные единицы СИ в механике Это единицы времени — секунда (с), единица длины — метр (м) и единица массы — килограмм (кг). Секунда — это промежуток времени, в течение которого совер- шается 9 192 631770 колебаний электромагнитного излучения, соот- ветствующего переходу между двумя определенными сверхтонкими уровнями основного состояния атома цезия-133. Эталон времени и частоты состоит из атомно-лучевой трубки с пучком атомов цезия и радиотехнического устройства, которое дает набор электрических сигналов фиксированной частоты. Секунда приблизительно равна 1/86400 средних солнечных суток. Метр — это длина, равная 1 650 763,73 длин волн в вакууме оран- жевой линии атома криптона-86 (линии, соответствующей переходу между уровнями 2рю и 5d5 данного атома). Эталон для воспроиз- ведения метра представляет собой комплекс аппаратуры, включающей интерферометры для точного измерения длин. Метр приблизитель- но равен 1/40 000 000 доле длины земного меридиана. Килограмм — это масса платино-иридиевого эталона, храняще- гося в Международном бюро мер и весов (в Севре, близ Парижа). Масса эталона близка к массе 1 дм3 чистой воды при 4° С. 241
6. Формулы 1Лгебры и тригонометрии Корни квадратного уравнения ax2+bx + c=0: Х^~ 2а Некоторые приближенные формулы. Если а<С1, то A ±а)" = 1 ± па еа = 1 + а 1пA +а) =а sin a = a cos a = 1 — a2/2 tga=a Основные тригонометрические формулы: sin2a + cos2a = 1 sec2 a — tg2 a = 1 csc2 a — ctg2 a = 1 sin ct-csca = 1 cos a-sec a = 1 tga-ctga = 1 1 sin a = V 1 + ctg2a 1 /1 +tg2a sin2a = 2 sin a • cos a cos 2 a = cos2 a — sin2 a, . „ 2tga l^~ 1_tg2ce ctg2a-l cig la -- 2clg a a 1 — cos a MI1" 2 ~ 2 a 1 + cos a — 2 - 2 sin (a"± P) = sin a cos p ± cos (a d-'P) = cos a cos P T tirfa 1 ft tga^tgP ^(а1Р>-1Т18а.12р ct-Га I C^ ctSa-ctgP ctgtoiP)- ctgpTcti tt + P sin a + sin p = 2sin — cc -j- с CO a-f os a + cos p — 2 cos соь a sin sin a sin T 1 I a a — соь 2 P a — sin I Э o- cos 2 a _i_ В а — i U — COS P -^ — <?Sin sin(a±P) tg U ± tg p — cos a•cos f sin (a ctg a, ±ctg [i — ± sma- 2siu a- sinp = cos(a — P) — 2cos a • cos p = cosD — P) + 2sina-cos P = sin(a— P) + ° 2 ±P) sin p cos(a + cos (a + sin(a + P TD \ I* P P) P) P)
7. Таблица производных и интегралов Функция хп 1 X 1 X4 V~x е ах \пх Va lnu и V Производная пх"-1 1 И 1 ех ле" 1 д: и' 2/~ и' и vu' — v'u V2 Функция siax COS X tgx ctg x arcsin x arctg x arcctg x sh x chx tn x cth^ Производная COS JC — sin x 1 cos2* 1 Slh2jt 1 УТ=Гх2 1 /1 — ^2 1 1 1 +X? ch x — sh jc 1 ch2* 1 ~ sh2^ xn+1 x"dx = , пФ — ; n+l \ — In x J sin jc djc = — cos x Jcos jc dx= sin x j tg * djc = — In cos jc x = In sin jc с их r dx \ . 9 = — ctgjc J Sin2 JC Si J= J exdx = ел dA: + *2 d^: 1 = arcsin x Y\ — xi (* dx —^—^^—
8. Некоторые сведения о векторах Скалярное произведение векторов: ab = ba = ab cos a a(b + с) =ab + ac. Векторное произведение векторов: [ab] = — [ba]; | [ab] | = ab sin а; [a, b + c] = [ab] + [ac]. Смешанное, или векторно-скалярное, произведение трех векто- ров является скаляром и численно равно объему параллеле- педа, построенного на этих векторах: a [be] =b[ca] =c[abj; a [be] = — b[ac] = — a[cb]. Двойное векторное произведение: [a [bc]]=b(ac) — c(ab). Произведение векторов в координатном представлении. Если a =atei + b = где ei, ег, ез — координатные орты (взаимно перпендикулярные и образующие правую тройку), то ab = а\Ъ\ + 1 ег е3 [ab] ах b Правила дифференцирования векторов, зависящих от некото- рой скалярной переменной t: d da db — (a + b) = — +—; d r ki Г dt da d^ da Градиент скалярной функции ф v* дх ^ di -a + b + 1 ']¦*¦ db E dt ; Г db 1 где i, j, k — координатные орты осей х, у, г.
9. Единицы механических величин в системах СИ и СГС Величина Единица СИ СГС Отно- шение ед, СИ ед. СГС Длина Время Угол Площадь Объем Скорость Ускорение Частота колебаний . . . . Круговая частота колебаний Угловая скорость Угловое ускорение . . . . Масса Плотность Сила Давление Работа, энергия Мощность Импульс Импульс силы Момент силы Момент импульса Момент инерции Импульс момента силы . . рад м2 м3 м/с м/с2 Гц с-1 рад/с рад/с2 кг кг/м3 Н Па Дж Вт кг • м/с Н с Н-м кг • м2/с кг-м2 Н-м-с см с рад см2 см3 см/с см/с2 Гц с-1 рад/с рад/с2 г г/см3 ДИН дин/см2 эрг эрг/с г • см/с дин • с дин-см г • см2/с г-см2 дин¦см • с Ю2 1 1 ,104 10е НО2 102 1 1 1 1 103 ю-3 ДО5 10 107 107 105 105 107 107 107 107 10. Десятичные приставки к названиям единиц Т —тера A012) Г —гига A09) М —мега A06) к — кило A03) г — гекто A02) да — дека A01) д —деци (Ю-1) с —санти (Ю-2) м — милли (К)-3) мк — микро A0~в) н —нано (Ю-9) п —пико (Ю-12) ф —фемто A0-15) а —атто (Ю-18) Примеры: нм — нанометр A0~9 м), кН — килоньютон A03 Н) МэВ — мегаэлектронвольт A06 эВ), мкВт — микроватт (Ю^6 Вт). 245
11. Некоторые внесистемные единицы Длина . . . Время . . . Масса . . . Сила . . . . Давление . . Энергия . . Мощность о , Л А (ангстрем) = 10-10 м \1 а. е. (астрономическая единица) « 1,5 Л О11 м 1 св. год (световой год) «0,95-1016 м 1 пк (парсек) —3,1 • 101в м . Л сут (сутки) =86 400 с 1 г (год)=ЗЛ6-107 с . Л а. е. м. (атомная единица массы) = 1,66X ХЮ7 кг 1 т (тонна) = 103 кг . . 1 кгс (килограмм-сила)—9,81 Н . Л бар=105 Па (точно) 1 атм= 1,013- 10s Па 1 мм рт. ст.= 133 Па . .1 эВ=1,60-10-1в Дж 1 Вт-ч = 3,6-103 Дж . 1 л. с. (лошадиная сила) =736 Вт 12. Астрономические величины Солнце Земля Луна Масса, кг 1,99 Л О30 5,98- 1024 7,35 Л О22 Средний радиус, м 6,96- 108 6,37 ЛО6 1,74- 106 Средний радиус, орбиты, м 1,50- Ю11 3,84- 108 13. Физические Скорость света в вакууме Гравитационная постоянная Ускорение свободного падения Постоянная Авогадро Элементарный заряд Масса покоя электрона Удельный заряд электрона Масса покоя протона Атомная единица массы 1 а. е= те= е те тр = е. м.= постоянные с=2,998 Л О8 м/с Y = 6,67 Л О-11 м3/(кгс2) § = 9,807 м/с2 ЛГА = 6,022Л023 моль-' / 1,602Л0-19 Кл 1 4,80 Л О0 СГСЭ Г 0,91 МО-30 кг 1 0,511 МэВ / 1,76 Л0п Кл/кг ~\ 5,27 Л О17 СГСЭ 1,673 Л О-27 кг Г 1,660ЛО7 кг 1 931,4 МэВ
ПРЕДМЕТНЫЙ УКАЗАТЕЛЬ Вектор аксиальный 19 — перемещения 10 — полярный 19 Вес 44 Время собственное 135 Гироскоп 159 Градиент 94 Движение вращательное 18, 150 — плоское 21, 154 — поступательное 17 — тела переменной массы 76 Диаграмма импульсов вектор- ная 118 — Минковского 201 — пространства — времени 201 Длина собственная 188 Задача Кеплера 239 Закон всемирного тяготения 43 — Гука 45 — Ньютона второй 40 первый 35 — — третий 41 — сохранения импульса 68 момента импульса 140 энергии 100, 109—111 Законы сохранения 63 Замедление времени 185 Изотропность пространства 36 Импульс 65 — момента силы 136 — силы 66 Импульс релятивистский 212 — системы 67 Интервал 197 — времениподобный 198 — пространственноподобный 198 — светоподобный 198 Килограмм (эталон) 241 Кинематика твердого тела 1'7 — точки 10 Координата дуговая 14 Линия мировая 20il Лоренц-фактор 223 Масса 38 — покоя 211 — приведенная 114 — релятивистская 212 Метр (эталон) 241 Момент гироскопический 161 — импульса 134 относительно оси 137 системы 138, 139 собственный 146 — инерции 150 — силы 133 относительно оси 138 Мощность 88 Напряженность поля 96 Одновременность 180, 194 Однородность времени 36 — пространства 36 Опыт Майкельсона 175 Оси тела главные 157 — свободные 157 Ось вращения мгновенная 24 Параметр прицельный 120 Парадокс близнецов 187 Пара сил 144 Плечо импульса 133 — пары 145 — силы 134 Поверхность эквипотенциаль- ная 95 Поле стационарное 89 Порог 121 Постулаты Эйнштейна 177 Потенциал 97 Потенциальный барьер I01 Преобразования Галилея 37, 173 — Лоренца 192 — импульса и энергии реляти- вистской 223 — скорости 25, 26 релятивистской 199 — ускорения 25—28 Прецессия гироскопа 15(9 Принцип дальнодействия 42 — относительности Галилея 36 Эйнштейна 177 — суперпозиции 41, 97 — эквивалентности 53 247
Приращение 6, 92 Работа 85 — при вращение твердого те- ла 1154 — упругой силы 86 — гравитационной силы 86 — однородной силы тяжести 87 Секунда (эталон) 241 Сила 39 — внешняя 66 — внутренняя 66 ¦— гироскопическая 161 Сила гравитационная 43 ¦— диссипативная 107 — инерции центробежная 50 ¦— консервативная 89, 104 — Кориолиса 50 — кулоновская 44 — Лоренца 231 -— нормальная 48 — равнодействующая 149 — реактивная 77 — сопротивления 45 — сторонняя 99, 111 1 — тангенциальная 48 •— трения 45 — тяжести 44 — упругая 45 —• центральная 90 Симметрия пространства и времени 35 Синхронизация часов 179 Система замкнутая 68 — консервативная 109 •— отсчета 7 — — гелиоцентрическая 35 — •— инерциальная 35, 49 — центра масс 75 Скорости космические 127 Скорость точки 15 — угловая 19 прецессии 154 — центра масс 72 Событие 178 События причинно-связанные ,Ш4, 198 Сокращение лоренцево 188, 196 Столкновение абсолютно неуп- ругое 115 упругое 116 — лобовое 116 — нелобовое 117 — неупругое 120 — экзоэнергетическое 120 — эндоэнергетическое 120 Схема энергетическая ядерной реакции 234 Теорема Штейнера ,146, 231 Точка мировая 201 Убыль 6, 92 Уравнение движения центра масс 73 — Мещерского 77 — моментов 133 в Д-системе 147 — релятивистской динамики основное 214 —¦ ньютоновской динамики ос- новное 45 Ускорение 11 — кориолисово 27 — нормальное 16 — осестремительное 27 — тангенциальное 16 — угловое 19 Условия начальные 111 — равновесия твердого тела 150 Центр масс 71 Д-система 75 ч Частица с нулевой массой по- коя 221 Часы «световые» 184 Энергия внутренняя ,1|13 — кинетическая 98 твердого тела 153, 156 релятивистская 216 — покоя (собственная) 219 — пороговая il22 Энергия потенциальная 91 «внешняя» 105, Ш — — собственная 102, 103, 104, 105 — распада 229 — реакции ядерной 234 Эффект гироскопический 161 — Доплера 206 поперечный 207 продольный 206
ОГЛАВЛЕНИЕ Предисловие 5 Система обозначений 6 Введение 7 Часть I Ньютоновская механика Глава /.Основы кинематики 10 § 1.1. Кинематика точки 10 § 1.2. Кинематика твердого тела 17 § 1.3. Преобразования скорости и ускорения при переходе к другой системе отсчета 25 Задачи 28 Глава 2. Основное уравнение динамики 34 § 2.1. Инерциальные системы отсчета 34 § 2.2. Основные законы ньютоновской динамики 38 § 2.3. Силы 43 § 2.4. Основное уравнение динамики 45 § 2.5. Неинерциальные системы отсчета. Силы инерции ... 49 Задачи 53 Глава 3. Закон сохранения импульса 63 §31.0 законах сохранения 63 § 3.2. Импульс системы 65 § 3.3. Закон сохранения импульса 68 § 3.4. Центр масс. Д-система 7.1 § 3.5. Движение тела переменной массы 76 Задачи 78 Глава 4. Закон сохранения энергии 84 § 4.1. Работа и мощность 84 § 4.2. Консервативные силы. Потенциальная энергия .... 89 § 4.3. Механическая энергия частицы в поле 98 § 4.4. Потенциальная энергия системы 101 § 4.5. Закон сохранения механической энергии системы . . . 106 § 4.6. Столкновение двух частиц 1113 Задачи 122 Глава 5. Закон сохранения момента импульса 132 § 5.1. Момент импульса частицы. Момент силы 132 § 5.2. Закон сохранения момента импульса 138 § 5.3. Собственный момент импульса . . . 144
§ 5.4. Динамика твердого тела 148 Задачи 162 Часть II Релятивистская механика Глава 6. Кинематика специальной теории относительности . . 172 § 6.1. Трудности дорелятивистской физики 172 § 6.2. Постулаты Эйнштейна 177 § 6.3. Замедление времени и сокращение длины 181 § 6.4. Преобразования Лоренца 190 § 6.5. Следствия из преобразований Лоренца 193 § 6.6. Геометрическая интерпретация преобразований Лоренца 201 Задачи 203 Глава 7. Релятивистская динамика 209 § 7.1. Релятивистский импульс 209 § 7.2. Основное уравнение релятивистской динамики .... 2113 § 7.3. Закон взаимосвязи массы и энергии 216 § 7.4. Связь между энергией и импульсом частицы 220 § 7.5. Система релятивистских частиц 224 Задачи 230 Приложения 237 1. Движение точки в полярных координатах 237 2. О задаче Кеплера 239 3. Доказательство теоремы Штейнера 240 4. Греческий алфавит , 241 5. Основные единицы СИ в механике 241 6. Формулы алгебры и тригонометрии 242 7. Таблица производных и интегралов 243 8. Некоторые сведения о векторах 244 9. Единицы механических величин в СИ и СГС 245 10. Десятичные приставки к названиям единиц 245 1'1. Некоторые внесистемные единицы 246 12. Астрономические величины 246 13. Физические постоянные 246 Предметный указатель 247